CT Everything

Pataasin ang iyong marka sa homework at exams ngayon gamit ang Quizwiz!

Stochastic Effects

"Increased dose equals increased probability of effects" best describe which of the following?

A) Asthma C) Atopic Syndrome D) Cardiac Dysfunction ***Shellfish allergies are now recognized to be unreliable***

***MAY HAVE MORE THAN 1 CORRECT ANSWER*** Which of the following are contraindications for the administration of IV Contrast? A) Asthma B) Shellfish allergies C) Atopic syndrome D) Cardiac dysfunction

B) The maximum volume of each syringe is 200cc D) They have internal heating mechanisms to maintain the temperature of the contrast

***MAY HAVE MORE THAN 1 CORRECT ANSWER*** Which of the following statements are characteristics of power injectors? A) They are only used for CTA studies B) The maximum volume of each syringe is 200 cc C) They cannot be used to inject ionic contrast D) They have internal heating mechanisms to maintain the temperature of the contrast

A) It's important to establish a feeling of trust with the patient. B) It's important to have the patient explain the procedure instructions back to you. D) It's important to access the patients level of anxiety

***MAY HAVE MORE THAN 1 CORRECT ANSWER*** Which of the following statements are part of the patient interaction process? A) It's important to establish a feeling of trust with the patient B) It's important to have the patient explain the procedure instructions back to you C) Instructions should always be given orally, never use written material D) It's important to access the patients level of anxiety

B. tissue radiosensitivity Effective dose accounts for the type of tissue that the radiation is deposited in. Different tissues are assigned weighting factors on the basis of their individual radiosensitivity.

*Effective dose* is a relative radiation dose measurement term that accounts for the: a. beam sensitivity b. tissue radiosensitivity c. detector pitch d. collimation

B. 16 The number of data channels controls the number of sections the scanner can simultaneously acquire with each gantry rotation. A 16-slice system possesses 16 data channels and is capable of acquiring 16 images per gantry rotation. The configuration of the detector array, with regard to the size of individual detector elements, controls the possible section widths for each reconstructed CT image.

A 16 slice MDCT system utilizes an adaptive array of 24 detectors, each ranging in size from 0.625mm to 1.25mm. What is the maximum number of sections the system can acquire with each rotation of the gantry? a. 8 b. 16 c. 32 d. 64

B) Edge gradient ***Edge gradient artifacts occur because of the CT system's inability to process high spatial frequency signal required for sharp edges. When 2 densities of different atomic number are scanned in the same plane, such as barium and air in the stomach. The system sees them as one structure resulting in the blurring at the boarders or edges of the high frequency object***

A CT examination of the abdomen was performed on a patient complaining of left upper abdominal pain. Upon reviewing the image, a blurring occurred at the air/fluid level of the stomach. What was the possible cause of this artifact? A) Beam hardening B) Edge gradient C) Motion D) Cone beam

B) Edge gradient ***Edge gradient artifacts occur because of the CT system's inability to process high spatial frequency signal required for sharp edges. When 2 densities of different atomic number are scanned in the same plane, such as barium and air in the stomach. The system sees them as one structure resulting in the blurring at the borders or edges of the high frequency object***

A CT examination of the abdomen was performed on a patient complaining of left upper abdominal pain. Upon reviewing the image, a blurring occurred at the air/fluid level of the stomach. What was the possible cause of this artifact? A) Beam hardening B) Edge gradient C) Motion D) Cone beam

C) 1:1

A CT examination of the hip was performed with a table speed of 40 mm at a 0.5 second gantry rotation time using a 32 detector row and a 0.625 detector aperture, what was the pitch? A) 0.3:1 B) 0.5:1 C) 1:1 D) 1.2:1

C) 1:1

A CT examination of the hip was performed with a table speed of 40 mm at a 0.5 second gantry rotation time using a 32 detector row and a 0.625 detector aperture, what was the pitch?? A) 0.3:1 B) 0.5:1 C) 1:1 D) 1.2:1

A) Bone ***In order to visualize small bony detail or lesions in the spine a high-pass convolution kernel is needed for image reconstruction. This kernel is intended to sharpen small objects in an image by suppressing the low spatial frequencies in the image, improving the over all detail of the image***

A CT examination was performed on a patient presenting with lung cancer. The radiologist wants to determine if the disease metastasized to the patient's thoracic spine. Which of the following convolution filters would you use to retrospectively reconstruct the image for this appropriate interpretation? A) Bone B) Smooth C) Standard D) Ultra-smooth

A) Bone ***In order to visualize small bony detail or lesions in the spine a high-pass convolution kernel is needed for image reconstruction. This kernel is intended to sharpen small objects in an image by suppressing the low spatial frequencies in the image, improving the over all detail of the image***

A CT examination was performed on a patient presenting with lung cancer. The radiologist wants to determine if the disease metastasized to the patients thoracic spine. Which of the following convolution filters would you use to retrospectively reconstruct the image for this appropriate interpretation? A) Bone B) Smooth C) Standard D) Ultra-smooth

C. Pixels with values between -300 and +700 HU will be assigned shades of gray The width of a window determines the range of pixel values that will be assigned a shade of gray around a given level. In this example, all pixels within the range of -300 and +700 HU will be assigned shades of gray. Pixels below -300 HU will appear black, and pixels above +700 HU will appear white. This calculation is performed by dividing the width in half (1000/2 = 500) and subtracting and adding this value to the level (+200 HU + 500 = +700 HU; +200 HU - 500 = -300 HU).

A CT image is displayed in a window with a level of +200 and a width of 1000. Which of the following statements is correct? A) Pixels with values between +200 HU and -1000 HU will appear white B) Pixels with values greater than +200 HU will appear black C) Pixels with values between -300 and +700 HU will be assigned shades of gray D) Pixels with values between -1200 HU and +1200 HU will be assigned shades of gray

B. pixels with values between -250 HU and +250 HU will be assigned shades of gray The width of a window determines the range of pixel values that will be assigned shades of gray around a given level. In this example, pixels within the range of: -250 to +250 = assigned shades of gray -250 = black +250 = white

A CT image is displayed in a window with a level of 0 and a width of 500. Which of the following statements is correct? A) Pixels with values between 0 HU and 500 HU will appear white B) Pixels with values between -250 HU and +250 HU will be assigned shades of gray C) Pixels with values greater than +500 HU will be black D) Pixels with values less than 0 HU will appear white

C. matrix The back-projection method of image reconstruction involves the acquisition of attenuation values, which are then projected back onto a matrix for subsequent display.

A CT image is formed in part by projecting back all of the attenuation values recorded during data acquisition onto a: a. pixel b. voxel c. matrix d. reformat

C. matrix The back-projection method of image reconstruction involves the acquisition of attenuation values, which are then projected back onto a matrix for subsequent display.

A CT image is formed in part by projecting back all of the attenuation values recorded during data acquisition onto a: a. pixel b. voxel c. matrix d. reformat

C. 0.78 mm The dimension of a pixel may be calculated by dividing the field of view by the matrix size. The display field of view (DFOV) of 40 cm must first be converted into 400 mm. This is then divided by 512 mm, for a pixel dimension of 0.78 mm (400/512). Keep in mind that the pixel is a square, two-dimensional item and that the measurement of 0.78 mm corresponds to only one side.

A CT image is reconstructed using a 512^2 matrix and a display field of view of 40 cm. What is the linear dimension of each pixel? a. 0.0015 mm b. 0.08 mm c. 0.78 mm d. 1.28 mm

C. 36 mL The most common formula for calculating the dose of contrast material for pediatric studies is 2 mL/kg

A CT of the abdomen and pelvis is ordered for a child wearing 18 kg. Using the standard pediatric formula, what dose of iodinated contrast medium is given? a. 9 mL b. 18 mL c. 36 mL d. 54 mL

D) Cross field uniformity (homogeneity) ***the terms uniformity or homogeneity can be interchanged with their simplest meanings are the same; therefor the foal of uniformity is to maintain a constant CT value for water***

A CT phantom was scanned with the resulting stating the CT numbers of the uniform phantom were identical throughout the phantom. What QA test was performed? A) Spatial resolution B) Low contrast resolution C) Linearity D) Cross field uniformity (homogeneity)

C. without IV contrast administration IV contrast administration is not necessary for all thoracic indications because the structures of the thorax possess high intrinsic natural contrast.

A CT study done for the screening, detection or exclusion of pulmonary nodules or infiltrates are typically done: A) using multiple axial-mode scans, so that the patient must only hold their breath for a few seconds at a time B) using a wide slice thickness (>8mm) so the scan can be completed quickly, in one breath-hold C) without IV contrast administration D) first without IV contrast administration, followed by a repeat series with IV contrast administration

A. -956 HU The CT number of a pixel may be calculated by subtracting the linear attenuation coefficient of water from the linear attenuation coefficient of the tissue within the voxel (0.008 - 0.181 = -0.173). This number is divided by the linear attenuation coefficient of water (-0.173 / 1.81 = -0.956). The quotient is multiplied by a contrast factor of 1000 to yield the value of the pixel in Hounsfield units.

A CT system measures the average linear attenuation coefficient of a voxel of tissue to be 0.008. The linear attenuation coefficient of water for this scanner equals 0.181. The CT number assigned to the pixel representing this voxel of tissue equals: a. -956 HU b. -173 HU c. +44 HU d. +1044 HU

D) 76.8 mm ***total beam collimation = number of rows X aperture size total beam collimation = 128 X 0.6 total beam collimation = 78.6

A CTA of the heart requires 128 detector row channels and each detector element is 0.6 mm, what is the total beam collimation? A) 213.3 mm B) 76.8 cm C) 213.3 cm D) 76.8 mm

D) 76.8 mm ***total beam collimation = number of rows X aperture size*** total beam collimation = 128 X 0.6 total beam collimation = 78.6

A CTA of the heart requires 128 detector row channels and each detector element is 0.6 mm, what is the total beam collimation? A) 213.3 mm B) 76.8 cm C) 213.3 cm D) 76.8 mm

A) One in which all sides are equal ***The term isotropic is described as uniformity in all orientations. For a CT image this means the X, Y, and Z planes must have the same dimensions.***

A CTA study was performed on a patient suspected of having an abdominal aneurysm. The 3D technologist requested the voxels to be isotropic to eliminate loss of detail. Which of the following constitutes an isotropic voxel? A) One in which all sides are equal B) One in which the slice thickness is larger than the pixel area C) One in which the image needs to be reconstructed on a smaller matrix D) One in which the pixel size is smaller than the slice thickness

A) One in which all sides are equal ***The term isotropic is described as uniformity in all orientations. For a CT image this means the X, Y, and Z planes must have the same dimensions.***

A CTA study was performed on a patient suspected of having an abdominal aneurysm. The 3D technologist requested the voxels to be isotropic to eliminate loss of detail. Which of the following constitutes an isotropic voxel? A) One in which all sides are equal B) One in which the slice thickness is larger than the pixel area C) One in which the image needs to be reconstructed on a smaller matrix D) One in which the pixel size is smaller than the slice thickness

D. hemangioma A congenital, benign mass containing blood-filled spaces. It is commonly found in the liver and spleen.

A ___________ is a benign, highly vascular mass commonly found in the liver. a. hematoid b. vasculoma c. hemogenic carcinoma d. hemangioma

C. evidence of renal impairment or dehydration Most institutions set guidelines as to the upper limit of contrast media that can routinely be given (typically 64 grams of iodine). However, the guidelines are typically quite cautious, and specific circumstances (such as equipment malfunction) may necessitate exceeding the limit. A radiologist will determine whether the guidelines can safely be exceeded by considering individual factors, such as a patient's level of hydration and renal function. *Several studies have shown a direct correlation between the volume of contrast administered and the risk of contrast-induced nephropathy.*

A bolus injection of 125 mL of iodinated contrast media with a concentration of 320mg of iodine/mL is administered to a 150-pound patient, but because of an unexpected equipment malfunction scan data could not be acquired. The decision must be made as to whether the patient should be moved to a functioning scanner within the department and the contrast injection repeated. What information will be particularly important for the radiologist making the decision? A) the amount of radiation the patient received from the failed attempt B) the patient's history of allergy to any food or medication C) evidence of renal impairment or dehydration D) the brand of the contrast media being used

A. all of the pixels within the region have identical value The standard deviation indicates the amount of CT number variance within the ROI.

A circular ROI is defined and placed on an image, and the resultant measurement is 3 HU. The standard deviation is 0. It can be surmised that: A) all of the pixels within the region have identical value B) the area measured is composed of a variety of very different tissue types C) the area is likely a calcified nodule D) the patient is obese

C. 2 mL per kg of body weight Dosage calculations for intravenous contrast agent administration should adhere to the general guiding principle of injecting the smallest quantity of contrast agent possible to meet the technical requirements of the examination. The maximum dose for any procedure should not exceed 2 mL per kg of body weight.

A common formula used to calculate the maximum dosage of intravenous iodinated contrast material used in any CT examination is: a. 5 mg per kg of body weight b. 5 mL per lb of body weight c. 2 mL per kg of body weight d. 1 mL per lb of body weight

D. Antecubital space Contains the basilic vein medially, and the cephalic vein laterally.

A common site for the IV administration of iodinated contrast media is the anterior recess of the elbow, otherwise known as the: a. brachial fossa b. olecranon fossa c. retroulnar space d. antecubital space

D. Thin section (1 - 3 mm) axial and coronal sections These specific sections are required to properly demonstrate the orbits. Isotropic MDCT axial-plane acquisition will yield multiplanar reformation images of sufficient quality to eliminate the need for an additional scan in a second (coronal) plane). Acquiring data in only a single plane will reduce patient dose.

A complete CT study of the orbits should include: A) Thick section (5 - 10 mm) axial images only B) Thick section (5 - 10 mm) axial and coronal images C) Thin section (1 - 3 mm) coronal sections only D) Thin section (1 - 3 mm) axial and coronal sections

A. Increased patient dose Any reduction in the focus-to-detector distance, based on the design of MDCT systems, would reduce the distance between the source (x-ray tube) and the patient positioned in the isocenter of the CT gantry. This reduced distance would cause an increase in the dose to the patient, according the inverse square law.

A decrease in the focus-to-detector distance of a multidetector CT (MDCT) system would result in: a. increased patient dose b. decreased patient dose c. magnification of the image d. loss of contrast resolution

C. ratio of the maximum signal measured to the minimum signal the detector can measure Choice a: the response time of the detector Choice b: the detector absorption efficiency Choice d: refers to the brief, persistent flash of scintillation that occurred with older detector materials

A detector's dynamic range is the: A) time that is required for the signal from the detector to return to zero B) number of photons absorbed by the detector C) ratio of the maximum signal measured to the minimum signal the detector can measure D) amount of afterglow exhibited

C) Two-dimensional square of data ***A pixel is a two-dimensional picture element that represents the smallest discrete block in a digital image display field. Each pixel representing a brightness level in the reconstructed image based on the atomic number and mass density of the tissue***

A digital image is composed of multiple pixels. What is considered a characteristic of a pixel? A) Cube-shaped volume of tissue B) Unit of linear attenuation C) Two-dimensional square of data D) Unit of x-ray exposure in air

Matrix

A digital image is composed of rows and columns known as a:

C. 180 degrees First generation CT scanners were based on a translate-rotate principle. The x-ray tube and detectors would translate across the patient's head and then rotate 1 degree. This process would repeat in a semicircular fashion, for 180 degrees around the patient's head.

A first-generation CT scanner consists of an x-ray tube and two detectors that translate across the patient's head while rotating in 1-degree increments for a total of: a. 45 degrees b. 90 degrees c. 180 degrees d. 360 degrees

C. Thin-section (0.5 - 2.0 mm) helical acquisition specific to the larynx should extend from just above the hyoid bone inferiorly through the cricoid cartilage.

A focused, thin-section axial CT acquisition through just the region of the vocal cords should extend: A) from the external auditory meatus inferiorly to the mandible B) from the hard palate inferiorly to the hyoid bone C) from just above the hyoid bone inferiorly through the cricoid cartilage D) from the cricoid cartilage inferiorly through sternoclavicular joint

D. source collimator The source collimator is located near the x-ray source and limits the amount of x-ray emerging to thin ribbons. Because it acts on the x-ray beam before it passes through the patient it is sometimes referred to as a prepatient collimation.

A hardware component of a CT system that resembles small shutters with an opening that adjusts, dependent on the operator's selection of slice thickness, is a A) bow-tie filter B) slip ring C) reference detector D) source collimator

A. scatter radiation Postpatient collimation occurs through a high-resolution comb placed over the detector array. Functioning like a grid, it removes unwanted scatter radiation and off-axis photons that result from the more divergent nature of the MDCT beam.

A high-resolution comb is utilized by a MDCT detector array in an effort to reduce: a. scatter radiation b. patient radiation dose c. low spatial frequency signal d. high spatial frequency signal

Poor spatial resolution

A low signal to noise ratio (SNR) provides an image with:

B) Relationship between the field-of-view and the pixel size ***As long as the FOV and the pixel size are known, the matrix size can be calculated matrix=FOV/ pixel size***

A matrix can be described by which of the following characteristics? A) Aperture size used during data acquisition B) Relationship between the field-of-view and the pixel size C) relationship between the field-of-view and the reconstruction algorithm D) Volume of tissue

B) Relationship between the field-of-view and the pixel size ***As long as the FOV and the pixel size are known, the matrix size can be calculated matrix=FOV/ pixel size***

A matrix can be described by which of the following characteristics?? A) Aperture size used during data acquisition B) Relationship between the field-of-view and the pixel size C) relationship between the field-of-view and the reconstruction algorithm D) Volume of tissue

C. overbeaming When the cone beam of a MDCT system extends beyond the width of the detector array along the z axis.

A modern CT system may employ complex complex collimation along the z-axis to reduce patient radiation exposure due to: a. binning b. beam hardening c. overbeaming d. interpolation

A. 0.69 mm^3 The linear dimension of the pixel must first be calculated by dividing the display field of view (DFOV), in mm, by the matrix (380 mm / 512 = 0/74 mm). The linear pixel dimension is squared to yield the pixel area in mm^2 (0.74 mm x 0.74 mm = 0.55 mm^2). The volume of the voxel may be calculated by multiplying the pixel area by the section width (0.55 mm^2 x 1.25 mm = 0.69 mm^3).

A multidetector CT (MDCT) image is reconstructed using a 512^2 matrix and a display field of view (DFOV) of 38 cm. If the detector collimation is set to a section width of 1.25 mm, what is the volume of each voxel? a. 0.69 mm^3 b. 0.93 mm^3 c. 1.26 mm ^3 d. 1.68 mm^3

B. displaying the white and gray matter of the brain Tissue types with similar densities, such as the white and gray matter of the brain, should be displayed in a narrow window width. Wider window settings are used to suppress image noise and to reduce the appearance of streak artifact. Wider window widths encompass greater anatomic diversity (such as aerated lung), but subtle density discrimination is lost.

A narrow window width is best for A) displaying the aerated lungs B) displaying the white and gray matter of the brain C) suppressing noise on an image D) reducing the appearance of streak artifact

B. 60 - 90 mm Hg Diastolic is the bottom (Down) number in blood pressure: 120/80 Diastolic pressure is the measurement of blood pressure at its lowest point, in between contractions.

A normal range for diastolic blood pressure in an adult is: a. 40 - 60 mm Hg b. 60 - 90 mm Hg c. 80 - 120 mm Hg d. 95 - 140 mm Hg

C) Contiguous interval ***Is one in which the thickness of the slice and the pacing between two slices are equal***

A particular CT examination requires the slice thickness to be 5 mm and the spacing between each slice to be 5 mm, what type of reconstruction interval is the considered? A) Overlapping interval B) Prospective interval C) Contiguous interval D) Gapping interval

D. 1, 2 and 3 -- tachycardia, rapid and shallow breathing, and cyanosis General signs and symptoms that a patient is going into shock include rapid breathing, tachycardia, hypotension, weak pulse, pallor, cyanosis, and cold, clammy skin.

A patient in shock may exhibit which of the following symptoms? 1. tachycardia 2. rapid, shallow breathing 3. cyanosis a. 1 only b. 1 and 2 only c. 1 and 3 only d. 1, 2, and 3

B. nuclear medicine thyroid scan The iodinated contrast material used for the CT neck scan can interfere with the results of a radioactive thyroid uptake. Therefore, CT studies with iodinated contrast material should not be scheduled sooner than 2 weeks before the nuclear medicine thyroid study.

A patient is being scheduled for a CT examination of the neck. The patient also needs a number of other diagnostic examinations. Which of the following examinations may affect the date schedule for the CT study? a) upper GI b) nuclear medicine thyroid scan c) MR abdomen and pelvis examination d) doppler ultrasound of the lower extremity

A. a CT examination using a pulmonary embolism protocol is done Allergy to foods that contain iodine (e.g. seafood) does not increase the risk of intravenous contrast medium reactions. Seafood allergy results from hypersensitivity to a protein within the seafood and has no association with iodine. Assuming the history does not reveal any contraindications of iodine, the examination should proceed as ordered.

A patient is being seen in the emergency department for acute respiratory distress. Pulmonary emboli are suspected and a CT is ordered. A review of the patient's history reveals that he is allergic to shellfish. What action should be taken? A) a CT examination using a pulmonary embolism protocol is done B) a CT examination of the chest, without contrast enhancement, is done C) the CT examination is cancelled; most likely it will be replaced by a nuclear medicine V/Q scan D) the patient is given steroids and the CT examination is delayed for 6 hours

B. Lethargic The patient appears drowsy but can be aroused.

A patient who appears drowsy but can be aroused is said to be: a. comatose b. lethargic c. obtunded d. semicomatose

A. the pressure within the arterial vessels during contraction of the heart The top number in blood pressure: systolic The bottom number in blood pressure: diastolic Systolic measures the pressure exerted on the arterial walls during a heart contraction.

A patient's blood pressure is measured as 140/70 mm Hg. The number 140 represents: A) the pressure within the arterial vessels during contraction of the heart B) the pressure exerted on the chambers of the heart while it is relaxed C) the pressure within the arterial vessels while the heart is relaxed D) the pressure exerted on the chambers of the heart during a contraction

A. network A computer network is a communication system designed to facilitate the transfer of data between computers. A picture archival and communications system (PACS) works on a network, which connects each of the involved imaging modalities, viewing stations, printers and so on.

A picture archival and communications system (PACs) is connected with imaging modalities and other peripheral devices on a computerized communications system called a: a. network b. web c. radiology information system (RIS) d. hospital information system (HIS)

B. 1 and 3 only -- provide precise anatomic location of tumor activity and provide attenuation correction information Positron emission tomography (PET) provides a map of normal and abnormal tissue function. PET offers excellent sensitivity for malignancy, but suffers from an inability to accurately localize tumor activity. Fusion of PET with CT provides such precise anatomic localization. CT data is also used to correct the differences in attenuation that occur as fluorodeoxyglucose (FDG) decay occurs throughout the patient. Decay photons emerging from the center of the patient will be attenuated more than those from the periphery of the patient. CT data during a PET-CT is used to correct this potential artifact.

A primary responsibility of computed tomography in combined PET-CT is to: 1. provide precise anatomic location of tumor activity 2. reduce patient radiation dose 3. provide attenuation correction information a. 1 only b. 1 and 3 only c. 2 and 3 only d. 1, 2 and 3

C. 2 and 3 only -- increased electronic noise and decreased tube output The scanner's reduced low-contrast resolution is most likely the cause of increased noise. When the noise level of a CT image increases, the low-contrast resolution decreases. Decreased patient radiation dose implies that the signal-to-noise ratio has decreased, thus raising the noise level of the image. The same situation would apply for an increase in electronic noise.

A quality control procedure determines that the low-contrast resolution of a CT scanner is extremely poor. Likely causes are: 1. tube arcing 2. increased electronic noise 3. decreased tube output a. 1 only b. 1 and 3 only c. 2 and 3 only d. 1, 2 and 3

Genetically significant dose

A radiation dose that if received by the entire population would cause the same genetic injury as the total of doses received by the members actually being exposed is called:

D. average CT number (HU) The region of interest (ROI) measurement provides a quantitative analysis of the Hounsfield values of a specific anatomic area. System software calculates the average CT number in HUs within the defined ROI. The average ROI measurement provides information regarding tissue characteristics that may be helpful for clinical diagnosis.

A region of interest (ROI) measurement placed over a portion of a CT image provides which of the following? a. distance (mm) b. diameter (mm) c. linear attenuation coefficient (u) d. average CT number (HU)

B. fatty infiltrate of the liver Fatty infiltration reduces the CT attenuation of the involved liver. *With fatty infiltration, the liver is at least 10HU lower than that of the spleen.*

A set of ROIs, one placed within the liver and one placed within the spleen, is often used to document: A) splenomegaly B) fatty infiltrate of the liver C) sickle cell disease D) lymphoma

A. CT enteroclysis A specialized evaluation of the small bowel whereby a nasogastric catheter is placed into the duodenum under fluoroscopic guidance. Then 1.5 to 2.0 L of enteral contrast agent is administered directly into the small intestine for maximal opacification. Non-IV contrast, thin-section (0.6 - 1.25 mm) CT images are acquired through the abdomen and pelvis.

A specialized CT examination involving the administration of an enteral contrast agent directly into the small bowel via nasogastric tube is called: a. CT enteroclysis b. CT colonography c. CT enterography d. CT colonoscopy

A. 0.5 MB Each CT image reconstructed on a 512^2 matrix is approximately 0.5 megabytes (MB) in digital size.

A standard CT image reconstructed on a 512^2 matrix has a digital size of approximately: a. 0.5 MB b. 3.0 MB c. 5.0 MB d. 10.0 MB

Standard precautions

A system that uses barriers between individuals and assumes all patients are infectious is called:

Algorithm

A term used to describe the mathematical formula used by the computer to reconstruct the image is:

D. 2.5 mL/sec Flow rate calculated by dividing the total contrast volume by the time of the injection. 125mL/50 sec = 2.5mL/sec

A total volume of 125 mL of an iodinated contrast agent is administered intravenously via automatic injector in 50 seconds. The flow rate for this injection is: a. 0.75 mL/sec b. 1.25 mL/sec c. 1.75 mL/sec d. 2.5 mL/sec

B. nonionic Nonionic contrast materials do not dissociate into charged particles (ions) when placed in solution. Ionic contrast materials are salts that form independent particles in aqueous solutions.

A(n) ___________ contrast material may be described as one that does not dissociate into charged particles in solution. a. neutral b. nonionic c. osmolar d. ionic

B. nosocomial Commonly referred to as health care-associated infections. They occur within the first 48 hours of a patient's stay in a health care facility.

A(n) ____________________ infection is one that a patient acquires during a stay in a health care institution. a. blood-borne b. nosocomial c. iatrogenic d. staphylococcal

Which of the following is the term utilized for defined set of instructions for solving a problem? A) Algorithm B) Fourier transform C) Interpolation D) Convolution

A) Algorithm ***In CT an algorithm is a process or set of step-by-step rules to be followed in calculations or other problem-solving operations by the computer.***

What type of artifacts can arise from insufficient projection sampling or from insufficient view sampling? A) Aliasing B) Motion C) Beam hardening D) Interpolation

A) Aliasing ***Image quality depends on the number of projections to reconstruct an image. If not an adequate number of samples are collected, it results in misregistration by the computer of information relating to sharp edges and small objects. This artifact appears as evenly spaced lines that are easy to distinguish from anatomic structures and seldom render an image undiagnostic.***

The left subclavian artery is a branch of which of the following artery? A) Arch of the aorta B) Descending aorta C) Ascending aorta D) Brachiocephalic trunk

A) Arch of the aorta ***The left subclavian artery is the first branch of the aortic arch from left to right and is posterior to the left common carotid artery. This artery like it's counterpart the right subclavian artery, arches toward the axilla, and continues as the axillary artery.***

What is described as any discrepancy between the reconstructed CT numbers in the image and the true attenuation coefficients of the object? A) Artifact B) Noise C) Spatial resolution D) Low-contrast detectability

A) Artifact ***Occurs when part of the contents of a CT image does not represent the physical object being imaged. They can appear as false structures or corruption of the CT numbers.***

Following the injection of 100cc of iodinated contrast media a patient exhibits a severe vagal reaction that includes bradycardia. Which of the following drugs would be used for the initial treatment of this condition? A) Atropine B) Epinephrine C) Ranitidine hydrochloride (Zantac) D) Albuterol sulfate (Proventil)

A) Atropine ***Bradycardia refers to a decrease in heart rate for which the drug atropine is the initial treatment of choice. Atropine is an anticholinergic drug that acts as a vasoconstrictor, which increases the patients heart rate.***

What is described as reduction of the intensity of a beam of radiation as it passes through an object? A) Attenuation B) Filtration C) Collimation D) Kerma

A) Attenuation

If a hepatic tumor is said to be hypervascular, it enhances during which phase of injection? A) Bolus B) Venous C) Nonequilibrium D) Delayed

A) Bolus ***The arterial or bolus phase of injection is used to visualize hypervascular lesion such as hepatomas, carcinoid metastases, and focal nodular hyperplasia, mainly because these lesions are supplied by the hepatic artery.**

What is the term used to describe the instrumentation and method used to measure patient dose from a CT scanner? A) CT dosimetry B) CT absorptiometry C) Radiation intensity D) Dose distribution

A) CT dosimetry ***Is the term used to described the instrumentation and methods of computing radiation exposure and the effects it has on the technologist and the population. It's also a method to compare their institutions doses with that of the national average, and can be utilized to educate hospital personal on the dose a patient received from a CT examination.***

With a window width of 1800 and a window level of -650, which of the following CT numbers would appear white in the image? A) CT numbers above 250 B) CT numbers below -1,550 C) CT numbers between 250 and -1,550 D) CT number above -1,550

A) CT numbers above 250 ***the equation for this is: WL+ WW/2 WL-WW/2 -650 +1800/2 -650 1800/2 -650+900-650-900 250-1550***

What is the main goal of computed tomography? A) Calculate the linear attenuation coefficients for image reconstruction. B) Increase superimposition C) Decrease image contrast D) Utilize thick sections to increase spatial resolution

A) Calculate the linear attenuation coefficients for image reconstruction ***The main goal in CT is to calculate the linear attenuation value along each ray to the source detectors. This is accomplished through the Lambert Beer Law, but taking into account the attenuation from the photoelectric absorption and Compton scatter.***

Which of the following pathological conditions is caused by the compression of the flexor retinaculum? A) Carpal tunnel syndrome B) Guyon's canal C) Avascular necrosis D) Kienbock disease

A) Carpal tunnel syndrome ***Occurs when there is compression of the median nerve and is associated with repetitive activity, such as typing.***

What technique is used to ensure optimum evaluation of the urinary tract? A) Compression device B) High mAs with low kVp C) High mAs with high kVp D) Thick slice acquisition

A) Compression Device ***Placed over the L5 region applies pressure to the ureters restricting the flow of contrast. A scan is performed, with the device inflated, from the kidneys to the iliac crest. Once the compression is released, the pelvic scan must be performed. This will demonstrate a urinary flow into the bladder.***

A CT image of homogeneous phantom is said to be uniform, what does this refer to? A) Constant CT numbers of an object B) Isotropic voxels C) Matrix detector array D) Accurate CT numbers

A) Constant CT numbers of an object ***The end results of the CT measurement is the CT number, calculated from the linear attenuation coefficient of the object, by scaling with the respective coefficient for pure water and given in HU. This is an important QA test.***

Which of the following laboratory values is a breakdown product of skeletal muscle? A) Creatinine B) GRF C) PT D) INR

A) Creatinine ***Is a breakdown product in skeletal muscle, and its rate of production is proportional to muscle mass. This test detects very early loss of kidney function before abnormal increase in serum levels of BUN or creatinine arise.***

Which of the following pathological processes has an attenuation value of near water and accumulates in the greater peritoneal space? A) Cyst B) Abscess C) Ascites D) Hemorrhage

A) Cyst ***Is the common term for the collection of serous fluid in the peritoneal cavity.***

Image magnification is a basic visualization tool integrated into the CT system. Which of the following is an effect of image magnification? A) Degrades the image display resolution B) Changes the apparent noise in the image C) Uses raw data to decrease the display FOV D) Uses the segmentation process to give the illusion of depth

A) Degrades the image display resolution ***Image magnification is a computer software program that changes the size of the pixels on the computer monitor. As the pixels resolution becomes larger, the image appears blurry on the display monitor.***

What measures the strength of a radiation field at some point in air? A) Exposure B) Absorbed dose C) Dose equivalent D) Dose rate

A) Exposure

What bone of the lower leg is situated most lateral in the anatomical position? A) Fibula B) Tibia C) Cuboid D) Navicular

A) Fibula

Which of the following statements is TRUE concerning sequential access storage? A) Files are faster to read and write if you always access records in the same order B) Use a disk drive to access files at any point on the disk without passing through all intervening points. C) Gives the operator the ability to read and write data in a random order D) Makes use of magnetic or floppy disk for storage

A) Files are faster to read and write if you always access records in the same order ***Sequential access is referred to as reading or writing data records in sequential order. Sequential-access files are faster to read and write if you always access records in the same order.***

What reconstruction algorithm was developed to eliminate the star pattern typically seen in back projection? A) Filtered back projection reconstruction B) Adaptive statistical iterative reconstruction C) Modeled-based iterative reconstruction D) Algebraic reconstruction

A) Filtered back projection reconstruction *** Is a type of reconstruction method known as the convolution method and was developed in order to eliminate the star pattern typical of the back projection method. This analytic method uses the Fourier Transform to convert a signal in the spatial location domain to a signal in the spatial frequency domain.***

What is the purpose of the taenia coli? A) Gathers the cecum and colon into pouch-like folds B) Attaches the intestine to the abdominal wall C) Sphincter muscle to control the flow of material from the ileum into the cecum D)Extracts water and salts from sold wastes before they are eliminated from the body

A) Gathers the cecum and colon into pouch-like folds ***Taenia coli is made up of three narrow but distinct longitudinal bands of smooth muscle running along the entire length the cecum and colon gathering them into pouch-like folds known as hastra. Taenia coli are not present on the rectum, anal canal, and vermiform appendix.***

What type of tumors show greater enhancement then the surrounding parenchyma during the arterial phase of injection? A) Hypervascular tumors B) Peripherally enhancing tumors C) Hypovascular tumors D) Complex cyst

A) Hypervascular tumors ***Have a good blood supply causing enhancement of the tumor as compared with the adjacent parenchyma and appear as a focus with transient hyperattenuation during the arterial or the bolus phase of injection.***

A radiologist is viewing the images of an abdomen and pelvis CT examination on the PACS workstation using a WW of 450 and a WL of 50, and notices there is a sense area of the patient's iliac crest. Which of the following adjustments can be made to make the image appear darker so that bony structures can be evaluated? A) Increase the window level B) Decrease the window level C) Increase the window width D) Decrease the window width

A) Increase the window level ***Window level controls the brightness of an image on the computer monitor. Other names for window level are window center and window length.***

Which of the following ligaments join the laminae of adjacent vertebral arches to preserve the normal curvature of the spine? A) Ligamentum flava B) Supraspinous ligament C) Anterior longitudinal ligament D) Transverse ligament

A) Ligamentum Flava ***Is located on either side of the spinous process consisting of strong yellow elastic tissue. Its purpose is to join the laminae of adjacent vertebral arches to preserve the normal curvature of the spine.***

Special considerations are taken with patients at risk for developing an adverse event following the administration of IV contrast media. Which of the following conditions would not put the patient at risk for an allergic reaction? A) Patients with shellfish allergies B) Patients who had prior reaction to contrast media C) Asthmatic patients D) Patients with low cardiac output

A) Patients with shellfish allergies ***Were once considered contraindications to IV contrast, but the predictive value of shellfish and diary allergies have proved to be unreliable. Along with there is not enough evidence to support the continuation of this practice.***

What pathological disorder presents with fluid in the posterobasal region forming a crescent-shaped collection that borders the chest wall? A) Pleural effusion B) Pulmonary embolism C) Pneumothorax D) Hemothorax

A) Pleural Effusion ***Is the abnormal collection of fluid between the pleural layers. Fluid within the pleural space will only be detected if the volume is 15mL or greater. A thoracentesis is useful in defining the underlying cause of the fluid.***

Which of the following correctly describes the position of the seminal vesicles in the male pelvis? A) Posterior to the bladder and anterior to the rectum B) Superior to the rectum and inferior to the prostate C) Anterior to the bladder and superior to the prostate D) Inferior to the bladder and posterior to the rectum

A) Posterior to the bladder and anterior to the rectum ***Found on either side of the ductus deferens.***

What pulmonary disease may be caused by an elevation of pulmonary venous pressure? A) Pulmonary edema B) Pulmonary embolism C) Pulmonary fibrosis D) Cystic fibrosis

A) Pulmonary edema ***Has two mechanical processes, an elevation of pulmonary venous pressure or an increased permeability of the alveolar capillary membrane. As the edema manifest, fluid leaks into the alveolar airspace from the interstitium tissue.***

What type of data are also known as scan data? A) Raw data B) Measurement data C) Image data D) Convoluted data

A) Raw data

The heart is divided into four chambers, which chamber of the heart receives blood from the SVC, IVC, cardiac sinus, and cardiac veins? A) Right atrium B) Left atrium C) Right ventricle D) Left ventricle

A) Right atrium ***The right atrium is the drainage point of the SVC, and IVC, which pumps venous blood into the right ventricle via the tricuspid valve. The right atrium is located in the mediastinum slightly superior to the right ventricle.***

When the SFOV is reduced for a particular image it results in what structure change? A) Spatial resolution B) Display image appears smaller C) Display image appears larger D) Fan angle increases

A) Spatial resolution ***As the SFOV decreases, going from a 500mm to a 250mm view, the fan angle decreases resulting in a reduction of radiation hitting the object outside the field. A narrow fan angle results in increased sampling rate which in turn improves spatial resolution.***

Which of the following superficial muscles of the vertebral column are located on the posterior and lateral aspect of the cervical and upper thoracic spine? A) Splenius muscles B) Erector spinae muscle C) Transversospinal muscles D) Gluteal compartment

A) Splenius Muscles ***Are located on the posterior and lateral aspect of the cervical and upper thoracic spine. They are divided into a cranial segment, the splenius capitis, and the cervical segment, the splenius cervicis. The function of these muscles are to extend the head and the neck.***

A sudden change in mental activity may indicate that a patient is experiencing a serious condition; which of the following level of consciousness can be described as- only vigorous repeated stimuli will arouse the individual? A) Stupor B) Lethargy C) Clouding D) Coma

A) Stupor ***Is the lack of critical cognitive function and a level of consciousness in which an individual is almost entirely unresponsive and only responds to base stimuli.***

Low contrast detectability can be defined by which of the following statements? A) The ability to distinguish region whose attenuation coefficient are very close to each other B) The ability to measure the smallest size structure that can be seen C) The ability to freeze motion D) The relationship between the linear attenuation coefficient and the HU

A) The ability to distinguish region whose attenuation coefficient are very close to each other. ***Contrast resolution can be described as the ability to distinguish one soft tissue from another with similar densities as their background. The term low-contrast detectability (LCD) is used to described contrast resolution in CT.***

What tubular-shaped organ is found in males is located inferior to the bladder extending through the prostate, urogenital diaphragm, and penis? A) Urethra B) Ureter C) Corpus cavernosum D) Corpus spongiosum

A) Urethra ***Is a tubular-shaped organ approximately 20cm in length that passes through the prostate gland, urogenital diaphragm, and penis. The function of the urethra is to drain the bladder and also to transmit sperm to the exterior during ejaculation.***

When the imaging the adrenal glands, the density of lipid-poor adenomas may resemble that of nonadenomas, since their kinetics are similar to lipid-rich adenomas. Which of the following equations would be used to determine the relative percentage washout? A) enhanced CT(HU) - delayed CT(HU) / enhanced CT(HU) X 100% B) delayed CT(HU) -enhanced CT(HU) x unenhanced CT(HU) C) unenhanced CT(HU) X enhanced CT(HU) / delayed CY(HU) X100% D) unenhanced CT(HU) - enhanced CT(HU) / delayed CT(HU) X 100%

A) enhanced CT(HU) - delayed CT(HU) / enhanced CT(HU) X100% ***The relative washout to determine the morphology of an adrenal lesion.***

a. 1 - 2 mm Thin sections are required to properly assess abnormalities of the minute structures of the inner ear. The thinnest sections possible provide maximum detail.

Abnormalities of the middle and inner ear may be best demonstrated with CT utilizing _______________ sections. a. 1 - 2 mm b. 3 - 4 mm c. 5 mm d. 10 mm

Cathode

According to the anode heel effect, the intensity of radiation is greater at the _____ side of the x-ray tube.

C. mediastinal lymphadenopathy Contrast enhancement greatly improves demonstration of lymphadenopathy within the chest in comparison with other mediastinal structures.

Accurate demonstration of ______________ would most likely require the intravenous injection of an iodinated contrast agent during a CT study of the chest. a. bronchioectasis b. pneumonia c. mediastinal lymphadenopathy d. pulmonary nodule

C. a dissecting aortic aneurysm Intravenous administration of an iodinated contrast agent is vital to making an accurate diagnosis of a dissecting aortic aneurysm. The contrast agent helps outline the wall of the aorta and improves visualization of any division within it. Because of differences in blood blow, CT number within the actual lumen of the aorta are different from those in the dissected portion.

Accurate demonstration of ____________________ would most likely require the intravenous injection of an iodinated contrast agent during a CT study of the chest. a. pneumonia b. asbestosis c. a dissecting aortic aneurysm d. a solitary pulmonary nodule

C. overranging Adaptive collimation technology may be employed in a modern CT system to reduce z-axis overscanning or overranging that may occur at the start and end of a helical acquisition.

Adaptive collimation may be uitilized to reduce the unnecessary radiation exposure at the start and end of a helical acquisition that may be referred to as: a. overbeaming b. interpolation c. overranging d. binning

D. 1, 2 and 3 -- reduction in required contrast dose, reduction in contrast-induced nephrotoxicity and reduction in streaking artifact in the vasculature The use of a saline flush also improves the overall contrast agent utilization efficiency of the CT examination.

Advantages of a saline flush immediately after the IV administration of iodinated contrast include: 1. reduction in required contrast agent dose 2. reduction in the incidence of contrast-induced nephrotoxicity 3. reduction of streaking artifacts from dense contrast agent in the vasculature a. 1 only b. 3 only c. 2 and 3 only d. 1, 2 and 3

Medical asepsis

After all radiographic or fluoroscopic procedures, the radiographer should use _____ to clean surfaces with which the patient was in contact.

B. 25 - 35 seconds The arterial phase is the period of peak arterial enhancement. During this phase, hypervascular tumors or tumors supplied by the hepatic artery are optimally enhanced.

After initiation of a rapid bolus administration of an iodinated contrast agent, the arterial phase of hepatic contrast enhancement occurs at approximately: a. 15 - 20 seconds b. 25 - 35 seconds c. 60 - 70 seconds d. 120 - 180 seconds

C. 60 - 70 seconds The portal (or hepatic) venous phase is the period of peak hepatic parenchymal enhancement, when contrast material redistributes from the blood into the extravascular spaces. The portal/hepatic venous phase occurs at approximately 60 - 70 seconds after the initiation of rapid bolus contrast agent administration.

After initiation of rapid bolus administration of an iodinated contrast agent, the portal venous phase of hepatic contrast enhancement occurs at approximately: a. 15 - 20 seconds b. 25 - 35 seconds c. 60 - 70 seconds d. 120 - 180 seconds

B. 35-45 seconds The pancreatic phase of contrast enhancement is a delayed arterial phase occurring approximately 35-45 seconds after the start of contrast administration, assuming an adequate volume injected at a rate of 3 mL/sec or greater.

After initiation of rapid bolus administration of iodinated contrast material, the pancreatic phase of contrast enhancement occurs at approximately: a. 15-25 seconds b. 35-45 seconds c. 60-70 seconds d. 120-180 seconds

C. 70 - 90 seconds The nephrographic phase occurs between 70-90 seconds after the start of injection. Enhancement of the renal cortex and medulla reaches equilibrium, providing optimal sensitivity for parenchymal lesions.

After initiation of rapid bolus adminsitration of an iodinated contrast agent, the nephographic phase of renal contrast enhancement occurs at approximately: a. 20-25 seconds b. 30-40 seconds c. 70-90 seconds d. 3-5 minutes

C. rest for 8 - 24 hours with the head slightly elevated The reduce the possibility of headaches after a CT examination involving an intrathecal injection of contrast material, the patient should rest for 8 - 24 hours with the head slightly elevated (35 - 45 degrees).

After the intrathecal injection of an iodinated contrast agent for a postmyelography CT study of the lumbar spine, the patine should be instructed to: A) take a cleansing enema B) resume normal activity C) rest for 8 - 24 hours with the head slightly elevated D) rest for 8 - 24 hours in the Trendelenburg position

D. minimum intensity projection (min-IPs) When using the minimum intensity projection (min-IP) technique, displayed pixels represent the minimum attenuation value encountered along each sampled ray. This type of multiplanar volume reconstruction (MPVR) can be applied to a volume of the thorax to demonstrate air trapping within the trachea and bronchial tree.

Air trapping within the trachea and bronchial tree may be optimally demonstrated with multiplanar volume reconstructions (MPVRs) of the thorax known as: a. maximum intensity projections (MIPs) b. 3D volume renderings c. shaded-surface displays (SSDs) d. minimum intensity projections (min-IPs)

A. at both inspiration and expiration CT imaging of the airways is most often performed in a helical mode, using thin sections and an overlapping z axis image reconstruction of 50% so that images can be postprocessed. Airway imaging is routinely performed at both inspiration and expiration.

Airway imaging is routinely performed: A) at both inspiration and expiration B) using a wide slice thickness (>8mm) C) using a low-contrast (i.e soft) algorithm D) using a representative technique in which only about 10% of the airway is imaged

Salts of organic iodine compounds

All iodinated contrast media used today contain:

A. to verify that the patient has not exceeded his radiation limit There is no established radiation limit for patients receiving diagnostic tests. A medical history can discover risk factors for contrast media administration, determine pregnancy status in women of childbearing age, verify that the appropriate examination protocol is selected, and provide the radiologist with diagnostic information that can help when interpreting the CT images.

All of the following are reasons for obtaining a history before a CT examination is performed EXCEPT: A) to verify that the patient has not exceeded his radiation limit B) to select the appropriate protocol C) to be sure the examination can be performed safely D) to aide the radiologist in his or her Interpretation of the CT examination

What does ACR stand for

American College of Radiology

What does ACR stand for?

American College of Radiology

D. 25 lp/cm The limiting resolution of a modern CT scanner is approximately 25 lp/cm. This resolution varies greatly with scan factors and is considerably less than that of projection radiography.

An accurate, modern CT scanner possesses a spatial resolution up to: a. 10 lp/mm b. 25 lp/mm c. 10 lp/cm d. 25 lp/cm

C. 60 mm The beam pitch for a given acquisition is equal to the table feed per rotation divided by the total collimation. The total collimation for this acquisition is equal to the total total number of sections (detectors) multiplied by the detector dimension, or 64 X 0.625 mm. The table feed per rotation can be calculated as the product of the total collimation and the beam pitch, or 40 mm X 1.5, and equals 60 mm.

An acquisition is made on a 64 slice multidetector CT (MDCT) system in which each detector element has a z-axis dimension of a 0.625 mm. With a selected beam width of 40 mm and the beam pitch set at 1.50, how much will the table move with each rotation of the gantry? a. 20 mm b. 40 mm c. 60 mm d. 80 mm

C. 125 mL A total contrast agent volume between 100 and 150 mL is sufficient for imaging the neck. The precise volume and administration technique vary from one institution to another.

An adequate volume of intravenous contrast agent for the single-slice CT (SSCT) examination of the neck in this figure would be: a. 25 mL b. 50 mL c. 125 mL d. 225 mL

A mathematical formula used to reconstruct the image in digital imaging

An algorithm is:

A. intracellular lipid content Although both adenomas and metastatic lesions enhance rapidly, because of differences in their intracellular lipid content, metastatic lesions retain the contrast longer.

An anatomic trait that can help to differentiate an adenoma from a malignant adrenal mass on CT scans is its: A) intracellular lipid content B) calcium content C) location D) iron content

C. +45 HU An average range of CT numbers for blood is +42 to +58. The CT number of any material is based on many factors, including the beam quality of a particular scanner.

An average CT number value for blood is: a. -20 HU b. +10 HU c. +45 HU d. +100 HU

C. +1000 HU The average CT number value for bone is +1000, however, this number may vary widely depending on the density of the particular bone and the beam quality of the CT scanner.

An average CT number value for bone is: a. +100 HU b. +500 HU c. +1000 HU d. +3000 HU

C. 25 - 35 seconds Partial thromboplastin time (PTT) is a laboratory measure of blood coagulation. This value is used to screen patients for invasive procedures. Because of individual laboratory practices, the average range may fluctuate slightly.

An average range for activated partial thromboplastin time (PTT) is: a. 10 - 12 seconds b. 17 - 21 seconds c. 25 - 35 seconds d. 43 - 55 seconds

B. Bladder An abnormal communication between the bowel and the ladder. An enterovesical fistula usually results from inflammatory processes of the colon such as diverticulitis.

An enterovesical fistula is an abnormal communication between the bowel and the: a. uterus b. bladder c. vagina d. umbilicus

D. increase the mA to 300 and increase the tube rotation time to 0.8 seconds Quantum mottle when there are an insufficient number of photons detected and results in image noise. Doubling the mAs of the study increases the number of photons detected per pixel by approximately 40%.

An image oft he abdomen is greatly degraded by quantum mottle. The scan parameters used are as follows: DFOV = 38cm; slice thickness = 5mm; mA = 240; tube rotation = 0.5 seconds; kVp = 120 What action will have the best chance of reducing or eliminating the quantum mottle for subsequent scans? A) increase DFOV to 44cm B) decrease slice thickness to 2mm C) decrease mA to 125 and increase the tube rotation time to 1.0 seconds D) increase the mA to 300 and increase the tube rotation time to 0.8 seconds

C. retrospective Retrospective reconstruction occurs when an image is reconstructed a second time with an adjustment in a technical factor. The scan data, or "raw" data, are used to reconstruct the image with a different display matrix, display field of view (DFOV), algorithm, and so on.

An image that is reconstructed a second time with some change in technical factors is said to be: a. reiterated b. postprocessed c. retrospective d. reformatted

A. views per rotation The sampling rate may be quantified as views per rotation (VPR). As the sampling rate rises, the number of VPR increases.

An increase in the sampling rate during CT data acquisition corresponds to an increase in: a. views per rotation b. patient radiation dose c. image noise d. matrix size

Characteristic

An interaction that produces x-rays at the anode as a result of outer shell electrons filling holes in the K-shell is called:

A. subarachnoid space Intrathecal injections of iodine are injected into the subarachnoid space, which is located between the arachnoid and pia mater. The subarachnoid space contains the cerebrospinal fluid.

An intrathecal injection before a CT examination of the lumbar spine administers iodinated contrast material directly into the: a. subarachnoid space b. dura mater c. vertebral foramen d. subdural space

A. Compton scatter Occurs when an incident x-ray photon interacts with an outer-shell electron of a target atom. The photon loses some energy in this ionization interaction and undergoes a resultant change in direction.

An x-ray photon may lose some of its energy in an interaction with an outer-shell electron of a target atom within the patient in the interaction known as: a. Compton scatter b. Rayleigh scatter c. photoelectric absorption d. pair production

C) Rectify the scatter issue with an increase cone beam ***Anti-scatter grids (ASG) were developed to rectify the scatter issue notably seen in detector system having 64 detectors. The x-ray beam becomes a cone leading to an increase of penumbra. These grids are placed in front of the detectors arranged in a 1D or 2D format system of thin lamellae made of strongly absorbing material***

Anti-scatter grids (ASG) have been incorporated in MDCT scanners. What is the purpose of these grids? A) Prevent cross-talk B) Define slice thickness C) Rectify the scatter issue with an increase cone beam D) Define the maximum permissible beam

C) Rectify the scatter issue with an increase cone beam ***Anti-scatter grids (ASG) were developed to rectify the scatter issue notably seen in detector system having 64 detectors. The x-ray beam becomes a cone leading to an increase of penumbra. These grids are placed in front of the detectors arranged in a 1D or 2D format system of thin lamellae made of strongly absorbing material***

Anti-scatter grids (ASG) have been incorporated in MDCT scanners. What is the purpose of these grids? A) Prevent cross-talk B) Define slice thickness C) Rectify the scatter issue with an increase cone beam D) Define the maximum permissible beam

B. high spatial frequencies Contrast of a CT image is controlled by the spatial frequencies of the tissue(s) within the section. Tissues of differing densities are represented electronically by different spatial frequencies. Adjacent tissues that greatly differ in density are represented by high spatial frequencies.

Areas of a CT image that contain abrupt changes in tissues density are electronically represented by which of the following? a. positive CT numbers b. high spatial frequencies c. negative CT numbers d. low spatial frequencies

D. low spatial frequencies The contrast of a CT image is controlled by the spatial frequencies of the tissue(s) within the section. Tissues of differing densities are represented electronically by different spatial frequencies. Adjacent tissues with similar densities or areas of tissue with minimal differences in density are represented by low spatial frequencies.

Areas of a CT image that contain minimal changes in tissue density are electronically represented by: a. positive CT numbers b. high spatial frequencies c. negative CT numbers d. low spatial frequencies

D. hepatocellular carcinoma Hypervascular lesions that appear hyperdense in comparison with surrounding normal hepatic parenchyma during the arterial phases of contrast enhancement.

Arterial phase CT imaging of the liver is used to optimally demonstrate which of the following? a. fatty infiltration b. hepatic cysts c. portal vein thrombosis d. hepatocellular carcinoma (HCC)

D. window level

As a general rule, this is set at a point that has roughly the same value as the average attenuation number of the tissue of interest. What is it? A) the x-axis in a histogram B) standard deviation C) sampling rate D) window level

Short wavelengths

As kVp increases, there is an increased production of:

Contrast increases

As kVp is decreased:

Scale of contrast lengthens

As kVp is increased:

C. 1 and 3 -- precisely limit the acquisition to the indicated anatomical area and reduce and reduce the technical parameters (mAs, kVp) based upon the body habitus "Child-sizing" protocols, minimizing the anatomic scan area, and eliminating unnecessary enhancements phase acquisitions are all valuable components of a comprehensive dose reduction approach for pediatric patients. Rather than refuse to perform an ordered CT examination, technologists are encouraged to discuss the justification for a pediatric CT procedure with a supervising physician or other independent license practitioner and to make every effort to minimize dose when the clinical indication for the procedure is confirmed.

As part of a comprehensive approach to minimize CT radiation exposure to the pediatric patient, the technologist should: 1. precisely limit the acquisition to the indicated anatomical area 2. refuse to perform unnecessary CT examinations 3. reduce the technical parameters (mAs, kVp) based upon the body habitus a. 2 only b. 1 and 2 only c. 1 and 3 only d. 1, 2 and 3

A. diastole Diastole refers to the relaxation of heart muscle. *Diastole is the period of the cardiac cycle that occurs between successive contractions of the heart muscle*, which are known as systole.

As related to the cardiac cycle, the term _______________ refers to the relaxation of heart muscle. a. diastole b. equilibrium c. systole d. a-fib

A. increases The transmitted intensity of CT x-ray beam and the attenuation of the tissue imaged are inversely related. As the tissue begins to attenuate less radiation, the transmitted intensity of the beam increases. Areas of less dense tissue allow more radiation to pass onto the detectors, and vice versa.

As the attenuation of a volume of tissue decreases, the transmitted intensity of a CT x-ray beam: A) increases B) remains unchanged C) decreases D) increases to a peak value and then rapidly decreases

B. decrease the pitch A change in window settings has no effect on radiation dose, as it is done after the data acquisition. Using a wider slice for an examination that contains multiple slices will reduce the radiation dose, as will decreasing the mAs. Decreasing the pitch will increase the radiation dose to the patient. For example, it is estimated that decreasing the pitch from 1.5 to 1.0 will result in a dose increase of approximately 33%.

Assuming all other factors are held constant, which of the following will increase the radiation dose to the patient for a CT examination of the abdomen? A) widening the window width; increasing the window level B) decreasing the pitch C) using a wider slice D) decreasing the mAs

C. 1, 2, 4 and 5 only -- eliminate a clinically unnecessary contrast phase acquisition, increase the tolerated noise level of the scan, increase the pitch from 1.0 to 1.5 and decrease the scan length along the z-axis An increase in kVp, with no other compensating adjustment in technical parameters, will result in greater tube output and a higher subsequent patient dose. Decreases in kVp will significantly lower patient dose (according to radiography's 15% rule) if the mAs is not adjusted to compensate.

Assuming no other technical changes are made, which of the following adjustments would result in DECREASED radiation dose to the patient for a given acquisition? 1. Eliminate a clinically unnecessary contrast phase acquisition 2. increase the tolerated noise level of the scan 3. increase tube potential (kVp) from 100 to 140 4. increase the pitch from 1.0 to 1.5 5. decrease the scan length along the z-axis a. 1, 3 and 5 only b. 2, 3 and 4 only c. 1, 2, 4 and 5 only d. All (1-5)

A. bone According to the modulation transfer function (MTF) diagram, the bone algorithm has a resolution of approximately 12 lp/cm. The standard algorithm and smooth algorithm are approximately 10 and 7 lp/cm, respectively.

At a modulation transfer function (MTF) of 20%, which reconstruction algorithm will demonstrate the greatest number of line pairs per centimeter (lp/cm)? A) bone B) standard C) smooth D) impossible to determine from the information provided

C. 50% It is widely recommended that volumetrically acquired CT sections be reconstructed with the use of at least a 50% overlap to improve the quality of 3D and multiplanar reformation (MPR) images.

At what percentage of overlap should CT sections be reconstructed to improve the quality of multiplanar reformation (MPR)? a. 10% b. 25% c. 50% d. 100%

The number of protons plus the number of neutrons

Atomic mass refers to:

Describes changes in the x-ray beam as it travels through the patient.

Attenuation

C. effectively accommodate for individual differences in circulation time by adjusting the scan delay Both the test bolus method and the bolus triggering method allow the scan delay to be individualized to the patient. Unfortunately, they do not improve patient safety. In fact, the major disadvantage of bolus triggering is that the technologist is no longer able to monitor the injection site during the initial seconds of the contrast injection. Because a timing bolus requires an additional series of scans to calculate the delay, it increases the total examination time.

Automated injection triggering methods are used to: A) simplify the process of contrast injection, so that assistance can perform the task and patient throughput can be increased B) improve patient safety by reducing contrast extravasations C) effectively accommodate for individual differences in circulation time by adjusting the scan delay D) effectively accommodate for individual differences in circulation time by adjusting the contrast flow rate

C. reducing patient radiation dose Automatic tube current modulation (ATCM) is a form of automatic exposure control for a CT system. ATCM manages patient radiation dose on the basis of the size, density, and overall attenuation of the part being examined.

Automatic tube current modulation (ATCM) is employed by a multidetector (MDCT) system for the purpose of: A) z-axis interpolation B) improving 3D and multiplanar reformation (MPR) image quality C) reducing patient radiation dose D) freezing heart motion during cardiac CT

A. patient radiation dose Automatic tube current modulation (ATCM) programs adjust the mA throughout an acquisition to reduce patient radiation dose to a minimum. ATCM automatically alters the applied mA on the basis of a predetermined noise index that is acceptable for appropriate image quality.

Automatic tube current modulation (ATCM) software is used in current multidetector CT (MDCT) systems to control: a. patient radiation dose b. cardiac motion c. timing of the contrast agent bolus d. tissue perfusion assessment

What is the detector aperture size of a particular MDCT scanner with the beam collimation of 60mm and 128 detector rows? A) 1.2mm B) 0.46mm C) 0.3mm D) 1.5mm

B) 0.46mm ***detector aperture size= beam collimation/detector rows detector aperture size= 60mm/128 detector aperture size= 0.46mm***

Optimal IV access for CT procedures is achieved with that size angiocatheter when utilizing a power injector? A) 1 inch, 24 gauge B) 1 inch, 20 gauge C) .75 inch, 24 gauge D) .75 inch, 22 gauge

B) 1 inch, 20 gauge ***CT injection procedures require the use of a power injector to administer contrast media with consistent flow rates and duration times. Due to the mechanism of the power injectors, a 20-gauge, 1 inch angiocatheter is optimal, since they provide stability when the power injector releases pressure.***

What is the voxel volume of a CT digital image having a 320mm field of view (FOV) , a 512^2 matrix, and a slice thickness of 3mm? A) 0.625mm^3 B) 1.17mm^3 C) 0.39mm^2 D) 1.17mm^2

B) 1.17mm^3 ***The equation is voxel volume = pixel area^2 X slice thickness.***

What is the general iodine concentration of water-soluble contrast media for gastrointestinal opacification during routine abdominopelvic CT examinations? A) 5 to 10mg I/mL B) 13 to 15 mg I/mL C) 20 to 30 mg I/mL D) 50 mg I/mL

B) 13 to 15 mg I/mL ***Various iodine concentrations of water-soluble contrast media ranging from 4 to 48 mg I/mL, but in general, a solution containing 13-15 mg I/mL is recommended for oral and rectal administration in adults undergoing CT examinations.***

What is the tale speed if the pitch is 1.5, the gantry rotation is 1 second, and the detector configuration is: 16 x 1.2 A) 19.2mm/s B) 28.8mm/s C) 13mm/s D) 16mm/s

B) 28.8mm/s ***Pitch= table speed (mm/sec) X gantry rotation (sec)/ total beam collimation 1.5= X table speed (mm/sec) X1 sec/ 16X1.2 1.5= X/19.2 table speed= 28.8mm/sec***

What is the usual contrast delay when scanning the pancreas for hypervascular lesions or carcinomas using a flow rate of 4 to 5 cc/s ? A) 15 to 20 seconds B) 30 to 40 seconds C) 70 to 80 seconds D) 120 to 180 seconds

B) 30 to 40 seconds ***The blood supply to the pancreas is completely arterial; therefore the attenuation of the parenchyma rises faster than the liver, but last for a shorter time because of the absence of the portal phase. Peak enhancement occurs 5 seconds after the aortic plateau.***

What is the dose limit for radiation workers in the USA? A) 20 mSv/year B) 50 mSv/year C) 75 mSv/year D) 100 mSv/year

B) 50 mSv/year

What ligament is longest and strongest tendon in the lower leg? A) Anterior cruciate B) Achilles C) Posterior cruciate D) Medial meniscus

B) Achilles

What type of detector array system contains detectors in which their width gradually increases in thickness as it moves away from the center of axis rotation? A) Hybrid array B) Adaptive array C) Matrix array D) Uniform array

B) Adaptive array ***Adaptive, variable, or nonuniform array detectors consist of detector rows that grow in width from the center of the section to the periphery.***

Which of the following can occur if NOT enough samples are collected during data acquisition? A) Beam hardening B) Aliasing artifact C) Image reconstruction is not possible D) Blurry Image

B) Aliasing artifact ***Can occur if the appropriate number of samples is not achieved during data acquisition. It appears in the final image as fine streaks and web-like patterns.***

What emotional state can lead to severe adverse reaction to contrast media? A) Confusion B) Anxiety C) Exaltation D) Consciousness

B) Anxiety

Patient history taking in children usually does not typically include which of the following? A) History of contrast reaction B) Blood urea nitrogen (BUN) level C) Patient anxiety level D) History of asthma

B) Blood urea nitrogen (BUN) level ***Is a poor indicator in renal function in children due to it depends on too many factors along with diet. The best way for estimating GFR from serum creatinine in children is the Bedside Schwartz equation.***

Which of the following primary cancer usually does not metastasize to the lungs? A) Kidney B) Bone C) Breast D) Colon

B) Bone ***The lungs are the most common site of metastasize disease from primary cancers outside the lungs. Primary cancers such as colon, breast, kidney, pancreas, and uterus are most likely to metastasize to the lungs. The disease spreads through the blood circulation and lymphatic system.***

The purpose of contrast media is their ability to differentiate between radiographic densities and enable differences in anatomic tissues to be visualized. Which of the following contrast media agents is considered a negative agent? A) Omnipaque B) Carbon dioxide C) Diatrizoate meglumine D) Iohexol

B) Carbon dioxide ***The body absorbs photons according to the various atomic numbers and the amount of matter per volume of tissue, since carbon dioxide has an atomic number of 6, it's considered a negative agent.***

What pathological condition is characterized by submucosal edema with ulcerations involving a thickened segment of distal ileum? A) Small bowel obstruction B) Crohn's disease C) Diverticulosis D) Pelvic inflammatory disease

B) Crohn's Disease ***Is a characterized by submucosal edema with ulcerations involving a thickened segment of distal ileum. The terminal ileum is affected in 80% of patients, while the colon is affected in 50%.***

The ratio of the largest signal to the smallest signal measured by a CT detector is considered: A) Response time B) Dynamic range C) Efficiency D) Stability

B) Dynamic range ***Is the ratio of the largest signal to be measured to the precision of the smallest signal to be discriminates. In other words, it can be described as the range of voltage or input signals that results in a digital output. The dynamic range for many modern CT scanners today is a million to one.***

The wall of the uterus is composed of glandular and muscular linings, which lining of the uterus is highly vascular and responsible for the main contraction force during childbirth? A) Endometrium B) Epimetrium C) Myometrium D) Perimetrium

B) Epimetrium

If one wants to improve spatial resolution, which of the following combinations of FOV and matrix size would be most effective? A) FOV 10 cm, matrix 256^2 B) FOV 10 cm, matrix 512^2 C) FOV 15 cm, matrix 512^2 D) FOV 25 cm, matrix 256^2

B) FOV 10 cm, matrix 512^2 ***The smaller the pixels the better the spatial resolution. Small pixels are produced with a combination of a small FOV and a large matrix with the assumption that the entire area of interest is covered.***

Of the following, which organ is located in the right upper quadrant? A) Ileum B) Gall bladder C) Descending colon D) Fungus of the stomach

B) Gall Bladder

What is the region called that is on the medial side near the center of both the right and left lungs, and is where the bronchi, veins, and arteries enter and exit the lungs next to the heart? A) Pulmonary truck B) Hilum C) Left atrium D) Carina

B) Hilum ***The hilum is a depression or pit where structures are attached. As in the chest, the hilum is part of the lungs where structures such as blood vessels and nerves enter.***

Which of the following forms of consent is used when a patient is unconscious in the emergency room? A) Simple consent B) Implied consent C) Formal consent D) Expressed

B) Implied Consent **It implies that we save your life if you come to ER and need medical help.***

When speaking of radiation dose, a low pitch ratio greater than 1 will produce what effect on dose? A) Decrease dose B) Increase doe C) Dose is not dependent on pitch D) A pitch ratio greater than 1 is never used in CT

B) Increase dose ***Pitch can be described as the ratio of the distance of the table travels per one revolution of the x-ray tube to the total collimated beam width. If the mAs is kept constant as the table moves, the radiation dose is proportionally decreased as pitch increased.***

Immobilization devices prevent a patient from undesirable motion during the CT examination. Which of the following is categorized as a restraint and requires physician's orders? A) Tape B) Leather wrist guards C) Velcro straps D) Sponges

B) Leather Wrist Guards

Which of the following chest muscle lies on the anterior surface of the third to fifth ribs? A) Pectoralis major muscle B) Pectoralis minor muscle C) Subclavius muscle D) Serratus anterior muscle

B) Pectoralis minor muscle ***This muscle lies directly behind the pectoralis major on the anterior surface of the third through fifth ribs. It acts to elevate and protract the scapula.***

What mater is located in the innermost layer of the meninges? A) Dura mater B) Pia mater C) Arachnoid mater D) Falx cerebri

B) Pia mater ***Pia mater is the innermost layer of the meninges surrounding the brain and the spinal cord.***

Contrast resolution can be improved by which of the following? A) Imaging faster B) Reducing image noise C) Using a bone reconstruction filter D) Using lower kVp

B) Reducing image noise ***Contrast and noise have a direct relationship. Noise can be defined as the unwanted fluctuation of pixel values in an image of a homogeneous material. The LCD definition concludes that the visibility of an object depends not only on the size of the object, but on its contrast to the background; therefore this visibility is highly dependent by the presence of noise.***

In recent years, CT dose has been a hot topic in the radiology community; therefor, many institutions have developed dose reduction team members to assess the radiation dose of each patient protocol. Which of the following is NOT a role of the CT dose reduction team? A) Identify examination for dose reduction B) Review service logs C) Perform image quality review for each protocol D) Communicate changes with radiologists

B) Review service logs ***The major roles of the team are to identify examinations for dose reduction, perform image quality review of each examination, and communicate the changes made with radiologist. Each member of the team has their own responsibility, but the primary goal with dose reduction endeavors is to uphold the diagnostic performance of CT, which in turn is influenced by the image quality.***

Which of the following statements defines quality control? A) Is any systematic process of checking the CT scanner's performance B) Reviews the results of standard measurements and takes steps to correct any discrepancies C) Ensures you are doing the right things, the right way D) Concept that covers all policies and systematic activities implemented within a quality system

B) Reviews the results of standard measurements and takes steps to correct any discrepancies ***The term "quality assurance" and "quality control" are often used interchangeably and should not be confused even though, both refer to ways of ensuring the quality of a CT scanner. Quality control on the other hand reviews the results of the quality assurance testing and makes corrections to improve the scanner's performance.***

Which of the following salivary ducts are responsible for the flow of saliva withing the mouth? A) Stensen's duct B) Rivinus's duct C) Wharton's duct D) Treitz's duct

B) Rivinus's duct ***Is located within the sublingual gland is responsible for the flow of saliva within the mouth.***

The pituitary gland lies within what structure of the cranium? A) Pituitary stalk B) Sella turcica C) Cavernous sinus D) Sphenoid sinus

B) Sella Turcica

What structure separates the lateral ventricles? A) Choroid plexus B) Septum pellucidum C) Falx cerebri D) Corpus callosum

B) Septum pellucidum ***Is a thin partition in cerebral hemisphere located between the columns of the fornix and the corpus callosum separating the anterior horns of the lateral ventricles.***

Which of the following radiation dose quantities is determined by multiplying CTDIvolume by a conversion factor based on the patient's effective diameter? A) Absorbed dose B) Size-specific dose estimation C) Effective dose D) CTDIvolume

B) Size-specific dose estimation ***SSDE is determined by multiplying CT dose index (CTDI) volume by a conversion factor based on the patient's effective diameter. Is defined as the square root of the project of the anterior-posterior and the lateral patient diameter while not currently displayed with the other CT parameters such as CTDIvolume or the dose length product (DLP) on the DICOM header of an individual patient's studies, SSDE could be calculated automatically by the scanner using the patient localizer images.***

What is the term used for the amount of friction generated by the molecules of a contrast media? A) Osmolality B) Viscosity C) Orinase D) Nasoenteric

B) Viscosity ***Is the amount of friction generated by the concentration and the size of the contrast molecules. The higher the viscosity, the thicker the agent, and the more difficult to inject.***

What is considered the basic principle of estimating an unknown value of a function from two or more known values of the same function? A) Filtered back projection B) z- interpolation C) Algorithm D) Convolution

B) z-interpolation ***In mathematics, engineering, and science the basic idea of interpolation is to estimate an unknown value of a function from two or more known values of the same function. The two types of interpolation are known as 360-degree interpolation and 180-degree interpolation being the technique of choice especially with MDCT.***

C. hyperdense Especially when compared to the surrounding soft tissues/organs around the thyroid, the thyroid is hyperdense because it naturally contains iodine. This thyroid's density is even more increased when iodine is injected.

Because of its inherent iodine concentration, the thyroid gland appears __________ on CT images. a. hypodense b. isodense c. hyperdense d. mottled

B. analog-to-digital converter Responsible for transforming the analog signal from the detectors into a digital form that may be used by the computer. Analog information is based on a scale, whereas digital information is in numerical form.

Before a CT image can be reconstructed by a computer, the transmission signal produced by the detectors must be converted into numerical information by a(n): a. kernal b. analog-to-digital converter c. array processor d. digital-to-analog converter

D. 1, 2 and 3 -- renal function, allergic history and cardiac history The technologist and/or radiologist should always ask the patient about allergic history as well as current renal function status and history of cardiac illness (high blood pressure, congestive heart failure, and so on) before administration of any contrast material.

Before the intravenous injection of iodinated contrast material, patients should be questioned regarding their: 1. renal function 2. allergic history 3. cardiac history a. 1 only b. 2 only c. 3 only d. 1, 2 and 3

The available gray scale of an imaging system

Bit depth describes:

C. patient radiation dose Bow-tie filters, which are thicker at the ends than in the middle, help shape the beam to reduce patient radiation exposure. Because most body parts are circular or cylindrical, less radiation is necessary at the periphery than in the center. Bow-tie filters reduce beam intensity toward the outer margins, resulting in a reduction in patient radiation dose.

Bow-tie filters are employed in a CT x-ray system to reduce: a. image noise b. low spatial frequency signal c. patient radiation dose d. high spatial frequency signal

C. ileus Used to describe an area of intestine that has lost normal contractile motion, resulting in obstruction.

Bowel obstruction due to loss of normal contractile motion in an area of intestine is termed: a. dyspepsia b. intussusception c. ileus d. adhesion

Fewer than 60 beats per minute

Bradycardia indicates a pulse of:

A trauma patient received a chest/abdomen/pelvis study with a total beam collimation of 153.6mm and the detector aperture size equaled 1.2mm, what detector row scanned was this procedure on? A) 32 B) 64 C) 128 D) 320

C) 128 ***The equation is number of detector rows = total beam collimation / detector aperture size***

A particular CT examination requires the patient to receive 75cc of nonionic contrast delivered over a 30 second duration time. What would the flow rate be for this study? A) 1.5cc/s B) 2.0cc/s C) 2.5cc/s D) 3.0cc/s

C) 2.5cc/s ***Flow rate = volume/ duration time***

Extravasated iodinated contrast media are toxic to the surrounding tissues, particularly to the skin, producing an acute local inflammatory response that sometimes peaks within how many hours post injection? A) 1 to 5 hours B) 10 to 12 hours C) 24 to 48 hours D) 60 to 72 hours

C) 24 to 48 hours ***Produces an acute local inflammatory response time within 24 to 48 hours following the accident.***

What is the reconstructed DFOV, when the pixel area is 0.49mm^2 with a 512^2 matrix A) 250mm B) 731mm C) 358mm D) 1,044mm

C) 358mm ***DFOV= pixel size X matrix size***

Which of the following flow rates is utilized for a CTA examination of the chest for pulmonary embolism? A) 1 to 2 cc/s B) 2 to 3 cc/s C) 4 to 5 cc/s D) 7 to 10 cc/s

C) 4 to 5 cc/s ***Is the protocol for PE studies. This fast injection rate with a short delay time will enable the contrast media to enhanced the pulmonary arteries in order to depict a filling defect within the artery.***

What is the matrix size if the isotropic voxel volume equals 0.125mm^3 with a 256 display field of view? A) 256 B) 320 C) 512 D) 1024

C) 512 ***The first problem is to solve is determining the pixel size. Since an isotropic voxel contains three equal sides, the cube root of 0.125mm^3 is equal to 0.5mm pixel size. The following equation is utilized to find the matrix.***

The endocrine system is a collection of glands that produce hormones that regulate your body's growth, metabolism, and sexual development and function. Which of the following organs accelerates metabolism and energy and are responsible for the body's "fight or flight" response? A) Thymus gland B) Thyroid gland C) Adrenal gland C) Pituitary gland

C) Adrenal Gland ***Adrenal glands produce the hormones epinephrine and norepinephrine. These hormones secrete into the body's system to influence the body's metabolism, blood chemicals, and body characteristics, as well as influence the part of the nervous system that involved in the response and defense against stress; hence, "fight or flight" response.***

Which of the following pathological conditions can lead to mesothelioma? A) Histoplasmosis B) Emphysema C) Asbestosis D) Cystic fibrosis

C) Asbestosis ***Is the inhalation of asbestos fiber causing scarring of the lungs that can lead to lung CA (Mesothelioma). On CT images it may show ground-glass opacities and honeycomb patterns.**

What new technique employed in multislice CT leads to reduction of radiation dose? A) Slip rings B) Size of the detector aperture C) Automatic tube current modulation D) Subsecond scanning

C) Automatic tube current modulation ***Tube current modulation (mA modulation) is an essential tool to ensure proper patient exposure with CT examinations. It allows the tube current to be actively modulated during the scan to more efficiently apply radiation to the patient. This potentially saves dose because instead of using a fixed tube current optimized for the thickest part of the patient, the scanner will produce fewer x-ray photons in regions of the lower attenuation and modulate higher values of tube current in regions of higher attenuation.***

Which of the following spine fractures is due to a severe compression of the vertebral body? A) Jefferson's fracture B) Hangman's fracture C) Burst fracture D) Vacuum phenomenon

C) Burst fracture ***Burst fracture occurs when there is hyperflexion and axial loading on the spine. It most commonly occurs in the thoracic and lumbar spine. It involves the posterior body and the posterior longitudinal ligament with the compressed disc herniating into the vertebral body***

A dedicated CT examination of the kidneys was performed on a patient complaining of frequent urination, upon the radiologist review, a small lesion with the u of .189 was discovered. What did the radiologist attribute this measurement to be in HU? A) Water B) Air C) Fat C) Calcium

C) Calcium

Name a characteristic of image data: A) Can be retrieved for later use B) Is subjected to preprocessing C) Can be sent to the CD burner D) Is a nonvisible digital image

C) Can be sent to the CD burner ***Image data are the result of raw data being convoluted and processed by the computer. Once this process is completed a visible digital image appears on the scanner console. The only operation that can be performed on this data is reformation in the form of multiplanar of 3D manipulations. Image data can be copied to a CD and reviewed by means of a DICOM viewer or it can be sent to a laser printer for hard copy viewing.***

Which of the following CT examinations is temporal resolution of great importance? A) HRCT chest B) Inner ear C) Cardiac angiography D) Bone mineral density

C) Cardiac angiography ***Temporal resolution has become a hot topic. Even though modern multidetector computed tomography (MDCT) scanners have the capability of producing scan speeds as low as 33 milliseconds, it's not sufficient enough to completely freeze the motion of the beating heart.***

What is the purpose of the analog-to-digital converter? A) Converts digital data to analog form B) Convert transmitted data into attenuation and thickness data C) Encode the analog measurements into binary data D) Send data to the optical receiver array

C) Encode the analog measurements into binary data ***The purpose of the analog-to-digital converter (ADC) is to divide the electrical signal into multiple parts, the more divisional parts, and the greater the accuracy. The parts are measured into bits, 1-bit divides the signal into two digital values. The ADC's dynamic range must be large to preserve the large range of the x-ray image. Converts with 16-bit resolution or greater are common in CT. These values determine the gray-scale resolution of the image.***

The same CTDIvolume was used on two different patients, one with a small body habitus and the other patient was extremely large. What would the absorbed dose be to the smaller patient as compared with the larger patient? A) Lower B) Stay the same C) Higher D) Half of the larger patient's dose

C) Higher ***The absorbed dose to the smaller patient would be higher because the larger patient will have a more diluted deposition of absorbed dose, since the same dose is distributed over a larger mass than a smaller patient exposed to the same radiation.***

Which of the following is NOT a characteristic of CT collimation? A) Ensure a constant beam width at the detectors B) Reduce patient dose C) Increase scatter radiation D) Improve image quality

C) Increased scatter radiation ***The purpose of collimation is to protect the patient by restricting the beam to the anatomy of interest. They are arranged to ensure a constant beam width at the detectors.***

The American College of Radiology (ACR) publishes guidelines on ways of reducing radiation levels for various types of scans. Which of the following is NOT a way of minimizing radiation dose to patients? A) Safety checks B) Dose customization C) Increasing tube potential output D) Duplicate examination prevention

C) Increasing tube potential output ***Safety checks provide procedures for checking and double-checking patient information and the type of exam to be performed to prevent patients receiving an unnecessary examination.***

Which of the following reconstruction algorithms is used as a means of reducing patient dose? A) Filtered back projection B) Algebraic reconstruction C) Iterative reconstruction D) Convolution reconstruction algorithms

C) Iterative reconstruction ***The sophisticated iterative reconstruction algorithms starts with in initial guess of the object and iteratively improves on the initial estimate of the attenuation value by comparing the estimate noise projection with the acquired projection data and making an incremental model-based change to the previous guess. Thus reducing patient dose.***

Which of the following structure reside in the retroperitoneal cavity? A) Spleen B) Liver C) Kidney D) Ileum

C) Kidney ***Consists of structures posterior to the peritoneum.***

Which of the following characteristics contributes to heat dissipation in a CT x-ray tube? A) High-frequency generator B) Increased beam filtration C) Large diameter disk D) Precise beam collimation

C) Large diameter disk ***The anode is the positive terminal and part of the tube where the target material is located. The spiral/helical scanners employ a 200 mm diameter rotating anode disk allowing for the use of higher tube currents in the range of 120 to 140kVp. The heat storage capacity with these disks is increased with an improvement in heat dissipation.***

What classification of computer system does a CT scanner utilized? A) Supercomputer B) Mainframe C) Minicomputer D) Microcomputer

C) Minicomputer ***The midrange or minicomputer is the computer system used in CT. The midrange computer deals efficiently with the convoluted mathematical input and produces output data at subsecond time intervals. Its major characteristics are its large storage capacity and fast and efficient processing of various kinds of data.***

Bolus or power injectors of IV contrast material is superior to drip infusion for enhancing normal and abnormal structures during body CT examinations. Which of the following is NOT considered the proper technique to avoid the potentially serious complications of contrast media extravasation and/or air embolism? A) The patient's full cooperation should be obtained when even possible B) Use of a clean syringe C) Once the syringe has been purged of air, keep the syringe in the upright position D) Catheter tip should be checked for venous back flow.

C) Once the syringe has been purged of air, keep the syringe in the upright position ***The main steps in avoiding a potentially serious complication associated with the use of a power injector is to obtain complete cooperation from the patient if possible. Communication is the key. Always uses a clean syringe, and after being purged of air keep the tubing downward.***

What benign pathological condition that occurs in women is seen on CT images as a well-defined mass with fluid density usually in the range of +/- 15 HU? A) Pelvic inflammatory disease B) Ovarian cancer C) Ovarian cyst D) Uterine cancer

C) Ovarian Cyst ***On CT imaging, ovarian cysts are homogeneous near water densities and have thin well defined walls.***

A dedicated CT examination of the liver was performed on a male patient with elevated LFTs. One image revealed a shading appearance on the right lobe of the liver and an erroneous HU measurement. The radiologist requested the technologist to perform thinner slices; what could be the reason for this request? A) Beam hardening artifact B) Cone beam artifact C) Partial volume averaging artifact D) Motion artifact

C) Partial volume averaging artifact ***Is when tissues of widely different absorption are encompassed on the same CT voxel producing a beam attenuation proportional to the average value of these tissues. Using thinner slices will differentiate the different tissue types eliminating the shading artifact that appeared on the liver as In the above situation.***

What is the name of the ligament that extends between the patella and the tibial tuberosity? A) Medial collateral ligament B) Lateral collateral ligament C) Patellar ligament D) Ligamentum flavum

C) Patellar ligament ***Is the largest ligament of the knee.***

What are the two muscles found on the anterior surface of the chest having their primary function to mover the upper limbs? A) Trapezius muscle B) Rhombus muscle C) Pectoralis muscle D) Scalene muscle

C) Pectoralis muscle ***Are two muscles found on the anterior surface of the chest with their primary function being movement of the upper limbs. It's also the most anterior muscle and aids in flexion, abduction, and rotation of the arms.***

What type of radiation interaction with matter occurs when an incident x-ray photon is totally absorbed during the ionization of an inner shell electron? A) Compton scatter B) Bremsstrahlung interaction C) Photoelectric interaction D) Stochastic effect

C) Photoelectric Interaction ***Occurs when an incident x-ray photon is totally absorbed during the ionization of an inner shell electron (usually the K or L shell). The incident x-ray photon's energy must be greater than the binding energy of the inner shell electron.***

Which of the following places a patient is at risk for developing a chemotoxic adverse reaction to iodinated contrast media? A) Pregnancy B) Cirrhosis C) Pre-existing renal insufficiency D) Pre-exiting renal calculi

C) Pre-existing renal insufficiency ***Chemotoxic reactions result from the physicochemical properties of the contrast media that can result in contrast induced nephropathy (CIN).

Documentation is imperative whenever IV contrast is administered. Which of the following is NOT a method used for medication reconciliation of IV contrast? A) Time the drug was administered B) Name and strength of the medication C) Scan delay time D) Patient's tolerance

C) Scan Delay Time ***Medication reconciliation is a formal process in which healthcare providers ensure accurate account of the contrast media given to each individual patient. This documentation should include the contrast media's name and strength, volume, route, data, and time of administration. along with the patient's tolerance of the medication.***

What is the largest and first carpal bone in the proximal row of the wrist? A) Lunate B) Capitate C) Scaphoid D) Hamate

C) Scaphoid

A trauma patient enters the ER after sustaining a 20 foot fall from a building. Which of the following convolution kernels would be applied for both the prospective and retrospection reconstruction of the patient's trauma survey examination? A) Ultrasmooth and smooth B) Ultrasmooth and standard C) Standard and bone D) Bone and ultrasmooth

C) Standard and bone ***When a patient sustains a fall from a great distance the major concerns are for internal bleeding and bone fractures. To enhance the abdominal cavity, a standard convolution is applied in order to demonstrate soft tissue structures and detailed peripheral borders. The bone or high-pass kernel is used to enhance bony details where fine hair-line fractures could be seen.***

What meningeal space is filled with cerebrospinal fluid? A) Epidural B) Subdural C) Subarachnoid D) Subpial

C) Subarachnoid Space ***Is the interval between the arachnoid membrane and the pia mater containing cerebrospinal fluid and vessels.***

Which artery may compress the left renal vein against the aorta causing venous stasis or Nutcracker syndrome? A) Iliac artery B) Inferior mesenteric artery C) Superior mesenteric artery D) Right renal artery

C) Superior mesenteric artery ***Superior mesenteric artery extends from the aorta and branches into several arteries that supply blood to the majority of the small intestine and ascending and transverse colon. Due to location of the left renal vein passing posterior to the superior mesenteric artery, the artery may compress against the left renal vein compromising the drainage of the vein into the IVC.***

Which CT scanner is based on a fan beam geometry and complete rotation of the x-ray tube and detectors? A) First generation B) Second generation C) Third generation D) Fourth generation

C) Third generation ***Third generation CT scanners employed the rotate-rotate scanning geometry principle with the fan beam geometry. This principle allowed the x-ray tube and detectors to make a complete 360-degree rotation around the patient in order to collect a large set of data samples for the reconstruction process.***

What area within the neck are common areas for foreign bodies to become lodge? A) Piriformis sinus B) Aryepiglottic folds C) Valleculae D) Rima glottis

C) Valleculae ***Are spaces on either side of the glossoepiglottic fold. On cross-sectional images it can be seen posterior to the tongue and the epiglottis. It's a common site for foreign bodies to lodge.***

What arteries join together to form the basilar artery? A) Internal carotid arteries B) Common carotid arteries C) Vertebral arteries D) Posterior cerebral arteries

C) Vertebral arteries ***The arterial system of the brain commences with the basilar artery. Its found at the midline on the anterior surface of the pons. It originates from the junction of the right and left vertebral arteries.***

24%

CT accounts for approximately what percent dose to the population (NCRP Report #160)?

B. Duodenum and pancreatic head

CT examinations of the abdomen are often performed in the position on the figure to demonstrate the relationship between the: A) ureters and renal collecting systems B) duodenum and pancreatic head C) large and small colon D) liver and gallbladder

A. a full inspiration Patient inspiration provides optimal chest expansion and allows for better demonstration of anatomic structures.

CT images of the chest should be acquired with the patient: a. at full inspiration b. breathing quietly c. at full expiration d. breathing normally

A. Hounsfield units The CT number is a relative value based on the attenuation that occurs within a voxel of tissue. The Hounsfield unit is used for this value.

CT numbers are usually provided in the form of: a. Hounsfield units b. EMI numbers c. Cormacks d. u

D) Small storage capability ***A mid-range computer was known in the past as a mini-computer. This computer is smaller than a mainframe computer, but larger than a microcomputer (personal computer)***

CT scanners utilized a mulititude of complex computation; therefore, they employ a mid range computer. Which of the following is NOT a characteristic of a mid-range computer? A) Networking capability B) Dealing efficiently with the convoluted mathematical input C) Fast and efficient processing D) Small storage capability

D) Small storage capability ***A mid-range computer was known in the past as a mini-computer. This computer is smaller than a mainframe computer, but larger than a microcomputer (personal computer)***

CT scanners utilized a mulititude of complex computation; therefore, they employ a mid range computer. Which of the following is NOT a characteristic of a mid-range computer? A) Networking capability B) Dealing efficiently with the convoluted mathematical input C) Fast and efficient processing D) Small storage capability

A. 1 only As the beam traverses the patient, beam hardening alters the penetrability of the beam and affects the absorption across the field of view. The CTDIw is calculated by summing two-thirds of the exposure recorded at the periphery of the field with one-third of the centrally recorded radiation dose. This weighting yields a more accurate dose approximation.

CTDIw is a weighted index that approximates radiation dose on the basis of the variations that occur across the the field of view because of: 1. beam hardening 2. detector configuration 3. 3 pitch a. 1 only b. 1 and 2 only c. 2 and 3 only d. 1, 2 and 3

Long-term somatic effects

Cataractogenesis, life-span shortening, embryological effects, and carcinogenesis are examples of:

A. z axis reconstruction or changing the reconstruction interval Changing the slice incrementation is only possible with helical CT data.

Changing the slice incrementation is also referred to as: A) z axis reconstruction or changing the reconstruction interval B) dynamic r cine scanning C) changing the zoom, target, or display field-of-view D) electron beam or fifth-generation CT

C. 1 and 3 only -- at the x-ray tube regulating slice thickness, and before the detectors, limiting the amount of scatter radiation absorbed Collimation of the x-ray beam in CT is accomplished by both prepatient and predetector collimators, or detectors. The prepatient detectors restrict the field size, directly influencing the reconstructed section thickness. The predetector or postpatient detectors absorb scatter radiation before it contributes to the signal produced by the detector array.

Collimation of the CT x-ray beam occurs: 1. at the x-ray tube, regulating slice thickness 2. just beyond the patient, focusing off-axis transmitted radiation 3. before the detectors, limiting the amount of scatter radiation absorbed a. 1 only b. 1 and 2 only c. 1 and 3 only d. 1, 2 and 3

A. Ultravist 370 has a higher viscosity Both the viscosities and the osmolalities of contrast agents classified as nonionic/low-osmolality will rise slightly with the concentration. Because higher-concentration agents are more viscous they may not flow through small-gauge IV catheters as easily as lower-concentration agents. *One hundred milliliters of a 300 concentration agent will contain 30 grams of iodine, whereas 100 mL of a 370 concentration agent will contain 37 grams of iodine.*

Compare 100mL of Ultravist (iopromide) 300 with 100mL of Ultravist (iopromide) 370. A) Ultravist 370 has a higher viscosity B) Ultravist 370 contains fewer grams of iodine C) Ultravist 370 is a better choice if the IV catheter is a 22 - 24 gauge D) Ultravist has a lower osmolality

A. low-contrast resolution Computed tomography has better low-contrast resolution over conventional radiography. The CT system is extremely sensitive to small changes in tissue density and removes the problem of superimposition, both leading to greater contrast resolution.

Compared with conventional radiography, computed tomography produces diagnostic images with better: a. low-contrast resolution b. spatial resolution c. minute detail d. patient radiation dose reduction

B. 1 and 2 only -- use of a cone beam of radiation and acquisition at thinner section widths Dose can be considerably higher with MDCT due to a decrease in the focal spot-detector distance, the use of a cone beam instead of a more z-axis-collimated fan beam, an increase in number of imaging phases enabled by shorter scan times, and the acquisition of thinner sections for improvement of 3D and multiplanar reformation (MPR) images.

Compared with single-slice CT (SSCT), patient radiation dose during a MDCT study may be higher because of: 1. use of a cone beam of radiation 2. acquisition at thinner section widths 3. higher-powered x-ray tubes a. 1 only b. 1 and 2 only c. 1 and 3 only d. 1, 2 and 3

D. CT is inferior to general radiography The spatial resolution of a CT image does not compare favorably with other modalities. Where CT excels is in its low-contrast resolution, for which it is superior to all other clinical modalities.

Comparing the spatial resolution of CT to other modalities: A) CT is superior to general radiography, MRI, and ultrasound B) CT is superior to general radiography, but is inferior to MRI C) CT is comparable to all other modalities D) CT is inferior to general radiography

C. top of the apices through liver The adrenal gland and liver are common sites for metastatic disease in patients with primary lung neoplasm. Scanning may be done in a caudocranial direction to ensure maximum contrast enhancement of the liver and adrenals when indicated.

Complete CT examinations of the chest for investigation of bronchogenic carcinoma should include sections from the: a. mandible through the liver b. apices to the diaphragm c. top of the apices through the liver d. clavicles through the adrenals

D. T wave Complete cardiac diastole is the period of atrial and ventricular relaxation after heart contraction. The T wave of an electrocardiogram (ECG) corresponds to cardiac diastole.

Complete cardiac diastole corresponds to which portion of the cardiac cycle on an electrocardiogram (ECG)? a. P wave b. QRS complex c. alpha wave d. T wave

B. scaphoid

Complex fractures of the wrist are most common in which of the following carpal bones? a. hamate b. scaphoid c. capitate d. triquetrum

D. 1, 2 and 3 -- precontrast sequence of the brain, CTA of the brain and carotids, and CTP of the brain Found typically in the emergency setting, a comprehensive acute stroke CT imaging protocol begins with a precontrast sequence to evaluate for hemorrhage. After the IV administration of an iodinated contrast agent, a CTA acquisition is performed throughout he carotid vessels and brain. CTP is then performed to evaluate potential cerebral perfusion defects secondary to stroke.

Components of a comprehensive CT stroke management protocol typically include: 1. precontrast sequence of the brain 2. CT angiogram (CTA) of the brain and carotids 3. CT perfusion (CTP) of the brain a. 2 only b. 1 and 2 only c. 2 and 2 only d. 1, 2 and

A. 1 and 2 only -- thorough hand washing between patients and wearing of disposable gloves Aseptic technique: cleaning venipuncture sites before needle insertion; applying pressure with alcohol swab to site after removal of needle or catheter Sterile technique: establishing sterile field

Components of the aseptic technique utilized for the IV administration of contrast agents include: 1. thorough hand washing between patients 2. wearing of disposable gloves 3. establishment of a sterile field around the site a. 1 and 2 only b. 1 and 3 only c. 2 and 3 only d. 1, 2 and 3

Decreases contrast in the radiographic image

Compton interaction:

A. read-only memory (ROM) ROM is a type of solid-state memory. It is part of the system's primary memory. Primary storage refers to the computer's internal memory. It is accessible to the CPU without the use of the computer's input/output channels.

Computer memory that is imprinted at the factory and is used to store frequently used instructions such as those required for starting the CT system is called: A) read-only memory (ROM) B) serial access memory (SAM) C) write-once-read-many times (WORM) memory D) compact disk-recordable (CD-R)

A. Bismuth Placed directly over radiosensitive tissues and organs during CT acquisition. Referred to as an "in-plane" type of shielding, the bismuth material may be placed within the acquisition area, resulting in minimal image artifact. The clinical use of bismuth shielding is controversial, with the American Association of Physicists in Medicine (AAPM) recommending against the use of bismuth shielding in favor of more comprehensive dose reduction techniques.

Contact shields made of ______________ may be used to selectively protect radiosensitive organs that lie within the scanned region during a CT acquisition. a. bismuth b. aluminum c. lead d. molybdenum

A. an acute impairment of renal function that follows the intravascular administration of contrast material and for which alternative causes have not been excluded The common measurement of contrast induced nephropathy is a reported rise in serum creatinine of 25% above baseline or an absolute rise of 0.5mL/dL within 48 hours of receiving an iodinated contrast agent.

Contrast media induced nephropathy can be defined as: A) an acute impairment of renal function that follows the intravascular administration of contrast material and for which alternative causes have not been excluded B) chronic renal failure that is associated with diabetes mellitus C) a short period of mildly reduced kidney function that does not produce symptoms D) any renal impairment, whether mild or severe, that occurs within a month of the intravenous administration of contrast material

B. atherosclerotic disease The primary clinical indication for MDCT coronary artery calcium (CAC) quantitation is the assessment of atherosclerotic disease. The presence of CAC is a specific indicator of atherosclerotic disease.

Coronary artery calcium (CAC) quantitation by MDCT examination is used primarily to assess: a. aortic aneurysm b. atherosclerotic disease c. coronary artery stenosis d. ejection fraction

CT manufactures and professional associations are all working towards a better method or tracking and documenting patient doses. What organization is responsible for setting the reference levels for tracking patient dose? A) NEMA and ACR B) OSHA C) HIPPA D) ACR

D) ACR ***The ACR in use with their CT Accreditation Program are working toward a better method of tracking and documenting patient doses in addition to ensuring that examinations are conducted with the least amount of radiation possible. The ACR has established reference levels for each specific patient protocol requiring an institution to investigate a protocol if it exceeds the reference levels and then determine if a lower radiation level can be utilized without a loss of image quality.***

What reconstruction technique was used by Hounsfield in the first EMI brain scanner? A) Filtered back projection reconstruction B) Multiplanar reconstruction C) Iterative reconstruction D) Algebraic reconstruction

D) Algebraic reconstruction ***the simplest form of reconstruction was the algebraic method of reconstruction performed by Hounsfield. This method used linear system of equations to solve a problem.***

What is the principal advantage that 180-degree linear z-interpolation has over 360-degree linear z-interpolation? A) Better temporal resolution B) Better spatial resolution C) Fewer image artifacts D) Better z-axis resolution

D) Better z-axis resolution ***The 180-degree linear z-interpolation technique substantially narrowed the slice sensitivity profile because the distance between corresponding projections in the real and virtual spirals are less than between the corresponding projections in the real spiral alone. The data set generation of thinner slices, and scanning at higher pitches.***

Which of the following networking topologies uses a common channel to connect all devices? A) Star networking B) Ring networking C) Redundant array of independent disk topology D) Bus networking

D) Bus Networking ***Bus networking topology uses a common channel to connect all devices. A single cable (Ethernet cable) function as a shared communication, medium that devices, such as computer and printer, attached or tap into with an interface connector. This topology works best when there are a limited number of devices.***

Which of the following CT examinations utilizes the injection of contrast media and air into a joint space? A) CTA of the aorta B) CT cystogram C) CT portogram D) CT arthrogram

D) CT arthrogram

Which of the following CT examination is performed with the use of an intrathecal injection? A) CT angiography B) Virtual colonoscopy C) CT enteroclysis D) CT myelography

D) CT myelography ***Commences with an injection of contrast media in the patients intrathecal space, which is located in the space surrounding the spinal cord. The purpose of this procedure is to demonstrate the spinal cord, spinal canal, and nerve roots.***

What is the first midline branch of the abdominal aorta? A) Inferior mesenteric artery B) Superior mesenteric artery C) Renal artery D) Celiac artery

D) Celiac artery ***Celiac artery or trunk is the first unpaired branch of the abdominal aorta. It's a very short vessel that leaves the anterior wall of the aorta just after the aorta passes through the diaphragm at the level of T12-L1***

Which of the following image display techniques has become the principle mode of image evaluation with the onset of MDCT? A) Region of interest B) Multiplanar reformation (MPR) C) Pixel analysis D) Cine mode

D) Cine mode ***Cine mode is an interactive display tool that gives the observer the ability to view large data sets in a comparatively short amount of time. Using this mode, the viewer has the capability of examining images in the axial, coronal, or sagittal planes alone or using a combination of planes. The user has full control of the cine loop with the use of a mouse or trackball.***

Which of the following pathological conditions is characterized by inflammation of the bowel mucosa, bowel wall, and mesentery with marked submucosal edema? A) Appendix B) Angiomyolipoma C) Kidney stones D) Crohn's disease

D) Cohn's disease

Which of the following medication is delivered orally prior to a CT of the abdomen/pelvis examination? A) Barbiturate sedative medications before CT scanning B) Nonionic contrast media C) Suppository D) Diatrizoate meglumine

D) Diatrizoate meglumine ***Is intended to be therapeutically and biologically inert when ingested/injected into the body for use in organ or tissue enhancement.***

What is the term used to address the dose an individual receives annually or accumulates over a working lifetime? A) Stochastic effects B) Deterministic effects C) Dose justification D) Dose limitations

D) Dose limitations ***The concept of dose limitations is a major integral component of regulatory guidance on radiation protection. This concept addresses the dose an individual receives annually or accumulates over a working lifetime.***

The kidneys are located in the retroperitoneum are bound by a band of fibrous connective tissue; what is the name of this fibrocartilaginous tissue? A) Coopers ligament B) Fascia of Camper C) Linea alba D) Gerota's fascia

D) Gerota's Fascia ***Is perirenal fat that surrounds and protects the kidneys.***

The liver is surrounded by a strong connective tissue, which gives shape and stability to the soft hepatic tissue. What is this tissue called? A) Gibson's compartment B) Gerota's fascia C) Morrison's pouch D) Glisson's capsule

D) Glisson's capsule ***The liver is surrounded by a strong connective tissue referred to as Glisson's capsule that gives shape and stability to the soft hepatic tissue.***

Which of the following essential questions is not included in the patient history taking? A) Are you currently taking any medications B) Have you ever had a reaction to contrast media C) Have you ever received radiation treatments D) How many children do you have

D) How many children do you have ***A focused evaluation is necessary to assess the patient's degree of risk for adverse events during the CT procedure.***

Compared with conventional computed tomography scanners, slip-rings offer a number of advantages. Which of the following is NOT an advantage of slip-ring technology? A) Removal of cable wraparound B) Elimination of start-stop action C) Continuous rotation of x-ray tube with minimal interscan delays D) Inability to perform CT angiography

D) Inability to perform CT angiography ***Slip-ring technology facilitates continuous rotation of the x-ray tube in order to produce volume data that is so important with CT angiography protocols.***

Which of the following is a manufacture features that aids in the rectification of metal artifacts? A) Cardiac gating B) Use of low HU-value oral contrast C) Dual energy CT scanners D) Interpolation techniques to substitute the over range values in attenuation profile

D) Interpolation techniques to substitute the over range values in attenuation profile ***The large gaps in detectors measurements created by metal edges are amplified by the filter in filter back projection: therefore, interpolation techniques are used to substitute for the over range values in the attenuation profile.***

Surgical aseptic techniques should be employed during which of the following CT procedures? A) Administration of rectal contrast B) Unenhanced sequences C) Administration of oral contrast D) Interventional CT procedures

D) Interventional CT procedures

Which of the following medication has the potential risk of producing lactic acidosis? A) Advil B) Heparin C) Lasix D) Metformin

D) Metformin ***Is excreted without any decomposition of the medication by the kidneys; therefor the renal route eliminates approximately 90% of the absorbed drug within the first 24 hrs. Metformin increases the lactic acid production by the intestines.***

The pulse or heart rate is defined as the rate at which the heart beats in 1 minute. A pulse can be palpable where an artery is near the surface of the body. Where can the dorsalis pedis pulse be found? A) Lateral aspect of the wrist B) Behind the knee C) Apex of the heart D) Over the instep of the foot

D) Over the instep of the foot

The size of a pixel can be calculated by which of the following mathematical calculations? A) Pixel size = matrix X voxel size B) Pixel size = FOV X matrix size C) Pixel size = FOV/ the entire volume of tissue scanned D) Pixel size= FOV/ matrix

D) Pixel size = FOV / matrix

Which of the following is NOT a function of the DAS? A) Measures the detected signal B) Converts the detected signal to analog form C) Transmits the detected signal to the computer D) Post processes the detected signal

D) Post processes the detected signal ***The data acquisition system's main functions are to measure the transmitted radiation beam, encode these measurements into binary data, and lastly to transmit the binary data to the computer.***

Which of the following x-ray tube anodes is most likely used in spiral/helical computed tomography scanners? A) Tungsten target fixed anode tube B) Tungsten target rotating anode tube C) Molybdenum target rotating anode tube D) Rotating anode tube with a rhenium tungsten disk

D) Rotating anode tube with a rhenium tungsten disk ***Modern CT scanners employs a rotating anode tube with a rhenium tungsten disk. The base of the disk is composed of graphite to increase the heat storage capacity. These large lightweight disk have a large heat storage capacity and a fast cooling rate to accompany the high demands of spiral/helical scanning.***

Name an advantage of gas ionization detectors: A) Lower quantum detector efficiency B) Produce afterglow C) Utilize hydrogen in the ionization chamber D) Sensitivity of the individual chambers are uniform

D) Sensitivity of the individual chambers are uniform ***Xenon gas chambers offer several advantages. First their construction is relatively simple in principle and the sensitivity of the individual chambers is exactly the same, since constant pressure exists for the complete set of detector pressure channels.***

What section of the large intestine is an S-shaped structure located at the terminal end of the descending colon originating at the lower left side of the abdominal cavity extending to midline? A) Ascending colon B) Rectum C) Splenic flexure D) Sigmoid

D) Sigmoid

What is deemed an overlapping reconstruction interval? A) Slice thickness 5mm, slice interval 5mm B) Slice thickness 3mm, slice interval 7mm C) Slice thickness 1mm, slice interval 2mm D) Slice thickness 1mm, slice interval 0.5mm

D) Slice thickness 1mm, slice interval 0.5mm ***An overlapping reconstruction interval is one which the interval is half the thickness improve image quality and decreases partial volume averaging.***

Which of the following combinations will produce an image with the best spatial resolution? A) Large pixels and thin slice width B) Large pixels and thick slice width C) Small pixels and thick slice width D) Small pixels and thin slice width

D) Small pixels and thin slice width ***The combination of small pixels and thin slice widths produces images with the least amount of volume averaging yielding a sharper image; hence, better spatial resolution.***

What pathological condition occurs due to a sufficiently reduced spinal canal resulting in nerve root impingement from causes such as: osteophyte formation, disk herniation, and ligamentum flava hypertrophy? A) Spinal bifida B) Spondylolysis C) Disc herniation D) Spinal stenosis

D) Spinal stenosis ***Is a condition where the spinal canal is sufficiently reduced in diameter size resulting in nerve root impingement from such causes as osteophyte formation and/or disk herniation.***

Which detector characteristic refers to the reliability of producing a useful signal? A) Response time B) Dynamic range C) Efficiency D) Stability

D) Stability ***Refers to the steadiness of the detectors response. Instability causes a signal not to be useful. It's an important characteristic of detectors, since it will be free or almost free from change, variation, or fluctuation, and will produce a uniform signal.***

Which of the following cranial nerves supplies motor fibers for the muscle of mastication? A) Olfactory B) Trochlear C) Abducens D) Trigeminal

D) Trigeminal ***The trigeminal is a motor and sensory nerve of the face consisting of three major divisions, ophthalmic (V1), maxillary (V2), and mandibular (V3). It supplies motor fibers for the muscles of mastication.***

How long following the administration of an IV contrast media should a dialysis patient be dialyzed? A) Within 2 hours B) Within 6 hours C) Within 12 hours D) When they are regularly scheduled

D) When they are regularly scheduled ***According to the American College of Radiology (ACR) manual on contrast media, a dialysis patient undergoing an enhanced CT examination does not need to be immediately dialyzed, since the contrast agents are not protein-bound.***

What is the equation for DFOV?

DFOV = pixel size X matrix size

D. linearity The accuracy between calculated CT numbers and their respective linear attenuation coefficients is termed LINEARITY. Daily system calibration and routine vendor maintenance procedures work to establish and maintain CT system linearity.

Daily air or water calibration of the CT system is performed to evaluate which of the following components of image quality? a. contrast resolution b. temporal resolution c. uniformity d. linearity

Somatic

Damage to the cell being irradiated is called:

A. CT venography Shortly after a CTA of the pulmonary arteries (CTPA) study, CT venography (CTV) of the lower extremities for the identification of deep vein thrombosis (DVT) may also be performed. After the CTPA acquisition, a delay of 2-3 minutes is employed, and an acquisition is made from the iliac crest to the ankles. This may be commonly referred to as a CT runoff.

Delayed postcontrast acquisition of the lower extremities after a CTA` of the pulmonary arteries is termed: a. CT venography b. CT perfusion c. femoral CTA d. iliac CTA

Transformers

Devices in the x-ray circuit that operate on the principle of mutual induction are called:

B. steatosis Hepatic steatosis, or fatty liver disease, is a condtion in which large quantities of fats (lipids) are retained within the liver. Normal liver parenchyma exhibit attenuation values at approximately 10 HU higher of the spleen, whereas a liver with steatosis has attenuation values at least 10 HU below those of the spleen.

Diffuse fatty infiltration of the hepatic parenchyma may be referred to as: a. chromatosis b. steatosis c. cirrhosis d. lipomatosis

D. DICOM The Digital Imaging and Communications in Medicine (DICOM) standard confirms the process of recording, storing, printing and transmitting medical image data.

Digital CT images are networked and archived in accordance with a standardized computer protocol identified by which of the following abbreviations? a. HIPAA b. HTTP c. TCP/IP d. DICOM

A. 1 only -- prior life-threatening reaction to iodine Direct contraindications: prior life-threatening reaction to iodinated contrast material Indirect contraindications: multiple myeloma, diabetes

Direct contraindications to the administration of iodinated contrast material include: 1. prior life-threatening reaction to iodinated contrast material 2. multiple myeloma 3. diabetes a. 1 only b. 1 and 2 only c. 1 and 3 only d. 1, 2 and 3

D. required on all patients and should include the name of the agent, the dose, the flow rate and the injection site Documentation of IV contrast administration is a legal necessity and should include all injection parameters. It should also include the date, the time of the injection, and the person administering it. Accurate documentation allows the study to be reproduced, should follow-up CT examinations be necessary.

Documentation of IV contrast administration is: A) only necessary when an adverse event occurs B) only necessary when routine injection parameters are modified C) required on all patients and must be cosigned by a radiologist D) required on all patients and should include the name of the agent, the dose, the flow rate, and the injection site

C) CTDI Volume ***Its a standardized measure of the radiation output of a CT system. To consider the dose in the z-axis the CTDI volume was introduced.

Dose descriptors are used to identify approximate patient doses, which of the following is the dose descriptor utilized for modern multi-slice CT scanners? A) Multiple-scan average dose (MSAD) B) Dose profile C) CTDI volume D) Roentgen

A. Joint space

During CT arthrography, iodinated contrast media is injected directly into the: a. joint space b. intrathecal space c. subarachnoid space d. venous bloodstream

D. vocal cords Phonating the letter "E' during a CT scan allows for the thorough evaluation of the vocal cords' mobility because of their vibrations.

During CT examination of the larynx, the patient is often instructed to phonate the letter "E" in order to properly evaluate the: a. epiglottis b. uvula c. trachea d. vocal cords

D. bismuth In-plane bismuth shielding of particularly radiosensitive areas such as the orbits, thyroid, and breast tissue can substantially reduce effective radiation dose.

During CT examinations of the chest and abdomen, the highly radiosensitive breast tissue can be protected with minimal image artifact with the use of specialized shielding composed of: a. barium b. aluminum c. lead d. bismuth

A. the rotational nature of the x-ray tube A patient receives exposure from all sides during a CT examination, due to the rotational nature of the x-ray tube. Therefor, lead shielding should be placed above and below the patient.

During CT examinations, the patient's body should be shielded above and below because of: a. the rotational nature of the x-ray tube b. extremely high kVp techniques c. lack of filtration of the CT beam d. reduced collimation of the CT beam

B. cerebral blood flow (CBF) Describes the quantity of blood (in mL) that moves through 100 g of brain tissue each minute. Normal CBF range in gray matter is 50 - 60 mL/100 g/min.

During CT perfusion studies, the quantity of blood (in mL) that moves through 100 g of tissue each minute is termed the: a. mean transit time (MTT) b. cerebral blood flow (CBF) c. cerebral blood volume (CBV) d. percentage washout value (PWV)

A. 15 degrees above the IOML An angle of 15 degrees above the infraoribital-meatal line produces axial sections of the brain, while limiting beam-hardening artifacts and direct orbital exposure.

During CT scanning of the head, the gantry should be angled: a. 15 degrees above the infraorbital-meatal line b. 10 degrees below the IOML c. 0 degrees d. 20 degrees above the skull base

D. constant mAs The mA setting in coordination with the scan time (seconds) gives the constant mAs for a CT acquisition.

During CT x-ray exposure, the product of thee selected mA setting and the scan time is called the: a. effective mAs b. peak mAs c. absorbed mAs d. constant mAs

D. flying focal spot Involves the electromagnetic steering of the electron beam emitted from the cathode. The beam of electrons is directed toward two separate locations on the rotating anode, resulting in two sources of x-radiation. As the tube rotates around the patient, the number of data samples is essentially doubled because of the electronic switching between the two focal spots, improving the system's temporal and spatial resolution.

During CT x-ray production, the electromagnetic steering of the electron beam from the cathode to two alternating targets is referred to as: a. prospective gating b. ultrafast CT c. twin CAT d. flying focal spot

A. the cul-de-sac The area posterior to the uterus ans ovaries in female patients. The cul-de-sac is also a a common site for ascites in patients with pelvic pathology.

During a CT examination of a female pelvis for a suspected malignancy, ascites may be present in an area posterior to the uterus and ovaries known as: a. the cul-de-sac b. Morrison pouch c. the space of Retzius d. the prevesical compartment

B. superior vena cava The use of a saline flush reduces artifact from dense contrast in the superior vena cava. The streaking artifact can hamper visualization of the upper mediastinum and surrounding lung tissue.

During a CT examination of the chest, the administration of a saline flush after the bolus injection of iodinated IV contrast media helps alleviate artifact from dense contrast in the: a. ascending aorta b. superior vena cava c. descending aorta d. inferior vena cava

D. metabolic activity Malignant cells demonstrate an increase in metabolic activity and glucose utilization. Once administered, fludeoxyglucose F 18 (FDG) mimics glucose and is taken up by normal and abnormal tissue. The amount of FDG uptake is directly proportional to the area's metabolic activity. Malignant cells will take up a disproportionately larger amount of FDG than metabolically normal tissue.

During a PET-CT examination, the amount of fludeoxyglucose F 18 (FDG) uptake in an anatomic region is directly proportional to the area's: a. size b. central vs. peripheral location c. radiosensitivity d. metabolic activity

A. iliac crests to the pubic symphysis A complete CT scan of the pelvis should include from the iliac crests to the pubic symphysis.This scan range may be extended if clinically warranted. Scan parameters (i.e. slice thickness, incrementation) can be adjusted to correlate with the patient's clinical history.

During a complete CT scan of the pelvis, sections should be obtained from the: a. iliac crests to the pubic symphysis b. kidneys through the bladder c. bottom of the kidneys to the pubic symphysis d. iliac crests to the lesser trochanter

B. superior vena cava There is often significant artifact within the mediastinum from high concentrations of iodine in the superior vena cava. This artifact may be alleviated by administering a bolus of contrast agent with reduced iodine concentration, which is achieved by diluting a full-strength contrast agent with saline.

During a contrast-enhanced CT examination of the chest, streaking artifact may obscure portions of the superior mediastinum because of dense concentrations of iodine in the: a. descending aorta b. superior vena cava c. ascending aorta d. pulmonary arteries

B. 1.0 - 3.0 mL/sec This rate should be sufficient to provide the enhancement necessary for proper evaluation of the abdomen. This is a general range that may be adjusted to meet the needs of the examination at the discretion of the physician and medical personnel involved for each patient.

During a general CT examination of the abdomen, an intravenous contrast agent is indicated and is administered with the aid of automatic injector. Which of the following ranges of flow rate should be used? a. 0.2 - 1.0 mL/sec b. 1.0 - 3.0 mL/sec c. 4.0 - 6.0 mL/sec d. 7.0 - 10.0 mL/sec

B. automatic tube current modulation (ATCM) ATCM programs adjust the mA throughout an acquisition to reduce patient radiation dose to a minimum. ATCM automatically alters the applied mA on the basis of a predetermined noise index that is acceptable for appropriate image quality.

During data acquisition, a CT system may continuously adjust the mA relative to the measured image noise for a method of dose reduction referred to as: a. iterative reconstruction b. automatic tube current modulation c. attenuation correction d. Nyquist sampling

D. prone High-resolution CT (HRCT) images with the patient in the prone position can be acquired to differentiate the dependent edematous changes often seen in the lung bases.

During high-resolution CT (HRCT) of the lungs, edematous changes in the posterior lungs may be differentiated by positioning the patient in the _______________ position. a. supine b. right lateral decubitus c. left lateral decubitus d. prone

D. 2 and 3 only -- lead shielding and use of a needle holder Low values for kVp and mA and short CT fluoroscopy time all reduce occupational radiation exposure. Greater distance from the source, as with the use of a needle holder to keep the operator's hands out of the CT fluoroscopy field, also greatly diminish exposure. Additional methods of exposure reduction are lead shielding, leaded patient drapes, and the utilization of a last-image-hold function to cut down on continuous CT fluoroscopy.

During the use of CT fluoroscopy for guided interventional procedures, which of the following technical consideration may be used to reduce occupational radiation exposure? 1. increased kVp 2. lead shielding 3. use of a needle holder a. 2 only b. 1 and 2 only c. 1 and 3 only d. 2 and 3 only

A. effective section width The effective section width corresponds to the slice sensitivity profile (SSP) of the reconstructed section in consideration of the widening that occurs during helical data acquisition.

During volumetric CT acquisition, the slice sensitivity profile (SSP) graphically represents the: a. effective section width b. dose profile c. beam width d. detector collimation

C) Corticomedullary Phase ***this phase of injection occurs at approximately 20 to 25 seconds post injection***

During what phase of renal enhancement demonstrates the vascular anatomy of the kidneys? A) Pre-contrast B) Nephrographic phase C) Corticomedullary phase D) excretory phase

B) Nephrographic phase ***Nephrographic or parenchymal phase of injection occurs at approximately 100 to 180 seconds from the start of injection***

During what phase of renal enhancement do the renal cortex and medulla show equal levels of enhancement? A) Precontrast B) Nephrographic phase C) Corticomedullary phase D) Excretory phase

D. abdomen Misregistration is the loss of anatomic information that occurs when a patient suspends respiraiton at different depths during consecutive scans. Misregsitration occurs only during CT examinations in which suspended respiration of the patient is necessary. The ability of MDCT systems to volumetrically acquire entire anatomic areas in a single, short breath-hold has greatly reduced the occurrence of misregistration artifact.

During which of the following CT examinations is a misregistration artifact most likely to occur? a. brain b. pelvis c. neck d. abdomen

D. abdomen scan after IV contrast agent administration

During which of the following CT procedures is the patient required to give informed consent? A) noncontrast CT of the chest B) renal stone survey C) 3D reconstruction of the hip D) abdomen scan after IV contrast agent administration

C. nephrographic Nephrographic (70-90 seconds after contrast administration) imaging provides enhancement of the bladder wall, which is used to evaluate the extent of tumor infiltration.

During which of the following IV contrast phases does the urinary bladder wall enhance? a. early arterial b. corticomedullary c. nephrographic d. excretory

The range of exposures over which a detector can acquire image data

Dynamic range is defined as:

Is the level of radiation that an organism can receive and probably sustain no appreciable damage

Effective dose limit:

B. a disadvantage often associated with the interpolation of helical data Interpolation methods used with helical data can result in a scan that is wider than that selected by the operator. This is referred to as slice thickness blooming or degradation of the slice sensitivity profile.

Effective slice-thickness blooming is: A) the process in CT by which different tissue attenuations are averaged to produce one less accurate pixel reading B) a disadvantage often associated with the interpolation of helical data C) an improvement to image resolution associated with helical scans D) the trend toward selecting a wider slice thickness to improve spatial resolution

D. During MDCT cardiac studies, prospective gating can be used to reduce the patient radiation dose. Electrocardiogram (ECG)-triggered tube current modulation allows for pulses of x-ray energy rather than continuous exposure to be used. Tube current is reduced during the cardiac phase not utilized for image reconstruction.

Electrocardiogram (ECG)-gated tube current modulation may effectively reduce patient radiation dose during which of the following CT procedures? a. CT colonography b. brain perfusion CT c. CT urogram d. cardiac CT

C. one CT examination is equal to from 100 - 250 chest radiographs Most physicians, medical personnel and patients underestimate the dose of a CT examination, based on a 2004 survey.

Estimate the radiation dose for one CT examination versus the dose for one chest radiograph A) one CT examination is less than or equal to one chest radiograph B) one CT examination is greater than one chest radiograph, but less than 100 chest radiographs C) one CT examination is equal to from 100-250 chest radiographs D) one CT examination is equal to or greater than 500 chest radiographs

A. 0.5 and 1.2 mm Modern CT system x-ray tubes utilize dual-filament systems with focal spot sizes ranging between 0.5 and 1.2 mm IN DIAMETER.

Filament sizes for modern CT x-ray tubes range between: a. 0.5 and 1.2 mm b. 2.0 and 3.0 mm c. 5.5 and 7.8 mm d. 25.0 and 31.5 mm

Energy

Filters made of aluminum and copper are placed in the film badge to measure x-ray:

Compensating

Filters that even out density of irregular anatomy are called:

A. pencil-thin x-ray beam The x-ray beam of a first generation CT scanner was highly collimated to the size of a single detector. It was often referred to as PENCIL BEAM.

First-generation CT scanners possess which of the following characteristics? a. pencil-thin x-ray beam b. silver halide detectors c. rotate-rotate geometry d. nutating detector array

C. translate-rotate First-generation CT scanners acquired data through a process based on the principle of tube translation and rotation around the patient's head.

First-generation CT scanners used a method of data acquisition based on a ______________ principle. a. multiplanar b. rotation-only c. translate-rotate d. transaxial

Anaphylactic shock

Flushing, urticaria, and nausea are symptoms of what type of contrast agent reaction?

A. hypervascular Upon IV contrast administration, a tumor may be described as HYPOVASCULAR if it is lower in density than the surrounding organ parenchyma, or HYPERVASCULAR if it is higher in density.

Following the IV administration of iodinated contrast media, a tumor that exhibits a greater density than the surrounding parenchyma is characterized as: a. hypervascular b. isovascular c. hypovascular d. cavernous

D. axial The axial plane is parallel to the plantar surface of the foot.

For a CT of the foot the patient is positioned supine, legs flat on the table, toes pointed straight up. There is no gantry tilt. Images acquired are in the direct ________ plane. A) sagittal B) oblique C) coronal D) axial

High kVp, low mAs

For optimal radiation protection, what type of exposure technique should be used?

C. 2.0 mL/s to 4.0 mL/s

For routine CT of the chest, abdomen or pelvis, the range of suggested IV contrast flow rate is: A) 0.2 mL/s to 0.5 mL/s B) 1.0 mL/s to 2.0 mL/s C) 2.0 mL/s to 4.0 mL/s D) 6 mL/s to 12 mL/s

C. rotate-stationary Fourth-generation CT scanners operate with a rotating x-ray tube and a stationary ring of detectors. Some fourth generation scanners also use a rotating x-ray tube with a nutating detector ring.

Fourth-generation CT scanners use a _____________ tube-detector configuration. a. rotate-translate b. electron beam-stationary c. rotate-stationary d. rotate-rotate

C. 70 The appearance of an intracranial hemorrhage (ICH) will change with the passage of time, as red blood cells begin to deteriorate outside the vasculature. As a general rule, *ICH will appear as hyperdense to normal brain tissue for approximately 3 days, after which it will gradually decrease in density.*

Fresh blood in the brain, as in the case of a recent-onset subdural hematoma, will measure approximately: A) -80HU B) 20 HU C) 70 HU D) 150 HU

Four silicon-based semiconductors

Full-wave rectification uses:

D) Bone cancer ***The most common malignant primary bone tumor is osteosarcoma. The tumors are commonly located in the knee, distal femur, or the proximal tibia and is seen in mostly the younger population***

Give the name of this primary cancer that has an etiology of unknown origin, mostly diagnosed in the younger population, but individuals exposed to radiation have a predisposition linked to cancer? A) Kidney cancer B) Lung cancer C) Brain cancer D) Bone cancer

D) Bone cancer ***The most common malignant primary bone tumor is osteosarcoma. The tumors are commonly located in the knee, distal femur, or the proximal tibia and is seen in mostly the younger population***

Give the name of this primary cancer that has an etiology of unknown origin, mostly diagnosed in the younger population, but individuals exposed to radiation have a predisposition linked to cancer? A) Kidney cancer B) Lung cancer C) Brain cancer D) Bone cancer

A. positive agents A positive agent creates image contrast because it has a higher density than surrounding tissues. *Iodine and barium sulfate are examples of positive agents.*

High-density contrast agents, such as barium sulfate, are referred to as: A) positive agents B) negative agents C) neutral agents D) ionic agents

C. lungs HRCT is a specialized technique using narrow section widths and a high-resolution algorithm for image reconstruction. It is used to maximize detail of high spatial frequency tissue, such as the lungs and bony structures.

High-resolution computed tomography (HRCT) is most commonly used for the evaluation of the: a. pancreas b. brain c. lungs d. pelvis

Free radicals

Highly reactive ions that have unpaired electrons in the outer shell are called:

C. smaller objects, which are scanned with a small scan field, always absorb a much higher dose than do larger objects, which are scanned with a large scan field Smaller objects always absorb a higher dose by a factor of at least two. This effect is primarily attributed to the fact that total exposure is made up of both entrance radiation and exit radiation. For smaller patients, the patient has less tissue to attenuate the beam, which results in a much more uniform dose distribution. Conversely, for a larger patient the exit radiation is much less intense owing to its attenuation through more tissue.

How does object size and scan field diameter affect radiation dose to the patient? A) scan field diameter has no effect; it is the display field diameter that has an impact on dose B) changing from a small scan field, such as that used for a head scan, to a large scan field, such as that used for a body scan, will have a negligible effect on the radiation dose. It could either reduce or increase the dose slightly, depending on the patient factors C) smaller objects, which are scanned with a small scan field, always absorb a much higher dose than do larger objects, which are scanned with a large scan field D) smaller objects, which are scanned with a small scan field, always absorb a much lower dose than do larger objects, which are scanned with a large scan field

B) cursor measurements are the HU measurement for a single pixel, whereas an ROI is the average HU for all pixels within the ROI Because the ROI is the average HU for all the pixels within the region, the standard deviation can also be calculated. Both a cursor measurement and an ROI use image data. Neither indicates what scan parameters were used for data acquisition.

How is a cursor measurement different than obtaining a region of interest measurement? A) cursor measurements provide only the standard deviation, whereas an ROI provides an HU measurement B) cursor measurements are the HU measurement for a single pixel, whereas an ROI is the average HU for all pixels within the ROI C) cursor measurements use raw data, whereas an ROI uses image data D) cursor measurements show what mAs was used to create the image, whereas an ROI obtains both mAs and kVp settings

D) Raising the kVP ***kVp is a qualitative measure of the x-ray beam, therefor an increase in kVp produces an increased on photon energy. This typical ranges of kVp in CT are 100 to 140 kVp***

How is it possible to increase the energy of a CT examination? A) Raising the mAs B) Lowering the kVp C) Changing the scanned field of view D) Raising the kVP

D) Raising the kVP ***kVp is a qualitative measure of the x-ray beam, therefor an increase in kVp produces an increased on photon energy. This typical ranges of kVp in CT are 100 to 140 kVp***

How is it possible to increase the energy of a CT examination? A) Raising the mAs B) Lowering the kVp C) Changing the scanned field of view D) Raising the kVP

8 Carpals

How many carpals are in the wrist

D) 4 valves

How many valves does the heart have? A) 1 B) 2 C) 3 D) 4

3) Splenic vein A) Splenic vein B) Superior mesenteric vein C) Inferior mesenteric vein

Identify structure *A* in the hepatic system. 1) Superior mesenteric vein 2) Inferior mesenteric vein 3) Splenic vein

3) Renal artery A) Renal artery B) Renal vein C) Superior mesenteric artery

Identify structure *A* in this anterior view of the renal veins and arteries. 1) Superior mesenteric artery 2) Renal vein 3) Renal artery

3) Sacroiliac joint A) Sacroiliac joint B) Lateral mass C) Sacral promontory D) Ilium

Identify structure *A* in this coronal CT slice of the sacroiliac joints. 1) Sacral promontory 2) Lateral mass 3) Sacroiliac joint 4) Ilium

1) Iliacus muscle A) Iliacus muscle B) Psoas muscle C) Sacral promontory D) Right external iliac artery E) Ilium)

Identify structure *A* on this axial CT pelvis slice of internal/external iliac arteries and veins. 1) Iliacus muscle 2) Psoas muscle 3) Sacral promontory 4) Right external iliac artery 5) Ilium

3) Anterior arch A) Anterior arch B) Posterior arch C) Lateral mass D) Transverse foramen E) Odontoid process

Identify structure *A* on this axial CT slice of the C1. 1) Posterior arch 2) Lateral mass 3) Anterior arch 4) Odontoid process 5) Transverse foramen

1) Bare area of liver A) Bare area of liver B) Right subphrenic compartment C) Left subphrenic compartment D) Spleen E) Stomach F) Azygous vein

Identify structure *A* on this axial CT slice of the abdomen. 1) Bare area of liver 2) Azygos vein 3) Right subphrenic compartment 4) Stomach 5) Left subphrenic compartment 6) Spleen

3) Abdominal aorta A) Abdominal aorta B) Portal vein C) Inferior vena cava D) Body of pancreas E) Spleen

Identify structure *A* on this axial CT slice of the abdomen. 1) Body of pancreas 2) Spleen 3) Abdominal aorta 4) Portal vein 5) Inferior vena cava

1) Left psoas muscle A) Left psoas muscle B) Inferior vena cava C) Left ureter D) Internal oblique muscle

Identify structure *A* on this axial CT slice of the abdomen. 1) Left psoas muscle 2) Left ureter 3) Internal oblique muscle 4) Inferior vena cava

1) Rectus abdominis muscle A) Rectus abdominis muscle B) Linea alba C) Right ureter D) Quadratus lumborum muscle E) Right and left common iliac arteries

Identify structure *A* on this axial CT slice of the abdomen. 1) Rectus abdominis muscle 2) Linea alba 3) Right ureter 4) Quadratus lumborum muscle 5) Right and left common iliac arteries

2) Anterior pararenal space A) Anterior pararenal space B) Gerota's fascia C) Renal pelvis D) Abdominal aorta E) Renal calyx

Identify structure *A* on this axial CT slice of the abdomen. 1) Renal pelvis 2) Anterior pararenal space 3) Gerota's fascia 4) Renal calyx 5) Abdominal aorta

4) Right adrenal gland A) Right adrenal gland B) Right crus C) Celiac axis D) Spleen

Identify structure *A* on this axial CT slice of the abdomen? 1) Spleen 2) Celiac axis 3) Right crus 4) Right adrenal gland

3) Choroid plexus A) Choroid plexus B) Vein of Galen C) Internal cerebral veins D) Thalamostriate vein E) Straight sinus

Identify structure *A* on this axial CT slice of the brain. 1) Vein of Galen 2) Thalamostriate vein 3) Choroid plexus 4) Straight sinus

2) Sylvian fissure A) Sylvian fissure B) Insula C) Quadrigeminal cistern D) Falx cerebri E) Colliculi F) Cerebellum

Identify structure *A* on this axial CT slice of the cerebral peduncles. 1) Quadrigeminal cistern 2) Sylvian fissure 3) Insula 4) Falx cerebri

4) Trachea A) Trachea B) Esophagus C) Left subclavian artery D) Left common carotid artery E) Brachiocephalic artery F) Right brachiocephalic vein G) Left brachiocephalic vein

Identify structure *A* on this axial CT slice of the chest. 1) Brachiocephalic artery 2) Left brachiocephalic vein 3) Left common carotid artery 4) Trachea 5) Left subclavian artery 6) Right brachiocephalic vein 7) Esophagus

2) Carina A) Carina B) Left mainstem bronchus C) Sternum D) Secondary bronchi E) Descending aorta F) Ascending aorta

Identify structure *A* on this axial CT slice of the chest. 1) Descending aorta 2) Carina 3) Sternum 4) Secondary bronchi 5) Ascending aorta 6) Left mainstem bronchus

5) Superior vena cava A) Superior vena cava B) Azygos vein C) Hemiazygos vein D) Pulmonary trunk E) Trapezius muscle F) Intercostal muscle

Identify structure *A* on this axial CT slice of the chest. 1) Trapezius muscle 2) Intercostal muscle 3) Hemiazygos vein 4) Azygos vein 5) Superior vena cava 6) Pulmonary trunk

2) Round ligament A) Round ligament B) Uterus C) Rectouterine pouch D) Uterosacral ligament E) Rectum

Identify structure *A* on this axial CT slice of the female pelvis. 1) Uterus 2) Round ligament 3) Rectum 4) Uterosacral ligament 5) Rectouterine pouch

2) Palatine process of maxilla A) Palatine process of maxilla B) Palatine bones C) Pterygoid process

Identify structure *A* on this axial CT slice of the hard palate. 1) Palatine bones 2) Palatine process of maxilla 3) Pterygoid process

1) Left atrium A) Left atrium B) Left ventricle C) Right atrium D) Right ventricle E) Ascending aorta

Identify structure *A* on this axial CT slice of the heart. 1) Left atrium 2) Right atrium 3) Right ventricle 4) Ascending aorta 5) Left ventricle

5) Nucleus pulposus A) Nucleus pulposus B) Annulus fibrosus C) Superior articular process D) Inferior articular process E) Apophyseal joint

Identify structure *A* on this axial CT slice of the lumbar spine with intervertebral disk. 1) Annulus fibrosus 2) Superior articular process 3) Apophyseal joint 4) Inferior articular process 5) Nucleus pulposus

1) Body A) Body B) Pedicle C) Lamina D) Transverse process E) Posterior spinous process F)Vertebral foramen

Identify structure *A* on this axial CT slice of the lumbar vertebra. 1) Body 2) Transverse process 3) Vertebral foramen 4) Posterior spinous process 5) Lamina 6) Pedicle

3) Seminal vesicles A) Seminal vesicles B) Rectum C) Spermatic cord D) Bladder E) Acetabulum F) Ischial spine

Identify structure *A* on this axial CT slice of the male pelvis. 1) Bladder 2) Ischial spine 3) Seminal vesicles 4) Rectum 5) Spermatic cord

1) Masseter muscle A) Masseter muscle B) Temporalis muscle C) Lateral pterygoid muscle

Identify structure *A* on this axial CT slice of the neck with muscles of mastication. 1) Masseter muscle 2) Lateral pterygoid muscle 3) Temporalis muscle

4) Epiglottis A) Epiglottis B) Internal jugular vein C) Vallecula D) Hyoid bone E) Sternocleidomastoid muscle

Identify structure *A* on this axial CT slice of the neck with oropharynx. 1) Vallecula 2) Internal jugular vein 3) Hyoid bone 4) Epiglottis 5) Sternocleidomastoid muscle

2) Submandibular gland A) Submandibular gland B) Sublingual gland C) Sternocleidomastoid muscle

Identify structure *A* on this axial CT slice of the neck. 1) Sternocleidomastoid muscle 2) Submandibular gland 3) Sublingual gland

1) Thyroid cartilage A) Thyroid cartilage B) Arytenoid cartilage C) Rima glottis D) Vocal cord E) Platysma F) Anterior jugular vein

Identify structure *A* on this axial CT slice of the neck. 1) Thyroid cartilage 2) Arytenoid cartilage 3) Rima glottis 4) Platysma muscle 5) Anterior jugular vein 6) Vocal cord

3) Lens A) Lens B) Posterior chamber C) Optic canal D) Optic nerve E) Medial rectus muscle

Identify structure *A* on this axial CT slice of the orbit. 1) Posterior chamber 2) Optic canal 3) Lens 4) Optic nerve

1) Foramen magnum A) Foramen magnum B) Foramen spinosum C) Foramen ovale D) Mandibular condyle E) Greater wing of sphenoid F) Clivus

Identify structure *A* on this axial CT slice of the sphenoid bone. 1) Foramen magnum 2) Foramen spinosum c) Foramen ovale 4) Clivus

1) Carotid canal A) Carotid canal B) Jugular foramen C) Sphenoid sinus D) Ethmoid sinus E) External auditory meatus F) Mastoid air cells

Identify structure *A* on this axial CT slice of the temporal bone. 1) Carotid canal 2) Jugular foramen 3) Sphenoid sinus 4) Ethmoid sinus

4) Mandibular condyle A) Mandibular condyle B) Temporalis muscle C) Masseter muscle D) Medial pterygoid muscle E) Lateral pterygoid muscle

Identify structure *A* on this axial CT slice of the temporomandibular joint. 1) Medial pterygoid muscle 2) Masseter muscle 3) Temporalis muscle 4) Mandibular condyle

3) Epidural fat A) Epidural fat B) Dorsal root C) Ventral root D) Conus medullaris E) Costovertebral articulation

Identify structure *A* on this axial CT slice of the thoracic vertebra. 1) Dorsal root 2) Ventral root 3) Epidural fat 4) Costovertebral articulation 5) Conus medullaris

2) Malleus A) Malleus B) Incus C) Stapes D) Cochlea

Identify structure *A* on this axial CT slice of the unilateral petrous portion at the level of the external auditory meatus. 1) Cochlea 2) Malleus 3) Stapes 4) Incus

3) Anterior cerebral artery A) Anterior cerebral artery B) Posterior cerebral artery C) Anterior communicating artery D) Posterior communicating artery E) Middle cerebral artery

Identify structure *A* on this axial CTA of the Circle of Willis. 1) Posterior communicating artery 2) Anterior communicating artery 3) Anterior cerebral artery 4) Posterior cerebral artery

3) Inferior nasal concha A) Inferior nasal concha B) Maxillary sinus C) Crista galli D) Ethmoid sinus E) Alveolar process of maxilla

Identify structure *A* on this coronal CT slice of the ethmoid bone. 1) Crista galli 2) Maxillary sinus 3) Inferior nasal concha 4) Alveolar process of maxilla

1) Optic canal A) Optic canal B) Superior orbital fissure C) Inferior orbital fissure D) Anterior clinoid process

Identify structure *A* on this coronal CT slice of the orbit. 1) Optic canal 2) Superior orbital fissure 3) Inferior orbital fissure 4) Anterior clinoid process

2) Zygoma A) Zygoma B) Ethmoid bulla C) Infundibulum D) Middle nasal meatus E) Uncinate process F) Orbital plate of frontal bone

Identify structure *A* on this coronal CT slice of the osteomeatal complex. 1) Middle nasal meatus 2) Zygoma 3) Ethmoid bulla 4) Infundibulum

4) Pelvic brim A) Pelvic brim B) Pectineal line C) Arcuate line D) Symphysis pubis E) Iliac fossa

Identify structure *A* on this coronal CT slice of the pelvic brim. 1) Arcuate line 2) Pectineal line 3) Iliac fossa 4) Pelvic brim 5) Symphysis pubis

4) Sacral promontory A) Sacral promontory B) Sacral foramina C) Lateral mass (Ala) D) Sacroiliac joint E) Body of S1

Identify structure *A* on this coronal CT slice of the sacrum. 1) Lateral mass (Ala) 2) Sacroiliac joint 3) Body of S1 4) Sacral promontory 5) Sacral foramina

4) Pterygoid process A) Pterygoid process B) Pterygoid hamulus C) Temporal bone D) Anterior clinoid process E) Sphenoid sinus F) Ramus of mandible

Identify structure *A* on this coronal CT slice of the sphenoid bone. 1) Anterior clinoid process 2) Temporal bone 3) Pterygoid hamulus 4) Pterygoid process

3) Clivus A) Clivus B) Sphenoid sinus C) Dorsum sellae D) Squamous portion of occipital bone

Identify structure *A* on this sagittal reformat of the occipital bone. 1) Sphenoid sinus 2) Dorsum sellae 3) Clivus 4) Squamous portion of occipital bone

1) Superior mesenteric vein A) Splenic vein B) Superior mesenteric vein C) Inferior mesenteric vein

Identify structure *B* in the hepatic system. 1) Superior mesenteric vein 2) Inferior mesenteric vein 3) Splenic vein

2) Renal vein A) Renal artery B) Renal vein C) Superior mesenteric artery

Identify structure *B* in this anterior view of the renal veins and arteries. 1) Superior mesenteric artery 2) Renal vein 3) Renal artery

2) Lateral mass A) Sacroiliac joint B) Lateral mass C) Sacral promontory D) Ilium

Identify structure *B* in this coronal CT slice of the sacroiliac joints. 1) Sacral promontory 2) Lateral mass 3) Sacroiliac joint 4) Ilium

2) Psoas muscle A) Iliacus muscle B) Psoas muscle C) Sacral promontory D) Right external iliac artery E) Ilium)

Identify structure *B* on this axial CT pelvis slice of internal/external iliac arteries and veins. 1) Iliacus muscle 2) Psoas muscle 3) Sacral promontory 4) Right external iliac artery 5) Ilium

1) Posterior arch A) Anterior arch B) Posterior arch C) Lateral mass D) Transverse foramen E) Odontoid process

Identify structure *B* on this axial CT slice of the C1. 1) Posterior arch 2) Lateral mass 3) Anterior arch 4) Odontoid process 5) Transverse foramen

3) Right subphrenic compartment A) Bare area of liver B) Right subphrenic compartment C) Left subphrenic compartment D) Spleen E) Stomach F) Azygous vein

Identify structure *B* on this axial CT slice of the abdomen. 1) Bare area of liver 2) Azygos vein 3) Right subphrenic compartment 4) Stomach 5) Left subphrenic compartment 6) Spleen

4) Portal vein A) Abdominal aorta B) Portal vein C) Inferior vena cava D) Body of pancreas E) Spleen

Identify structure *B* on this axial CT slice of the abdomen. 1) Body of pancreas 2) Spleen 3) Abdominal aorta 4) Portal vein 5) Inferior vena cava

4) Inferior vena cava A) Left psoas muscle B) Inferior vena cava C) Left ureter D) Internal oblique muscle

Identify structure *B* on this axial CT slice of the abdomen. 1) Left psoas muscle 2) Left ureter 3) Internal oblique muscle 4) Inferior vena cava

2) Linea alba A) Rectus abdominis muscle B) Linea alba C) Right ureter D) Quadratus lumborum muscle E) Right and left common iliac arteries

Identify structure *B* on this axial CT slice of the abdomen. 1) Rectus abdominis muscle 2) Linea alba 3) Right ureter 4) Quadratus lumborum muscle 5) Right and left common iliac arteries

3) Gerota's fascia A) Anterior pararenal space B) Gerota's fascia C) Renal pelvis D) Abdominal aorta E) Renal calyx

Identify structure *B* on this axial CT slice of the abdomen. 1) Renal pelvis 2) Anterior pararenal space 3) Gerota's fascia 4) Renal calyx 5) Abdominal aorta

3) Right crus A) Right adrenal gland B) Right crus C) Celiac axis D) Spleen

Identify structure *B* on this axial CT slice of the abdomen. 1) Spleen 2) Celiac axis 3) Right crus 4) Right adrenal gland

1) Vein of Galen A) Choroid plexus B) Vein of Galen C) Internal cerebral veins D) Thalamostriate vein E) Straight sinus

Identify structure *B* on this axial CT slice of the brain. 1) Vein of Galen 2) Thalamostriate vein 3) Choroid plexus 4) Internal cerebral veins

3) Insula A) Sylvian fissure B) Insula C) Quadrigeminal cistern D) Falx cerebri E) Colliculi F) Cerebellum

Identify structure *B* on this axial CT slice of the cerebral peduncles. 1) Colliculi 2) Sylvian fissure 3) Insula 4) Cerebellum

7) Esophagus A) Trachea B) Esophagus C) Left subclavian artery D) Left common carotid artery E) Brachiocephalic artery F) Right brachiocephalic vein G) Left brachiocephalic vein

Identify structure *B* on this axial CT slice of the chest. 1) Brachiocephalic artery 2) Left brachiocephalic vein 3) Left common carotid artery 4) Trachea 5) Left subclavian artery 6) Right brachiocephalic vein 7) Esophagus

6) Left mainstem bronchus A) Carina B) Left mainstem bronchus C) Sternum D) Secondary bronchi E) Descending aorta F) Ascending aorta

Identify structure *B* on this axial CT slice of the chest. 1) Descending aorta 2) Carina 3) Sternum 4) Secondary bronchi 5) Ascending aorta 6) Left mainstem bronchus

4) Azygos vein A) Superior vena cava B) Azygos vein C) Hemiazygos vein D) Pulmonary trunk E) Trapezius muscle F) Intercostal muscle

Identify structure *B* on this axial CT slice of the chest. 1) Trapezius muscle 2) Intercostal muscle 3) Hemiazygos vein 4) Azygos vein 5) Superior vena cava 6) Pulmonary trunk

1) Uterus A) Round ligament B) Uterus C) Rectouterine pouch D) Uterosacral ligament E) Rectum

Identify structure *B* on this axial CT slice of the female pelvis. 1) Uterus 2) Round ligament 3) Rectum 4) Uterosacral ligament 5) Rectouterine pouch

1) Palatine bones A) Palatine process of maxilla B) Palatine bones C) Pterygoid process

Identify structure *B* on this axial CT slice of the hard palate. 1) Palatine bones 2) Palatine process of maxilla 3) Pterygoid process

5) Left ventricle A) Left atrium B) Left ventricle C) Right atrium D) Right ventricle E) Ascending aorta

Identify structure *B* on this axial CT slice of the heart. 1) Left atrium 2) Right atrium 3) Right ventricle 4) Ascending aorta 5) Left ventricle

1) Annulus fibrosus A) Nucleus pulposus B) Annulus fibrosus C) Superior articular process D) Inferior articular process E) Apophyseal joint

Identify structure *B* on this axial CT slice of the lumbar spine with intervertebral disk. 1) Annulus fibrosus 2) Superior articular process 3) Apophyseal joint 4) Inferior articular process 5) Nucleus pulposus

6) Pedicle A) Body B) Pedicle C) Lamina D) Transverse process E) Posterior spinous process F)Vertebral foramen

Identify structure *B* on this axial CT slice of the lumbar vertebra. 1) Body 2) Transverse process 3) Vertebral foramen 4) Posterior spinous process 5) Lamina 6) Pedicle

4) Rectum A) Seminal vesicles B) Rectum C) Spermatic cord D) Bladder E) Acetabulum F) Ischial spine

Identify structure *B* on this axial CT slice of the male pelvis. 1) Bladder 2) Ischial spine 3) Seminal vesicles 4) Rectum 5) Spermatic cord

3) Temporalis muscle A) Masseter muscle B) Temporalis muscle C) Lateral pterygoid muscle

Identify structure *B* on this axial CT slice of the neck with muscles of mastication. 1) Masseter muscle 2) Lateral pterygoid muscle 3) Temporalis muscle

2) Internal jugular vein A) Epiglottis B) Internal jugular vein C) Vallecula D) Hyoid bone E) Sternocleidomastoid muscle

Identify structure *B* on this axial CT slice of the neck with oropharynx. 1) Vallecula 2) Internal jugular vein 3) Hyoid bone 4) Epiglottis 5) Sternocleidomastoid muscle

3) Sublingual gland A) Submandibular gland B) Sublingual gland C) Sternocleidomastoid muscle

Identify structure *B* on this axial CT slice of the neck. 1) Sternocleidomastoid muscle 2) Submandibular gland 3) Sublingual gland

2) Arytenoid cartilage A) Thyroid cartilage B) Arytenoid cartilage C) Rima glottis D) Vocal cord E) Platysma F) Anterior jugular vein

Identify structure *B* on this axial CT slice of the neck. 1) Thyroid cartilage 2) Arytenoid cartilage 3) Rima glottis 4) Platysma muscle 5) Anterior jugular vein 6) Vocal cord

1) Posterior chamber A) Lens B) Posterior chamber C) Optic canal D) Optic nerve E) Medial rectus muscle

Identify structure *B* on this axial CT slice of the orbit. 1) Posterior chamber 2) Optic canal 3) Lens 4) Optic nerve

2) Foramen spinosum A) Foramen magnum B) Foramen spinosum C) Foramen ovale D) Mandibular condyle E) Greater wing of sphenoid F) Clivus

Identify structure *B* on this axial CT slice of the sphenoid bone. 1) Foramen magnum 2) Foramen spinosum 3) Foramen ovale 4) Clivus

4) Jugular foramen A) Carotid canal B) Jugular foramen C) Sphenoid sinus D) Ethmoid sinus E) External auditory meatus F) Mastoid air cells

Identify structure *B* on this axial CT slice of the temporal bone. 1) External auditory meatus 2) Sphenoid sinus 3) Carotid canal 4) Jugular foramen

3) Temporalis muscle A) Mandibular condyle B) Temporalis muscle C) Masseter muscle D) Medial pterygoid muscle E) Lateral pterygoid muscle

Identify structure *B* on this axial CT slice of the temporomandibular joint. 1) Medial pterygoid muscle 2) Masseter muscle 3) Temporalis muscle 4) Mandibular condyle

1) Dorsal root A) Epidural fat B) Dorsal root C) Ventral root D) Conus medullaris E) Costovertebral articulation

Identify structure *B* on this axial CT slice of the thoracic vertebra. 1) Dorsal root 2) Ventral root 3) Epidural fat 4) Costovertebral articulation 5) Conus medullaris

4) Incus A) Malleus B) Incus C) Stapes D) Cochlea

Identify structure *B* on this axial CT slice of the unilateral petrous portion at the level of the external auditory meatus. 1) Cochlea 2) Malleus 3) Stapes 4) Incus

4) Posterior cerebral artery A) Anterior cerebral artery B) Posterior cerebral artery C) Anterior communicating artery D) Posterior communicating artery E) Middle cerebral artery

Identify structure *B* on this axial CTA of the Circle of Willis. 1) Posterior communicating artery 2) Anterior communicating artery 3) Anterior cerebral artery 4) Posterior cerebral artery

2) Maxillary sinus A) Inferior nasal concha B) Maxillary sinus C) Crista galli D) Ethmoid sinus E) Alveolar process of maxilla

Identify structure *B* on this coronal CT slice of the ethmoid bone. 1) Crista galli 2) Maxillary sinus 3) Sphenoid sinus 4) Ethmoid sinus

2) Superior orbital fissure A) Optic canal B) Superior orbital fissure C) Inferior orbital fissure D) Anterior clinoid process

Identify structure *B* on this coronal CT slice of the orbit. 1) Optic canal 2) Superior orbital fissure 3) Inferior orbital fissure 4) Anterior clinoid process

3) Ethmoid bulla A) Zygoma B) Ethmoid bulla C) Infundibulum D) Middle nasal meatus E) Uncinate process F) Orbital plate of frontal bone

Identify structure *B* on this coronal CT slice of the osteomeatal complex. 1) Middle nasal meatus 2) Zygoma 3) Ethmoid bulla 4) Infundibulum

2) Pectineal line A) Pelvic brim B) Pectineal line C) Arcuate line D) Symphysis pubis E) Iliac fossa

Identify structure *B* on this coronal CT slice of the pelvic brim. 1) Arcuate line 2) Pectineal line 3) Iliac fossa 4) Pelvic brim 5) Symphysis pubis

5) Sacral foramina A) Sacral promontory B) Sacral foramina C) Lateral mass (Ala) D) Sacroiliac joint E) Body of S1

Identify structure *B* on this coronal CT slice of the sacrum. 1) Lateral mass (Ala) 2) Sacroiliac joint 3) Body of S1 4) Sacral promontory 5) Sacral foramina

3) Pterygoid hamulus A) Pterygoid process B) Pterygoid hamulus C) Temporal bone D) Anterior clinoid process E) Sphenoid sinus F) Ramus of mandible

Identify structure *B* on this coronal CT slice of the sphenoid bone. 1) Anterior clinoid process 2) Temporal bone 3) Pterygoid hamulus 4) Pterygoid process

1) Sphenoid sinus A) Clivus B) Sphenoid sinus C) Dorsum sellae D) Squamous portion of occipital bone

Identify structure *B* on this sagittal reformat of the occipital bone. 1) Sphenoid sinus 2) Clivus 3) Squamous portion of occipital bone 4) Dorsum sellae

2) Inferior mesenteric vein A) Splenic vein B) Superior mesenteric vein C) Inferior mesenteric vein

Identify structure *C* in the hepatic system. 1) Superior mesenteric vein 2) Inferior mesenteric vein 3) Splenic vein

1) Superior mesenteric artery A) Renal artery B) Renal vein C) Superior mesenteric artery

Identify structure *C* in this anterior view of the renal veins and arteries. 1) Superior mesenteric artery 2) Renal vein 3) Renal artery

1) Sacral promontory A) Sacroiliac joint B) Lateral mass C) Sacral promontory D) Ilium

Identify structure *C* in this coronal CT slice of the sacroiliac joints. 1) Sacral promontory 2) Lateral mass 3) Sacroiliac joint 4) Ilium

3) Sacral promontory A) Iliacus muscle B) Psoas muscle C) Sacral promontory D) Right external iliac artery E) Ilium)

Identify structure *C* on this axial CT pelvis slice of internal/external iliac arteries and veins. 1) Iliacus muscle 2) Psoas muscle 3) Sacral promontory 4) Right external iliac artery 5) Ilium

2) Lateral mass A) Anterior arch B) Posterior arch C) Lateral mass D) Transverse foramen E) Odontoid process

Identify structure *C* on this axial CT slice of the C1. 1) Posterior arch 2) Lateral mass 3) Anterior arch 4) Odontoid process 5) Transverse foramen

5) Left subphrenic compartment A) Bare area of liver B) Right subphrenic compartment C) Left subphrenic compartment D) Spleen E) Stomach F) Azygous vein

Identify structure *C* on this axial CT slice of the abdomen. 1) Bare area of liver 2) Azygos vein 3) Right subphrenic compartment 4) Stomach 5) Left subphrenic compartment 6) Spleen

5) Inferior vena cava A) Abdominal aorta B) Portal vein C) Inferior vena cava D) Body of pancreas E) Spleen

Identify structure *C* on this axial CT slice of the abdomen. 1) Body of pancreas 2) Spleen 3) Abdominal aorta 4) Portal vein 5) Inferior vena cava

2) Left ureter A) Left psoas muscle B) Inferior vena cava C) Left ureter D) Internal oblique muscle

Identify structure *C* on this axial CT slice of the abdomen. 1) Left psoas muscle 2) Left ureter 3) Internal oblique muscle 4) Inferior vena cava

3) Right ureter A) Rectus abdominis muscle B) Linea alba C) Right ureter D) Quadratus lumborum muscle E) Right and left common iliac arteries

Identify structure *C* on this axial CT slice of the abdomen. 1) Rectus abdominis muscle 2) Linea alba 3) Right ureter 4) Quadratus lumborum muscle 5) Right and left common iliac arteries

1) Renal pelvis A) Anterior pararenal space B) Gerota's fascia C) Renal pelvis D) Abdominal aorta E) Renal calyx

Identify structure *C* on this axial CT slice of the abdomen. 1) Renal pelvis 2) Anterior pararenal space 3) Gerota's fascia 4) Renal calyx 5) Abdominal aorta

2) Celiac axis A) Right adrenal gland B) Right crus C) Celiac axis D) Spleen

Identify structure *C* on this axial CT slice of the abdomen. 1) Spleen 2) Celiac axis 3) Right crus 4) Right adrenal gland

4) Internal cerebral veins A) Choroid plexus B) Vein of Galen C) Internal cerebral veins D) Thalamostriate vein E) Straight sinus

Identify structure *C* on this axial CT slice of the brain. 1) Vein of Galen 2) Thalamostriate vein 3) Choroid plexus 4) Internal cerebral veins

1) Quadrigeminal cistern A) Sylvian fissure B) Insula C) Quadrigeminal cistern D) Falx cerebri E) Colliculi F) Cerebellum

Identify structure *C* on this axial CT slice of the cerebral peduncles. 1) Quadrigeminal cistern 2) Sylvian fissure 3) Insula 4) Falx cerebri

5) Left subclavian artery A) Trachea B) Esophagus C) Left subclavian artery D) Left common carotid artery E) Brachiocephalic artery F) Right brachiocephalic vein G) Left brachiocephalic vein

Identify structure *C* on this axial CT slice of the chest. 1) Brachiocephalic artery 2) Left brachiocephalic vein 3) Left common carotid artery 4) Trachea 5) Left subclavian artery 6) Right brachiocephalic vein 7) Esophagus

3) Sternum A) Carina B) Left mainstem bronchus C) Sternum D) Secondary bronchi E) Descending aorta F) Ascending aorta

Identify structure *C* on this axial CT slice of the chest. 1) Descending aorta 2) Carina 3) Sternum 4) Secondary bronchi 5) Ascending aorta 6) Left mainstem bronchus

3) Hemiazygos vein A) Superior vena cava B) Azygos vein C) Hemiazygos vein D) Pulmonary trunk E) Trapezius muscle F) Intercostal muscle

Identify structure *C* on this axial CT slice of the chest. 1) Trapezius muscle 2) Intercostal muscle 3) Hemiazygos vein 4) Azygos vein 5) Superior vena cava 6) Pulmonary trunk

5) Rectouterine pouch A) Round ligament B) Uterus C) Rectouterine pouch D) Uterosacral ligament E) Rectum

Identify structure *C* on this axial CT slice of the female pelvis. 1) Uterus 2) Round ligament 3) Rectum 4) Uterosacral ligament 5) Rectouterine pouch

3) Pterygoid process A) Palatine process of maxilla B) Palatine bones C) Pterygoid process

Identify structure *C* on this axial CT slice of the hard palate. 1) Palatine bones 2) Palatine process of maxilla 3) Pterygoid process

2) Right atrium A) Left atrium B) Left ventricle C) Right atrium D) Right ventricle E) Ascending aorta

Identify structure *C* on this axial CT slice of the heart. 1) Left atrium 2) Right atrium 3) Right ventricle 4) Ascending aorta 5) Left ventricle

2) Superior articular process A) Nucleus pulposus B) Annulus fibrosus C) Superior articular process D) Inferior articular process E) Apophyseal joint

Identify structure *C* on this axial CT slice of the lumbar spine with intervertebral disk. 1) Annulus fibrosus 2) Superior articular process 3) Apophyseal joint 4) Inferior articular process 5) Nucleus pulposus

5) Lamina A) Body B) Pedicle C) Lamina D) Transverse process E) Posterior spinous process F)Vertebral foramen

Identify structure *C* on this axial CT slice of the lumbar vertebra. 1) Body 2) Transverse process 3) Vertebral foramen 4) Posterior spinous process 5) Lamina 6) Pedicle

5) Spermatic cord A) Seminal vesicles B) Rectum C) Spermatic cord D) Bladder E) Acetabulum F) Ischial spine

Identify structure *C* on this axial CT slice of the male pelvis. 1) Bladder 2) Ischial spine 3) Seminal vesicles 4) Rectum 5) Spermatic cord 6) Acetabulum

2) Lateral pterygoid muscle A) Masseter muscle B) Temporalis muscle C) Lateral pterygoid muscle

Identify structure *C* on this axial CT slice of the neck with muscles of mastication. 1) Masseter muscle 2) Lateral pterygoid muscle 3) Temporalis muscle

1) Vallecula A) Epiglottis B) Internal jugular vein C) Vallecula D) Hyoid bone E) Sternocleidomastoid muscle

Identify structure *C* on this axial CT slice of the neck with oropharynx. 1) Vallecula 2) Internal jugular vein 3) Hyoid bone 4) Epiglottis 5) Sternocleidomastoid muscle

1) Sternocleidomastoid muscle A) Submandibular gland B) Sublingual gland C) Sternocleidomastoid muscle

Identify structure *C* on this axial CT slice of the neck. 1) Sternocleidomastoid muscle 2) Submandibular gland 3) Sublingual gland

3) Rima glottis A) Thyroid cartilage B) Arytenoid cartilage C) Rima glottis D) Vocal cord E) Platysma F) Anterior jugular vein

Identify structure *C* on this axial CT slice of the neck. 1) Thyroid cartilage 2) Arytenoid cartilage 3) Rima glottis 4) Platysma muscle 5) Anterior jugular vein 6) Vocal cord

2) Optic canal A) Lens B) Posterior chamber C) Optic canal D) Optic nerve E) Medial rectus muscle

Identify structure *C* on this axial CT slice of the orbit. 1) Medial rectus muscle 2) Optic canal 3) Optic canal 4) Optic nerve

3) Foramen ovale A) Foramen magnum B) Foramen spinosum C) Foramen ovale D) Mandibular condyle E) Greater wing of sphenoid F) Clivus

Identify structure *C* on this axial CT slice of the sphenoid bone. 1) Foramen magnum 2) Foramen spinosum 3) Foramen ovale 4) Mandibular condyle

2) Sphenoid sinus A) Carotid canal B) Jugular foramen C) Sphenoid sinus D) Ethmoid sinus E) External auditory meatus F) Mastoid air cells

Identify structure *C* on this axial CT slice of the temporal bone. 1) External auditory meatus 2) Sphenoid sinus 3) Mastoid air cells 4) Ethmoid sinus

2) Masseter muscle A) Mandibular condyle B) Temporalis muscle C) Masseter muscle D) Medial pterygoid muscle E) Lateral pterygoid muscle

Identify structure *C* on this axial CT slice of the temporomandibular joint. 1) Medial pterygoid muscle 2) Masseter muscle 3) Temporalis muscle 4) Lateral pterygoid muscle

2) Ventral root A) Epidural fat B) Dorsal root C) Ventral root D) Conus medullaris E) Costovertebral articulation

Identify structure *C* on this axial CT slice of the thoracic vertebra. 1) Dorsal root 2) Ventral root 3) Epidural fat 4) Costovertebral articulation 5) Conus medullaris

3) Stapes A) Malleus B) Incus C) Stapes D) Cochlea

Identify structure *C* on this axial CT slice of the unilateral petrous portion at the level of the external auditory meatus. 1) Cochlea 2) Malleus 3) Stapes 4) Incus

2) Anterior communicating artery A) Anterior cerebral artery B) Posterior cerebral artery C) Anterior communicating artery D) Posterior communicating artery E) Middle cerebral artery

Identify structure *C* on this axial CTA of the Circle of Willis. 1) Middle cerebral artery 2) Anterior communicating artery 3) Anterior cerebral artery 4) Posterior cerebral artery

1) Crista galli A) Inferior nasal concha B) Maxillary sinus C) Crista galli D) Ethmoid sinus E) Alveolar process of maxilla

Identify structure *C* on this coronal CT slice of the ethmoid bone. 1) Crista galli 2) Maxillary sinus 3) Inferior nasal concha 4) Alveolar process of maxilla

3) Inferior orbital fissure A) Optic canal B) Superior orbital fissure C) Inferior orbital fissure D) Anterior clinoid process

Identify structure *C* on this coronal CT slice of the orbit. 1) Optic canal 2) Superior orbital fissure 3) Inferior orbital fissure 4) Anterior clinoid process

4) Infundibulum A) Zygoma B) Ethmoid bulla C) Infundibulum D) Middle nasal meatus E) Uncinate process F) Orbital plate of frontal bone

Identify structure *C* on this coronal CT slice of the osteomeatal complex. 1) Middle nasal meatus 2) Zygoma 3) Ethmoid bulla 4) Infundibulum

1) Arcuate line A) Pelvic brim B) Pectineal line C) Arcuate line D) Symphysis pubis E) Iliac fossa

Identify structure *C* on this coronal CT slice of the pelvic brim. 1) Arcuate line 2) Pectineal line 3) Iliac fossa 4) Pelvic brim 5) Symphysis pubis

1) Lateral mass (Ala) A) Sacral promontory B) Sacral foramina C) Lateral mass (Ala) D) Sacroiliac joint E) Body of S1

Identify structure *C* on this coronal CT slice of the sacrum. 1) Lateral mass (Ala) 2) Sacroiliac joint 3) Body of S1 4) Sacral promontory 5) Sacral foramina

2) Temporal bone A) Pterygoid process B) Pterygoid hamulus C) Temporal bone D) Anterior clinoid process E) Sphenoid sinus F) Ramus of mandible

Identify structure *C* on this coronal CT slice of the sphenoid bone. 1) Anterior clinoid process 2) Temporal bone 3) Pterygoid hamulus 4) Pterygoid process

2) Dorsum sellae A) Clivus B) Sphenoid sinus C) Dorsum sellae D) Squamous portion of occipital bone

Identify structure *C* on this sagittal reformat of the occipital bone. 1) Squamous portion of occipital bone 2) Dorsum sellae 3) Clivus 4) Sphenoid sinus

4) Ilium A) Sacroiliac joint B) Lateral mass C) Sacral promontory D) Ilium

Identify structure *D* in this coronal CT slice of the sacroiliac joints. 1) Sacral promontory 2) Lateral mass 3) Sacroiliac joint 4) Ilium

4) Right external iliac artery A) Iliacus muscle B) Psoas muscle C) Sacral promontory D) Right external iliac artery E) Ilium)

Identify structure *D* on this axial CT pelvis slice of internal/external iliac arteries and veins. 1) Iliacus muscle 2) Psoas muscle 3) Sacral promontory 4) Right external iliac artery 5) Ilium

5) Transverse foramen A) Anterior arch B) Posterior arch C) Lateral mass D) Transverse foramen E) Odontoid process

Identify structure *D* on this axial CT slice of the C1. 1) Posterior arch 2) Lateral mass 3) Anterior arch 4) Odontoid process 5) Transverse foramen

1) Body of pancreas A) Abdominal aorta B) Portal vein C) Inferior vena cava D) Body of pancreas E) Spleen

Identify structure *D* on this axial CT slice of the abdomen. 1) Body of pancreas 2) Spleen 3) Abdominal aorta 4) Portal vein 5) Inferior vena cava

3) Internal oblique muscle A) Left psoas muscle B) Inferior vena cava C) Left ureter D) Internal oblique muscle

Identify structure *D* on this axial CT slice of the abdomen. 1) Left psoas muscle 2) Left ureter 3) Internal oblique muscle 4) Inferior vena cava

4) Quadratus lumborum muscle A) Rectus abdominis muscle B) Linea alba C) Right ureter D) Quadratus lumborum muscle E) Right and left common iliac arteries

Identify structure *D* on this axial CT slice of the abdomen. 1) Rectus abdominis muscle 2) Linea alba 3) Right ureter 4) Quadratus lumborum muscle 5) Right and left common iliac arteries

5) Abdominal aorta A) Anterior pararenal space B) Gerota's fascia C) Renal pelvis D) Abdominal aorta E) Renal calyx

Identify structure *D* on this axial CT slice of the abdomen. 1) Renal pelvis 2) Anterior pararenal space 3) Gerota's fascia 4) Renal calyx 5) Abdominal aorta

1) Spleen A) Right adrenal gland B) Right crus C) Celiac axis D) Spleen

Identify structure *D* on this axial CT slice of the abdomen. 1) Spleen 2) Celiac axis 3) Right crus 4) Right adrenal gland

6) Spleen A) Bare area of liver B) Right subphrenic compartment C) Left subphrenic compartment D) Spleen E) Stomach F) Azygous vein

Identify structure *D* on this axial CT slice of the abdomen. 1) Bare area of liver 2) Azygos vein 3) Right subphrenic compartment 4) Stomach 5) Left subphrenic compartment 6) Spleen

2) Thalamostriate vein A) Choroid plexus B) Vein of Galen C) Internal cerebral veins D) Thalamostriate vein E) Straight sinus

Identify structure *D* on this axial CT slice of the brain. 1) Vein of Galen 2) Thalamostriate vein 3) Straight sinus 4) Internal cerebral veins

2) Falx cerebri A) Sylvian fissure B) Insula C) Quadrigeminal cistern D) Falx cerebri E) Colliculi F) Cerebellum

Identify structure *D* on this axial CT slice of the cerebral peduncles. 1) Colliculi 2) Falx cerebri 3) Insula 4) Cerebellum

3) Left common carotid artery A) Trachea B) Esophagus C) Left subclavian artery D) Left common carotid artery E) Brachiocephalic artery F) Right brachiocephalic vein G) Left brachiocephalic vein

Identify structure *D* on this axial CT slice of the chest. 1) Brachiocephalic artery 2) Left brachiocephalic vein 3) Left common carotid artery 4) Trachea 5) Left subclavian artery 6) Right brachiocephalic vein 7) Esophagus

4) Secondary bronchi A) Carina B) Left mainstem bronchus C) Sternum D) Secondary bronchi E) Descending aorta F) Ascending aorta

Identify structure *D* on this axial CT slice of the chest. 1) Descending aorta 2) Carina 3) Sternum 4) Secondary bronchi 5) Ascending aorta 6) Left mainstem bronchus

6) Pulmonary trunk A) Superior vena cava B) Azygos vein C) Hemiazygos vein D) Pulmonary trunk E) Trapezius muscle F) Intercostal muscle

Identify structure *D* on this axial CT slice of the chest. 1) Trapezius muscle 2) Intercostal muscle 3) Hemiazygos vein 4) Azygos vein 5) Superior vena cava 6) Pulmonary trunk

4) Uterosacral ligament A) Round ligament B) Uterus C) Rectouterine pouch D) Uterosacral ligament E) Rectum

Identify structure *D* on this axial CT slice of the female pelvis. 1) Uterus 2) Round ligament 3) Rectum 4) Uterosacral ligament 5) Rectouterine pouch

3) Right ventricle A) Left atrium B) Left ventricle C) Right atrium D) Right ventricle E) Ascending aorta

Identify structure *D* on this axial CT slice of the heart. 1) Left atrium 2) Right atrium 3) Right ventricle 4) Ascending aorta 5) Left ventricle

4) Inferior articular process A) Nucleus pulposus B) Annulus fibrosus C) Superior articular process D) Inferior articular process E) Apophyseal joint

Identify structure *D* on this axial CT slice of the lumbar spine with intervertebral disk. 1) Annulus fibrosus 2) Superior articular process 3) Apophyseal joint 4) Inferior articular process 5) Nucleus pulposus

2) Transverse process A) Body B) Pedicle C) Lamina D) Transverse process E) Posterior spinous process F)Vertebral foramen

Identify structure *D* on this axial CT slice of the lumbar vertebra. 1) Body 2) Transverse process 3) Vertebral foramen 4) Posterior spinous process 5) Lamina 6) Pedicle

1) Bladder A) Seminal vesicles B) Rectum C) Spermatic cord D) Bladder E) Acetabulum F) Ischial spine

Identify structure *D* on this axial CT slice of the male pelvis. 1) Bladder 2) Ischial spine 3) Seminal vesicles 4) Rectum 5) Spermatic cord 6) Acetabulum

3) Hyoid bone A) Epiglottis B) Internal jugular vein C) Vallecula D) Hyoid bone E) Sternocleidomastoid muscle

Identify structure *D* on this axial CT slice of the neck with oropharynx. 1) Vallecula 2) Internal jugular vein 3) Hyoid bone 4) Epiglottis 5) Sternocleidomastoid muscle

6) Vocal cord A) Thyroid cartilage B) Arytenoid cartilage C) Rima glottis D) Vocal cord E) Platysma F) Anterior jugular vein

Identify structure *D* on this axial CT slice of the neck. 1) Thyroid cartilage 2) Arytenoid cartilage 3) Rima glottis 4) Platysma muscle 5) Anterior jugular vein 6) Vocal cord

4) Optic nerve A) Lens B) Posterior chamber C) Optic canal D) Optic nerve E) Medial rectus muscle

Identify structure *D* on this axial CT slice of the orbit. 1) Medial rectus muscle 2) Optic canal 3) Optic canal 4) Optic nerve

2) Mandibular condyle A) Foramen magnum B) Foramen spinosum C) Foramen ovale D) Mandibular condyle E) Greater wing of sphenoid F) Clivus

Identify structure *D* on this axial CT slice of the sphenoid bone. 1) Clivus 2) Mandibular condyle 3) Greater wing of sphenoid 4) Foramen magnum

3) Ethmoid sinus A) Carotid canal B) Jugular foramen C) Sphenoid sinus D) Ethmoid sinus E) External auditory meatus F) Mastoid air cells

Identify structure *D* on this axial CT slice of the temporal bone. 1) Carotid canal 2) Mastoid air cells 3) Ethmoid sinus 4) Sphenoid sinus

1) Medial pterygoid muscle A) Mandibular condyle B) Temporalis muscle C) Masseter muscle D) Medial pterygoid muscle E) Lateral pterygoid muscle

Identify structure *D* on this axial CT slice of the temporomandibular joint. 1) Medial pterygoid muscle 2) Masseter muscle 3) Temporalis muscle 4) Lateral pterygoid muscle

5) Conus medullaris A) Epidural fat B) Dorsal root C) Ventral root D) Conus medullaris E) Costovertebral articulation

Identify structure *D* on this axial CT slice of the thoracic vertebra. 1) Dorsal root 2) Ventral root 3) Epidural fat 4) Costovertebral articulation 5) Conus medullaris

1) Cochlea A) Malleus B) Incus C) Stapes D) Cochlea

Identify structure *D* on this axial CT slice of the unilateral petrous portion at the level of the external auditory meatus. 1) Cochlea 2) Malleus 3) Stapes 4) Incus

2) Posterior communicating artery A) Anterior cerebral artery B) Posterior cerebral artery C) Anterior communicating artery D) Posterior communicating artery E) Middle cerebral artery

Identify structure *D* on this axial CTA of the Circle of Willis. 1) Middle cerebral artery 2) Posterior communicating artery 3) Anterior cerebral artery 4) Posterior cerebral artery

4) Ethmoid sinus A) Inferior nasal concha B) Maxillary sinus C) Crista galli D) Ethmoid sinus E) Alveolar process of maxilla

Identify structure *D* on this coronal CT slice of the ethmoid bone. 1) Crista galli 2) Maxillary sinus 3) Sphenoid sinus 4) Ethmoid sinus

4) Anterior clinoid process A) Optic canal B) Superior orbital fissure C) Inferior orbital fissure D) Anterior clinoid process

Identify structure *D* on this coronal CT slice of the orbit. 1) Optic canal 2) Superior orbital fissure 3) Inferior orbital fissure 4) Anterior clinoid process

1) Middle nasal meatus A) Zygoma B) Ethmoid bulla C) Infundibulum D) Middle nasal meatus E) Uncinate process F) Orbital plate of frontal bone

Identify structure *D* on this coronal CT slice of the osteomeatal complex. 1) Middle nasal meatus 2) Zygoma 3) Ethmoid bulla 4) Infundibulum

5) Symphysis pubis A) Pelvic brim B) Pectineal line C) Arcuate line D) Symphysis pubis E) Iliac fossa

Identify structure *D* on this coronal CT slice of the pelvic brim. 1) Arcuate line 2) Pectineal line 3) Iliac fossa 4) Pelvic brim 5) Symphysis pubis

2) Sacroiliac joint A) Sacral promontory B) Sacral foramina C) Lateral mass (Ala) D) Sacroiliac joint E) Body of S1

Identify structure *D* on this coronal CT slice of the sacrum. 1) Lateral mass (Ala) 2) Sacroiliac joint 3) Body of S1 4) Sacral promontory 5) Sacral foramina

1) Anterior clinoid process A) Pterygoid process B) Pterygoid hamulus C) Temporal bone D) Anterior clinoid process E) Sphenoid sinus F) Ramus of mandible

Identify structure *D* on this coronal CT slice of the sphenoid bone. 1) Anterior clinoid process 2) Temporal bone 3) Pterygoid hamulus 4) Pterygoid process

4) Squamous portion of occipital bone A) Clivus B) Sphenoid sinus C) Dorsum sellae D) Squamous portion of occipital bone

Identify structure *D* on this sagittal reformat of the occipital bone. 1) Clivus 2) Dorsum sellae 3) Sphenoid sinus 4) Squamous portion of occipital bone

5) Ilium A) Iliacus muscle B) Psoas muscle C) Sacral promontory D) Right external iliac artery E) Ilium)

Identify structure *E* on this axial CT pelvis slice of internal/external iliac arteries and veins. 1) Iliacus muscle 2) Psoas muscle 3) Sacral promontory 4) Right external iliac artery 5) Ilium

4) Odontoid process A) Anterior arch B) Posterior arch C) Lateral mass D) Transverse foramen E) Odontoid process

Identify structure *E* on this axial CT slice of the C1. 1) Posterior arch 2) Lateral mass 3) Anterior arch 4) Odontoid process 5) Transverse foramen

4) Stomach A) Bare area of liver B) Right subphrenic compartment C) Left subphrenic compartment D) Spleen E) Stomach F) Azygous vein

Identify structure *E* on this axial CT slice of the abdomen. 1) Bare area of liver 2) Azygos vein 3) Right subphrenic compartment 4) Stomach 5) Left subphrenic compartment 6) Spleen

2) Spleen A) Abdominal aorta B) Portal vein C) Inferior vena cava D) Body of pancreas E) Spleen

Identify structure *E* on this axial CT slice of the abdomen. 1) Body of pancreas 2) Spleen 3) Abdominal aorta 4) Portal vein 5) Inferior vena cava

5) Right and left common iliac arteries A) Rectus abdominis muscle B) Linea alba C) Right ureter D) Quadratus lumborum muscle E) Right and left common iliac arteries

Identify structure *E* on this axial CT slice of the abdomen. 1) Rectus abdominis muscle 2) Linea alba 3) Right ureter 4) Quadratus lumborum muscle 5) Right and left common iliac arteries

4) Renal calyx A) Anterior pararenal space B) Gerota's fascia C) Renal pelvis D) Abdominal aorta E) Renal calyx

Identify structure *E* on this axial CT slice of the abdomen. 1) Renal pelvis 2) Anterior pararenal space 3) Gerota's fascia 4) Renal calyx 5) Abdominal aorta

4) Straight sinus A) Choroid plexus B) Vein of Galen C) Internal cerebral veins D) Thalamostriate vein E) Straight sinus

Identify structure *E* on this axial CT slice of the brain. 1) Vein of Galen 2) Thalamostriate vein 3) Choroid plexus 4) Straight sinus

1) Colliculi A) Sylvian fissure B) Insula C) Quadrigeminal cistern D) Falx cerebri E) Colliculi F) Cerebellum

Identify structure *E* on this axial CT slice of the cerebral peduncles. 1) Colliculi 2) Sylvian fissure 3) Insula 4) Cerebellum

1) Brachiocephalic artery A) Trachea B) Esophagus C) Left subclavian artery D) Left common carotid artery E) Brachiocephalic artery F) Right brachiocephalic vein G) Left brachiocephalic vein

Identify structure *E* on this axial CT slice of the chest. 1) Brachiocephalic artery 2) Left brachiocephalic vein 3) Left common carotid artery 4) Trachea 5) Left subclavian artery 6) Right brachiocephalic vein 7) Esophagus

1) Descending aorta A) Carina B) Left mainstem bronchus C) Sternum D) Secondary bronchi E) Descending aorta F) Ascending aorta

Identify structure *E* on this axial CT slice of the chest. 1) Descending aorta 2) Carina 3) Sternum 4) Secondary bronchi 5) Ascending aorta 6) Left mainstem bronchus

1) Trapezius muscle A) Superior vena cava B) Azygos vein C) Hemiazygos vein D) Pulmonary trunk E) Trapezius muscle F) Intercostal muscle

Identify structure *E* on this axial CT slice of the chest. 1) Trapezius muscle 2) Intercostal muscle 3) Hemiazygos vein 4) Azygos vein 5) Superior vena cava 6) Pulmonary trunk

3) Rectum A) Round ligament B) Uterus C) Rectouterine pouch D) Uterosacral ligament E) Rectum

Identify structure *E* on this axial CT slice of the female pelvis. 1) Uterus 2) Round ligament 3) Rectum 4) Uterosacral ligament 5) Rectouterine pouch

4) Ascending aorta A) Left atrium B) Left ventricle C) Right atrium D) Right ventricle E) Ascending aorta

Identify structure *E* on this axial CT slice of the heart. 1) Left atrium 2) Right atrium 3) Right ventricle 4) Ascending aorta 5) Left ventricle

3) Apophyseal joint A) Nucleus pulposus B) Annulus fibrosus C) Superior articular process D) Inferior articular process E) Apophyseal joint

Identify structure *E* on this axial CT slice of the lumbar spine with intervertebral disk. 1) Annulus fibrosus 2) Superior articular process 3) Apophyseal joint 4) Inferior articular process 5) Nucleus pulposus

4) Posterior spinous process A) Body B) Pedicle C) Lamina D) Transverse process E) Posterior spinous process F)Vertebral foramen

Identify structure *E* on this axial CT slice of the lumbar vertebra. 1) Body 2) Transverse process 3) Vertebral foramen 4) Posterior spinous process 5) Lamina 6) Pedicle

6) Acetabulum A) Seminal vesicles B) Rectum C) Spermatic cord D) Bladder E) Acetabulum F) Ischial spine

Identify structure *E* on this axial CT slice of the male pelvis. 1) Bladder 2) Ischial spine 3) Seminal vesicles 4) Rectum 5) Spermatic cord 6) Acetabulum

5) Sternocleidomastoid muscle A) Epiglottis B) Internal jugular vein C) Vallecula D) Hyoid bone E) Sternocleidomastoid muscle

Identify structure *E* on this axial CT slice of the neck with oropharynx. 1) Vallecula 2) Internal jugular vein 3) Hyoid bone 4) Epiglottis 5) Sternocleidomastoid muscle

4) Platysma muscle A) Thyroid cartilage B) Arytenoid cartilage C) Rima glottis D) Vocal cord E) Platysma F) Anterior jugular vein

Identify structure *E* on this axial CT slice of the neck. 1) Thyroid cartilage 2) Arytenoid cartilage 3) Rima glottis 4) Platysma muscle 5) Anterior jugular vein 6) Vocal cord

1) Medial rectus muscle A) Lens B) Posterior chamber C) Optic canal D) Optic nerve E) Medial rectus muscle

Identify structure *E* on this axial CT slice of the orbit. 1) Medial rectus muscle 2) Optic canal 3) Optic canal 4) Optic nerve

3) Greater wing of sphenoid A) Foramen magnum B) Foramen spinosum C) Foramen ovale D) Mandibular condyle E) Greater wing of sphenoid F) Clivus

Identify structure *E* on this axial CT slice of the sphenoid bone. 1) Clivus 2) Mandibular condyle 3) Greater wing of sphenoid 4) Foramen magnum

2) External auditory meatus A) Carotid canal B) Jugular foramen C) Sphenoid sinus D) Ethmoid sinus E) External auditory meatus F) Mastoid air cells

Identify structure *E* on this axial CT slice of the temporal bone. 1) Carotid canal 2) External auditory meatus 3) Mastoid air cells 4) Jugular foramen

4) Lateral pterygoid muscle A) Mandibular condyle B) Temporalis muscle C) Masseter muscle D) Medial pterygoid muscle E) Lateral pterygoid muscle

Identify structure *E* on this axial CT slice of the temporomandibular joint. 1) Medial pterygoid muscle 2) Masseter muscle 3) Temporalis muscle 4) Lateral pterygoid muscle

4) Costovertebral articulation A) Epidural fat B) Dorsal root C) Ventral root D) Conus medullaris E) Costovertebral articulation

Identify structure *E* on this axial CT slice of the thoracic vertebra. 1) Dorsal root 2) Ventral root 3) Epidural fat 4) Costovertebral articulation 5) Conus medullaris

1) Middle cerebral artery A) Anterior cerebral artery B) Posterior cerebral artery C) Anterior communicating artery D) Posterior communicating artery E) Middle cerebral artery

Identify structure *E* on this axial CTA of the Circle of Willis. 1) Middle cerebral artery 2) Anterior communicating artery 3) Anterior cerebral artery 4) Posterior cerebral artery

4) Alveolar process of maxilla A) Inferior nasal concha B) Maxillary sinus C) Crista galli D) Ethmoid sinus E) Alveolar process of maxilla

Identify structure *E* on this coronal CT slice of the ethmoid bone. 1) Crista galli 2) Maxillary sinus 3) Inferior nasal concha 4) Alveolar process of maxilla

1) Uncinate process A) Zygoma B) Ethmoid bulla C) Infundibulum D) Middle nasal meatus E) Uncinate process F) Orbital plate of frontal bone

Identify structure *E* on this coronal CT slice of the osteomeatal complex. 1) Uncinate process 2) Zygoma 3) Orbital plate of frontal bone 4) Infundibulum

3) Iliac fossa A) Pelvic brim B) Pectineal line C) Arcuate line D) Symphysis pubis E) Iliac fossa

Identify structure *E* on this coronal CT slice of the pelvic brim. 1) Arcuate line 2) Pectineal line 3) Iliac fossa 4) Pelvic brim 5) Symphysis pubis

3) Body of S1 A) Sacral promontory B) Sacral foramina C) Lateral mass (Ala) D) Sacroiliac joint E) Body of S1

Identify structure *E* on this coronal CT slice of the sacrum. 1) Lateral mass (Ala) 2) Sacroiliac joint 3) Body of S1 4) Sacral promontory 5) Sacral foramina

3) Sphenoid sinus A) Pterygoid process B) Pterygoid hamulus C) Temporal bone D) Anterior clinoid process E) Sphenoid sinus F) Ramus of mandible

Identify structure *E* on this coronal CT slice of the sphenoid bone. 1) Anterior clinoid process 2) Ramus of mandible 3) Sphenoid sinus 4) Ethmoid sinus

2) Azygos vein A) Bare area of liver B) Right subphrenic compartment C) Left subphrenic compartment D) Spleen E) Stomach F) Azygous vein

Identify structure *F* on this axial CT slice of the abdomen. 1) Bare area of liver 2) Azygos vein 3) Right subphrenic compartment 4) Stomach 5) Left subphrenic compartment 6) Spleen

4) Cerebellum A) Sylvian fissure B) Insula C) Quadrigeminal cistern D) Falx cerebri E) Colliculi F) Cerebellum

Identify structure *F* on this axial CT slice of the cerebral peduncles. 1) Colliculi 2) Sylvian fissure 3) Insula 4) Cerebellum

6) Right brachiocephalic vein A) Trachea B) Esophagus C) Left subclavian artery D) Left common carotid artery E) Brachiocephalic artery F) Right brachiocephalic vein G) Left brachiocephalic vein

Identify structure *F* on this axial CT slice of the chest. 1) Brachiocephalic artery 2) Left brachiocephalic vein 3) Left common carotid artery 4) Trachea 5) Left subclavian artery 6) Right brachiocephalic vein 7) Esophagus

5) Ascending aorta A) Carina B) Left mainstem bronchus C) Sternum D) Secondary bronchi E) Descending aorta F) Ascending aorta

Identify structure *F* on this axial CT slice of the chest. 1) Descending aorta 2) Carina 3) Sternum 4) Secondary bronchi 5) Ascending aorta 6) Left mainstem bronchus

2) Intercostal muscle A) Superior vena cava B) Azygos vein C) Hemiazygos vein D) Pulmonary trunk E) Trapezius muscle F) Intercostal muscle

Identify structure *F* on this axial CT slice of the chest. 1) Trapezius muscle 2) Intercostal muscle 3) Hemiazygos vein 4) Azygos vein 5) Superior vena cava 6) Pulmonary trunk

3) Vertebral foramen A) Body B) Pedicle C) Lamina D) Transverse process E) Posterior spinous process F)Vertebral foramen

Identify structure *F* on this axial CT slice of the lumbar vertebra. 1) Body 2) Transverse process 3) Vertebral foramen 4) Posterior spinous process 5) Lamina 6) Pedicle

2) Ischial spine A) Seminal vesicles B) Rectum C) Spermatic cord D) Bladder E) Acetabulum F) Ischial spine

Identify structure *F* on this axial CT slice of the male pelvis. 1) Bladder 2) Ischial spine 3) Seminal vesicles 4) Rectum 5) Spermatic cord 6) Acetabulum

5) Anterior jugular vein A) Thyroid cartilage B) Arytenoid cartilage C) Rima glottis D) Vocal cord E) Platysma F) Anterior jugular vein

Identify structure *F* on this axial CT slice of the neck. 1) Thyroid cartilage 2) Arytenoid cartilage 3) Rima glottis 4) Platysma muscle 5) Anterior jugular vein 6) Vocal cord

1) Clivus A) Foramen magnum B) Foramen spinosum C) Foramen ovale D) Mandibular condyle E) Greater wing of sphenoid F) Clivus

Identify structure *F* on this axial CT slice of the sphenoid bone. 1) Clivus 2) Mandibular condyle 3) Greater wing of sphenoid 4) Foramen magnum

1) Mastoid air cells A) Carotid canal B) Jugular foramen C) Sphenoid sinus D) Ethmoid sinus E) External auditory meatus F) Mastoid air cells

Identify structure *F* on this axial CT slice of the temporal bone. 1) Mastoid air cells 2) External auditory meatus 3) Sphenoid sinus 4) Jugular foramen

3) Orbital plate of frontal bone A) Zygoma B) Ethmoid bulla C) Infundibulum D) Middle nasal meatus E) Uncinate process F) Orbital plate of frontal bone

Identify structure *F* on this coronal CT slice of the osteomeatal complex. 1) Uncinate process 2) Zygoma 3) Orbital plate of frontal bone 4) Infundibulum

d) Ramus of mandible A) Pterygoid process B) Pterygoid hamulus C) Temporal bone D) Anterior clinoid process E) Sphenoid sinus F) Ramus of mandible

Identify structure *F* on this coronal CT slice of the sphenoid bone. 1) Mandibular condyle 2) Sphenoid sinus 3) Anterior clinoid process 4) Ramus of mandible

2) Left brachiocephalic vein A) Trachea B) Esophagus C) Left subclavian artery D) Left common carotid artery E) Brachiocephalic artery F) Right brachiocephalic vein G) Left brachiocephalic vein

Identify structure *G* on this axial CT slice of the chest. 1) Brachiocephalic artery 2) Left brachiocephalic vein 3) Left common carotid artery 4) Trachea 5) Left subclavian artery 6) Right brachiocephalic vein 7) Esophagus

b) Brachiocephalic artery 1 Brachiocephalic artery; 2 Left common carotid artery; 3 Left subclavian artery; 4 Pulmonary arteries; 5 Pulmonary veins; 6 Left atrium; 7 Bicuspid valve; 8 Aortic semilunar valve; 9 Left ventricle; 10 Interventricular septum; 11 Right ventricle; 12 Inferior vena cava; 13 Tricuspid valve; 14 Right atrium; 15 Pulmonary semilunar valve; 16 Superior vena cava; 17 Aorta

Identify structure 1 on this heart diagram. a) Left common carotid artery b) Brachiocephalic artery c) Left subclavian artery d) Pulmonary arteries e) Pulmonary veins

b) Interventricular septum 1 Brachiocephalic artery; 2 Left common carotid artery; 3 Left subclavian artery; 4 Pulmonary arteries; 5 Pulmonary veins; 6 Left atrium; 7 Bicuspid valve; 8 Aortic semilunar valve; 9 Left ventricle; 10 Interventricular septum; 11 Right ventricle; 12 Inferior vena cava; 13 Tricuspid valve; 14 Right atrium; 15 Pulmonary semilunar valve; 16 Superior vena cava; 17 Aorta

Identify structure 10 on this heart diagram. a) Left ventricle b) Interventricular septum c) Left atrium d) Aortic semilunar valve e) Bicuspid valve

a) Right ventricle 1 Brachiocephalic artery; 2 Left common carotid artery; 3 Left subclavian artery; 4 Pulmonary arteries; 5 Pulmonary veins; 6 Left atrium; 7 Bicuspid valve; 8 Aortic semilunar valve; 9 Left ventricle; 10 Interventricular septum; 11 Right ventricle; 12 Inferior vena cava; 13 Tricuspid valve; 14 Right atrium; 15 Pulmonary semilunar valve; 16 Superior vena cava; 17 Aorta

Identify structure 11 on this heart diagram. a) Right ventricle b) Tricuspid valve c) Pulmonary semilunar valve d) Inferior vena cava e) Right atrium

d) Inferior vena cava 1 Brachiocephalic artery; 2 Left common carotid artery; 3 Left subclavian artery; 4 Pulmonary arteries; 5 Pulmonary veins; 6 Left atrium; 7 Bicuspid valve; 8 Aortic semilunar valve; 9 Left ventricle; 10 Interventricular septum; 11 Right ventricle; 12 Inferior vena cava; 13 Tricuspid valve; 14 Right atrium; 15 Pulmonary semilunar valve; 16 Superior vena cava; 17 Aorta

Identify structure 12 on this heart diagram. a) Right ventricle b) Tricuspid valve c) Pulmonary semilunar valve d) Inferior vena cava e) Right atrium

b) Tricuspid valve 1 Brachiocephalic artery; 2 Left common carotid artery; 3 Left subclavian artery; 4 Pulmonary arteries; 5 Pulmonary veins; 6 Left atrium; 7 Bicuspid valve; 8 Aortic semilunar valve; 9 Left ventricle; 10 Interventricular septum; 11 Right ventricle; 12 Inferior vena cava; 13 Tricuspid valve; 14 Right atrium; 15 Pulmonary semilunar valve; 16 Superior vena cava; 17 Aorta

Identify structure 13 on this heart diagram. a) Right ventricle b) Tricuspid valve c) Pulmonary semilunar valve d) Inferior vena cava e) Right atrium

e) Right atrium 1 Brachiocephalic artery; 2 Left common carotid artery; 3 Left subclavian artery; 4 Pulmonary arteries; 5 Pulmonary veins; 6 Left atrium; 7 Bicuspid valve; 8 Aortic semilunar valve; 9 Left ventricle; 10 Interventricular septum; 11 Right ventricle; 12 Inferior vena cava; 13 Tricuspid valve; 14 Right atrium; 15 Pulmonary semilunar valve; 16 Superior vena cava; 17 Aorta

Identify structure 14 on this heart diagram. a) Right ventricle b) Tricuspid valve c) Pulmonary semilunar valve d) Inferior vena cava e) Right atrium

c) Pulmonary semilunar valve 1 Brachiocephalic artery; 2 Left common carotid artery; 3 Left subclavian artery; 4 Pulmonary arteries; 5 Pulmonary veins; 6 Left atrium; 7 Bicuspid valve; 8 Aortic semilunar valve; 9 Left ventricle; 10 Interventricular septum; 11 Right ventricle; 12 Inferior vena cava; 13 Tricuspid valve; 14 Right atrium; 15 Pulmonary semilunar valve; 16 Superior vena cava; 17 Aorta

Identify structure 15 on this heart diagram. a) Right ventricle b) Tricuspid valve c) Pulmonary semilunar valve d) Inferior vena cava e) Right atrium

c) Superior vena cava 1 Brachiocephalic artery; 2 Left common carotid artery; 3 Left subclavian artery; 4 Pulmonary arteries; 5 Pulmonary veins; 6 Left atrium; 7 Bicuspid valve; 8 Aortic semilunar valve; 9 Left ventricle; 10 Interventricular septum; 11 Right ventricle; 12 Inferior vena cava; 13 Tricuspid valve; 14 Right atrium; 15 Pulmonary semilunar valve; 16 Superior vena cava; 17 Aorta

Identify structure 16 on this heart diagram. a) Right ventricle b) Aorta c) Superior vena cava d) Inferior vena cava e) Right atrium

b) Aorta 1 Brachiocephalic artery; 2 Left common carotid artery; 3 Left subclavian artery; 4 Pulmonary arteries; 5 Pulmonary veins; 6 Left atrium; 7 Bicuspid valve; 8 Aortic semilunar valve; 9 Left ventricle; 10 Interventricular septum; 11 Right ventricle; 12 Inferior vena cava; 13 Tricuspid valve; 14 Right atrium; 15 Pulmonary semilunar valve; 16 Superior vena cava; 17 Aorta

Identify structure 17 on this heart diagram. a) Right ventricle b) Aorta c) Superior vena cava d) Inferior vena cava e) Right atrium

a) Left common carotid artery 1 Brachiocephalic artery; 2 Left common carotid artery; 3 Left subclavian artery; 4 Pulmonary arteries; 5 Pulmonary veins; 6 Left atrium; 7 Bicuspid valve; 8 Aortic semilunar valve; 9 Left ventricle; 10 Interventricular septum; 11 Right ventricle; 12 Inferior vena cava; 13 Tricuspid valve; 14 Right atrium; 15 Pulmonary semilunar valve; 16 Superior vena cava; 17 Aorta

Identify structure 2 on this heart diagram. a) Left common carotid artery b) Brachiocephalic artery c) Left subclavian artery d) Pulmonary arteries e) Pulmonary veins

c) Left subclavian artery 1 Brachiocephalic artery; 2 Left common carotid artery; 3 Left subclavian artery; 4 Pulmonary arteries; 5 Pulmonary veins; 6 Left atrium; 7 Bicuspid valve; 8 Aortic semilunar valve; 9 Left ventricle; 10 Interventricular septum; 11 Right ventricle; 12 Inferior vena cava; 13 Tricuspid valve; 14 Right atrium; 15 Pulmonary semilunar valve; 16 Superior vena cava; 17 Aorta

Identify structure 3 on this heart diagram. a) Left common carotid artery b) Brachiocephalic artery c) Left subclavian artery d) Pulmonary arteries e) Pulmonary veins

d) Pulmonary arteries 1 Brachiocephalic artery; 2 Left common carotid artery; 3 Left subclavian artery; 4 Pulmonary arteries; 5 Pulmonary veins; 6 Left atrium; 7 Bicuspid valve; 8 Aortic semilunar valve; 9 Left ventricle; 10 Interventricular septum; 11 Right ventricle; 12 Inferior vena cava; 13 Tricuspid valve; 14 Right atrium; 15 Pulmonary semilunar valve; 16 Superior vena cava; 17 Aorta

Identify structure 4 on this heart diagram. a) Left common carotid artery b) Brachiocephalic artery c) Left subclavian artery d) Pulmonary arteries e) Pulmonary veins

e) Pulmonary veins 1 Brachiocephalic artery; 2 Left common carotid artery; 3 Left subclavian artery; 4 Pulmonary arteries; 5 Pulmonary veins; 6 Left atrium; 7 Bicuspid valve; 8 Aortic semilunar valve; 9 Left ventricle; 10 Interventricular septum; 11 Right ventricle; 12 Inferior vena cava; 13 Tricuspid valve; 14 Right atrium; 15 Pulmonary semilunar valve; 16 Superior vena cava; 17 Aorta

Identify structure 5 on this heart diagram. a) Left common carotid artery b) Brachiocephalic artery c) Left subclavian artery d) Pulmonary arteries e) Pulmonary veins

c) Left atrium 1 Brachiocephalic artery; 2 Left common carotid artery; 3 Left subclavian artery; 4 Pulmonary arteries; 5 Pulmonary veins; 6 Left atrium; 7 Bicuspid valve; 8 Aortic semilunar valve; 9 Left ventricle; 10 Interventricular septum; 11 Right ventricle; 12 Inferior vena cava; 13 Tricuspid valve; 14 Right atrium; 15 Pulmonary semilunar valve; 16 Superior vena cava; 17 Aorta

Identify structure 6 on this heart diagram. a) Left ventricle b) Interventricular septum c) Left atrium d) Aortic semilunar valve e) Bicuspid valve

e) Bicuspid valve 1 Brachiocephalic artery; 2 Left common carotid artery; 3 Left subclavian artery; 4 Pulmonary arteries; 5 Pulmonary veins; 6 Left atrium; 7 Bicuspid valve; 8 Aortic semilunar valve; 9 Left ventricle; 10 Interventricular septum; 11 Right ventricle; 12 Inferior vena cava; 13 Tricuspid valve; 14 Right atrium; 15 Pulmonary semilunar valve; 16 Superior vena cava; 17 Aorta

Identify structure 7 on this heart diagram. a) Left ventricle b) Interventricular septum c) Left atrium d) Aortic semilunar valve e) Bicuspid valve

d) Aortic semilunar valve 1 Brachiocephalic artery; 2 Left common carotid artery; 3 Left subclavian artery; 4 Pulmonary arteries; 5 Pulmonary veins; 6 Left atrium; 7 Bicuspid valve; 8 Aortic semilunar valve; 9 Left ventricle; 10 Interventricular septum; 11 Right ventricle; 12 Inferior vena cava; 13 Tricuspid valve; 14 Right atrium; 15 Pulmonary semilunar valve; 16 Superior vena cava; 17 Aorta

Identify structure 8 on this heart diagram. a) Left ventricle b) Interventricular septum c) Left atrium d) Aortic semilunar valve e) Bicuspid valve

a) Left ventricle 1 Brachiocephalic artery; 2 Left common carotid artery; 3 Left subclavian artery; 4 Pulmonary arteries; 5 Pulmonary veins; 6 Left atrium; 7 Bicuspid valve; 8 Aortic semilunar valve; 9 Left ventricle; 10 Interventricular septum; 11 Right ventricle; 12 Inferior vena cava; 13 Tricuspid valve; 14 Right atrium; 15 Pulmonary semilunar valve; 16 Superior vena cava; 17 Aorta

Identify structure 9 on this heart diagram. a) Left ventricle b) Interventricular septum c) Left atrium d) Aortic semilunar valve e) Bicuspid valve

C. 10 to 11.2 cGy In general, radiation that spills over from adjacent slices (or "tails") will contribute approximately 25% to 40% additional dose, as compared with one dose from a single slice. *Therefore, if a single slice delivered 8 cGy, then the entire dose from a contiguous slice study would range from 10 cGy (8 cGy + 2 cGy) to 11.2 (8 cGy + 3.2 cGy).*

If a single slice of the abddomen delivered a radiation dose of 8 cGy, the entire examination dose from 40 contiguous slices could be estimated as a. 0.2 to 0.8 cGy b. 8 cGy c. 10 to 11.2 cGy d. 320 cGy

C. increased pitch With a single-slice CT (SSCT) system, pitch equals the ratio of the table speed with the section thickness. As the detector pitch is increased, the patient translates through the gantry at a greater rate relative to each rotation of the x-ray tube, resulting in the acquisition of a given volume of anatomy with fewer tube-detector rotations and shorter scan times. Increases in pitch therefore allow for reduced patient radiation dose while covering the necessary anatomic area.

If all other technical factors remain constant, which of the following would serve to decrease patient radiation dose during a helical single-slice CT (SSCT) examination? a. decreased scan field of view (SFOV) b. decreased filtration c. increased pitch d. increased matrix size

Decrease kVp by 15%

If it is necessary to reduce radiographic density by half, and it is impossible to do so by changing mAs the radiographer may:

Decrease

If kVp is increased, what happens to radiographic contrast?

Decrease

If mAs is increased by four times, what is the effect on radiographic contrast?

D. cross-field uniformity Refers to the ability of the scanner to yield the same CT numbers in a homogeneous object (i.e. water phantom) regardless of the location of the region of interest.

If measurements taken around the perimeter of a water phantom are different from those taken at the center of the phantom, then there is a problem with the system's: A) linearity B) slice-thickness accuracy C) photon absorption D) cross-field uniformity

A) Increase of mAs ***Photon flux is primarily dependent on the combination of kVp, mAs, and beam filtration with mAs being the most straightforward technical factor to change when contrast resolution is required. mAs is directly proportional to the number of photons produced and the patient dose received.***

If one wants to increase the contrast resolution of an image, which of the following would be the best factor to change? A) Increase of mAs B) Increase of kVp C) Increase if beam filtration D) Decrease of kV

A) Increase of mAs ***Photon flux is primarily dependent on the combination of kVp, mAs, and beam filtration with mAs being the most straightforward technical factor to change when contrast resolution is required. mAs is directly proportional to the number of photons produced and the patient dose received.***

If one wants to increase the contrast resolution of an image, which of the following would be the best factor to change? A) Increase of mAs B) Increase of kVp C) Increase of beam filtration D) Decrease of kV

A) 43 HU

If the u for soft tissue is 0.215 and the u for water is 0.206, what is the correlated HU for soft tissue? A) 43 HU B) 2,000 HU C) -43 HU D) 1,000 HU

Decrease

If there is an increase in added filtration, what happens to contrast?

C. creating multiplanar reformation Multiplanar reformation software stack up images, and then slice the stacked whole in a plane specified by the operator. These can be created from image data, providing the images possess the same display field size, center, and gantry tilt and the slices are contiguous. Changes to the display field size, the reconstruction algorithm, or the slice increment must be done using raw data.

Image data can be used for... A) changing the display field size B) changing the reconstruction algorithm C) creating multiplanar reformation D) creating overlapping slices from helical data

Undesirable fluctuation in brightness

Image noise may be described as:

A. occurs with digital technology and refers to the fact that the relationship between dose and image quality is less direct than in film-screen radiography With digital technology, the image is uncoupled from the dose, so even when an mA or kVp setting that is too high is used, a good image results. Many technologists currently working in the field were trained before the routine use of digital radiography and may be unfamiliar with this characteristic.

In CT, the uncoupling effect: A) occurs with digital technology and refers to the fact that the relationship between dose and image quality is less direct than in film-screen radiography B) occurs only in CT and refers to the process of changing pitch C) refers to the process in conventional radiography in which the appropriate dose creates the proper film darkening; that is, too high a dose will create an image that is too dark, whereas too low a dose will result in an image that is too light D) refers to the lack of relationship, in CT, between patient size and radiation dose

D. total collimation Total collimation equals the combined thickness of all the sections that are simultaneously acquired with each gantry rotation. Example: A given MDCT system has an array of 64 detectors, each 0.625 mm wide. If the beam is collimated to expose the entire array, the total collimation for the acquisition is equal to 0.625 mm x 64, or 40 mm.

In MDCT, the combined thicknesses of all of the sections simultaneously acquired with each gantry rotation are called the: a. detector configuration b. beam pitch c. detector pitch d. total collimation

B. it produces high voltage and transmit it to the x-ray tube The generator produces high voltage and transmits it to the x-ray tube. This high voltage propels the electrons from the x-ray tube filament to the anode. High amperage is not necessary for the production of x-rays.

In a CT system, what is the function of the generator? A) it produces and electric current of very high amperage and transmits it to the filament B) it produces high voltage and transmits it to the x-ray tube C) it samples the detectors thousands of times per second D) it dissipates the heat built up during x-ray production

A. detector collimation With MDCT, detector collimation determines the width of the reconstructed section. By electronically adjusting the detector dimension, the operator can control the width of the x-ray beam contributing to a reconstructed section. Beam collimation no longer directly controls section width.

In a MDCT system, which of the following technical parameters determines the reconstructed section width? a. detector collimation b. number of data channels c. beam collimation d. detector pitch

D. 15-second total scan acquisition, 1.2 pitch The distance covered in an MDCT helical scan sequence can be calculated by using the formula: *Pitch x total acquisition time x 1/rotation time x (slice thickness x slices per rotation)* A) 1.0 x 10 x 1 x (5mm x 4) = 200mm B) 1.2 x 10 x 1 x (5mm x 4) = 240mm C) 1.0 x 15 x 1 x (5mm x 4) = 300mm D) 1.2 x 15 x 1 x (5mm x 4) = 360mm

In a preliminary scout view of a patient, the abdomen is determined to be 360mm in length. In this four-slice CT system, the gantry makes a 360-degree rotation each second. The protocol at this institution calls for a 5-mm slice thickness. To cover the entire abdominal area in a single helical scan, which set of parameters could be selected? A) 10-second total acquisition time, 1 pitch B) 10-second total acquisition time, 1.2 pitch C) 15-second total acquisition time, 1 pitch D) 15-second total acquisition time, 1.2 pitch

C. 2 and 3 only -- section increment and reconstruction algorithm Single-slice helical geometry allows for volumetric data acquisition. The reconstruction of CT sections at any point along the scanned volume is possible. The section width must be the same, however, and is controlled by the collimation (slice thickness) chosen before data acquisition. The advancement of multislice CT (MDCT) allows for retrospective reconstruction of sections at varying thickness.

In a single-slice CT (SSCT) system, which of the following technical parameters may be adjusted retrospectively? 1. section width 2. section increment 3. reconstruction algorithm a. 2 only b. 1 and 2 only c. 2 and 3 only d. 1, 2 and 3

C. a second scan series is obtained in a delayed venous phase (approximately 180 seconds after IV contrast injection) from the iliac crest through the knees

In addition to imaging the pulmonary arterial system for patients with suspected PE, CT venography is sometimes performed to assess for venous thrombosis within the pelvis and lower extremities. How is this accomplished? A) a single helical scan is obtained in the craniocaudal direction extending from the lung apices to the knees while contrast is in the bolus phase (20 - 30 seconds after IV contrast injection) B) a second scan series is obtained in the nonequilibrium phase (approximately 45 seconds after IV contrast injection) from the iliac crest through the knees C) a second scan series is obtained in a delayed venous phase (approximately 180 seconds after IV contrast injection) from the iliac crest through the knees D) a second scan series is obtained after most of the contrast has washed out of the venous structures (8 - 12 minutes after IV contrast injection) from the iliac crest through the knees

A. dose alert An automated dose alert (CT Dose check) function is common feature of a modern CT system. This function automatically alerts the CT operator if the selected scan parameters may result in a patient dose that exceeds the predefined dose index value. Dose alert, or dose check, is a required system function to meet current industry standards (NEMA XR 29-2013) for patient radiation dose reduction during CT.

In an effort to avoid excessive radiation exposure to the patient, a CT system may be equipped with a(n) ___________ system that automatically notifies the technologist about improper technical factors before data acquisition. a. dose alert b. electrocardigram (ECG) trigger c. multiple scan average dose (MSAD) d. interpolation

C. size or weight Regardless of age or sex indication, the CT protocol should be optimized according to size or weight.

In an effort to reduce patient radiation dose, the technical factors of the applied CT protocol should be optimized on the basis of: a. age b. sex c. size or weight d. physical condition

C. twelfth thoracic vertebra and the second lumbar vertebra

In general, the pancreas is located between the areas of the: A) fifth and eighth thoracic vertebrae B) ninth and eleventh thoracic vertebrae C) twelfth thoracic vertebra and the second lumbar vertebra D) second and fourth lumbar vertebrae

C. 2.0 mg/dL In general, the normal range for serum creatinine is 0.6 to 1.7 mg/dL; radiologists are typically consulted whenever the creatinine is 2.0 mg/dL or greater

In many institutions a radiologist is consulted before intravenous contrast medium is administered to patients in whom the creatinine value is greater than: a. 0.4 mg/dL b. 0.8 mg/dL c. 2.0 mg/dL d. 8.0 mg/dL

D) Water ***Water may be the preferred oral contrast utilized for CT exams of the pancreas***

In most institutions what is preferred oral contrast administered to patients undergoing a CT scan of the pancreas? A) Barium Sulfate B) VoLumen C) Water-soluble iodinated D) Water

D) Water ***Water may be the preferred oral contrast utilized for CT exams of the pancreas***er

In most institutions what is preferred oral contrast administered to patients undergoing a CT scan of the pancreas? A) Barium Sulfate B) VoLumen C) Water-soluble iodinated D) Water

B. CO2 induces a less spastic response of the bowel wall and is therefore better tolerated by most patients Both room air and CO2 provide very high negative contrast and reliably distend the colon. Although both are safe and produce only slight post-procedure discomfort, CO2 tends to be better tolerated by patients.

In performing CT colonography, which is an advantage of CO2 over room air? A) only CO2 can provide reliable colonic distention B) CO2 induces less spastic response of the bowel wall and is therefore better tolerated by most patients C) room air has been associated with infections of the colon D) CO2 provides a higher negative contrast on images

C. a nasogastric tube may be inserted for the administration of an oral contrast agent

In scanning the abdomen, what can be done if the patient is unable to take fluids by mouth? A) the volume of intravenous contrast material is increased by 25% to compensate for the lack of oral contrast medium B) 600 mL of dilute water-soluble agent is administered by enema C) a nasogastric tube may be inserted for the administration of an oral contrast agent D) a small volume of dilute water-soluble contrast agent can be administered by an inhaler

C. glabellomeatal line The glabellomeatal line is the imaginary reference line that runs approximately 15 degrees cephalad form the orbital meatal line. The glabella is the surface of the frontal bone lying between the eyebrows.

In scanning the brain, using which one of the following reference lines reduces the radiation exposure to the lens of the eye? A) canthomeatal line B) orbital meatal line C) glabellomeatal line D) infraorbital meatal line

A. before the CT examination, the Technologist explains the procedure to the patient and asks them if they agree. Choice b: describes informed consent Choice c: describes implied consent Choice d: describes parental consent

In the CT setting, which describes the concept of basic (or simple) consent? A) before the CT examination, the technologist explains the procedure to the patient and asks them if they agree B) a written document is given to the patient or guardian (or read to them, if necessary). The document lists all of the potential complications of the procedure. The patient must sign the form to acknowledge that they understand both the risks and benefits of the examination C) consent that is inferred from signs or action. For example, if a patient holds out his arm so that you can start an IV line, it can be implied that he gives his consent to the procedure D) the requirement that, for minors, one or more parent must consent to the procedure

C. Eight A byte is a series of eight bits Bits and bytes are part of the binary language used by computers to process information.

In the binary number system, a byte is a series of _______ bits of information a. two b. four c. eight d. sixteen

A. redundancy Redundancy can also refer to the duplication of data to provide an alternative in case of failure of one part of the process. Planning for redundancy is an important part of any radiology information system.

In the computing world, ____________ is used to describe an arrangement in which two or more components perform the same task -- if one element fails, the duplication keeps the system functioning while the failed component is repaired. A) redundancy B) bandwidth C) overlapping D) protocol

C) Algebraic reconstruction ***The simplest form of reconstruction was the algebraic method of reconstruction performed by Hounsfield. This method used a liner system of equations to solve a problem***

In the early days of CT scanning, reconstruction algorithms were very antiquated. What type of reconstruction algorithm was performed by Hounsfield during his first experiments? A) Back projections B) Filtered back projections C) Algebraic reconstruction D) Iterative reconstruction

C) Algebraic reconstruction ***The simplest form of reconstruction was the algebraic method of reconstruction performed by Hounsfiled. This method used a liner system of equations to solve a problem***

In the early days of CT scanning, reconstruction algorithms were very antiquated. What type of reconstruction algorithm was performed by Hounsfiled during his first experiments? A) Back projections B) Filtered back projections C) Algebraic reconstruction D) Iterative reconstruction

A) Uterus ***The Uterus is a pear-shaped organ located between the bladder and the rectum. The part of the uterus is the fundus, body, and the cervix***

In the female pelvis what soft tissue structure is located posterior to the urinary bladder and anterior to the rectum? A) Uterus B) Ovaries C) Urethra D) Symphysis Pubis

A) Uterus ***The Uterus is a pear-shaped organ located between the bladder and the rectum. The part of the uterus is the fundus, body, and the cervix***

In the female pelvis what soft tissue structure is located posterior to the urinary bladder and anterior to the rectum? A) Uterus B) Ovaries C) Urethra D) Symphysis Pubis

C) epiglottis

In the figure, #17 is which of the following? a) hyoid bone b) vallecular c) epiglottis d) piriformis sinus

A) mandibular condyle

In the figure, #33 represents which anatomical structure? a) mandibular condyle b) mastoid atrium c) internal auditory canal d) foramen rotundum

A) ethmoid sinus

In the figure, label #18 represents which of the following structures? a) ethmoid sinus b) maxillary sinus c) sphenoid sinus d) lacrimal duct

A) orbital floor

In the figure, label #31 represents which of the following structures? a) orbital floor b) inferior rectus muscle c) middle rectus muscle d) inferior oblique muscle

B) internal jugular vein

In the figure, label #5 represents which of the following structures? a) common carotid artery b) internal jugular vein c) internal carotid vein d) external carotid vein

B) hepatic cysts Cysts appear as homogenous, well-defined, round, or oval-shaped thin-walled lesions that have a near water attenuation value and should not enhance with IV contrast.

In the figure, the radiologist interpreted the case as low-density lesions in the liver having a near water CT attenuation value and smooth margins. What is the diagnosis? a) hepatic abscesses b) hepatic cysts c) hepatocellular carcinoma d) metastatic lesions

C) 31

In the figure, the semispinalis capitis muscle is illustrated by which of the following numbers? a) 41 b) 33 c) 31 d) 42

D) bronchogenic carcinoma

In the figure, what is the pathological process demonstrated? a) cystic fibrosis b) metastatic lung disease c) lung abscess d) bronchogenic carcinoma

D) renal artery stenosis Demonstrates atherosmatous plaque along the aortic wall and small filling defect in the proximal righty renal artery, resulting in renal artery stenosis.

In the figure, what pathologic disorder is depicted in the coronal MPR image? a) double renal artery b) hydronephrosis c) Nutcracker syndrome d) renal artery stenosis

B) 14

In the figure, which of the following illustrates the right lobe of the liver? a) 15 b) 14 c) 23 d) 18

B) transverse foramen

In the figure, which of the following is illustrated by #12? a) pedicle b) transverse foramen c) ventral rami d) intervertebral foramen

A) splenic flexure

In the figure, which of the following is illustrated by #6? a) splenic flexure b) transverse colon c) descending colon d) sigmoid colon

C) odontoid process

In the figure, which of the following structures is labeled #7? a) lateral mass of the atlas b) occipital bone c) odontoid process d) transverse ligament attachment

B. patient arms in the SFOV (scan field of view) The most likely cause of the streak artifact is from the patient's arms positioned down by his or her sides for scanning. This can be inferred, even without the scout image to confirm, by examining the streaks, which appear to have their origins outside, on either side, of the displayed image. This type of artifact is broadly categorized as out-of-field artifacts.

In the image, what is the likely cause of the artifact? A) patient motion B) patient arms in the SFOV C) metallic object D) not enough data collected (undersampling)

B. Bolus The bolus phases is that which immediately follows an IV bolus injection. It is characterized by a dramatic attenuation difference between the aorta and the inferior vena cava.

In the image, what phase of tissue enhancement is the iodinated contrast? A) noncontrast B) bolus C) nonequilibrium D) equilibrium

C) Scrotum ***it facilitates sperm formation by distending the testes outside the peritoneum in a cooler environment if an effort to regulate the temp of the testes***

In the male pelvis, what musculotendinous pouch encloses the testes, epididymis, and lower portion of the spermatic cord? A) Tunica albuginea B) Vas deferens C) Scrotum D) Corpora cavernose

D. 99% Dissipating the by-product heat generated in the production of x-rays is an important aspect of the CT system design.

In the production of x-rays, approximately what percentage of the kinetic energy of the projectile electrons is converted to thermal energy? A) 2% B) 50% C) 75% D) 99%

D. a dialysis catheter should never be used for contrast media administration

In what circumstances is it acceptable to use a large-bore tunneled dialysis catheter for the injection of iodinated contrast media? A) whenever a standard peripheral IV access is not available B) when the contrast flow rate is 2 mL/s or less C) when the patency of the line has been verified by flushing with saline D) a dialysis catheter should never be used for contrast media administration

B. when oral contrast is needed for neonatal CT examinations In most cases, HOCM (high osmolality contrast media - Hypaque) is used for oral administration because it is less expensive than LOCM (low osmolality contrast media - Omnipaque) and provides equivalent gastrointestinal opacification. However, in some cases LOCM has advantages. If aspirated, LOCM causes less pulmonary edema than HOCM. In newborns, researchers have noted a reduction in complications when LOCM is used compared with barium sulfate or HOCM. Barium sulfate is preferred when a patient is unable to drink the full dose of oral contrast. This is because in small amounts barium sulfate tends to cling onto the intestinal wall, providing at least a small amount of visible contrast. In comparison, the bowel usually absorbs a small quantity of water-soluble oral contrast.

In which of the following situations would an LOCM be beneficial when delivered orally? A) when a patient is unable to drink the full dose of oral contrast B) when oral contrast is needed for neonatal CT examinations C) when patients are older than 70 years of age D) when patients have a hiatal hernia

B. in the x,y direction *Resolution in the x,y direction is called in-plane resolution.* The in-plane resolution is affected by pixel size, which is affected by the DFOV. *Resolution in the z direction is called longitudinal resolution.* This is affected largely by slice thickness and (in helical scans) pitch.

In-plane resolution refers to the resolution... A) resulting from helical data B) in the x,y direction C) in the z direction D) related to the slice thickness

A. speed of the electrons as they travel from filament to anode Electrons "boil off" the heated filament and are propelled across to strike the anode, where they disarrange the target material to produce x-ray photons. The x-ray photons (not electrons) then travel from the anode to the patient. Increasing the kilovoltage increases the speed at which they travel. Ultimately, this results in a higher photon energy of the resultant x-ray beams.

Increasing the kilovoltage results in an increase in the: A) speed of the electrons as they travel from filament to anode B) number of electrons that "boil off" the filament C) speed of electrons as they travel from the anode through the patient D) rate at which the system can rid itself of by-product heat

C) 5cc/s ***When performing spiral scanning, contrast injection rates as high as 5cc/s provides excellent opacification of arterial structures***

Injection flow rate is a pertinent aspect when scanning, which of the following flow rates will deliver a higher concentration of contrast to the desired area when the scan commences? A) 1cc/s B) 3cc/s C) 5cc/s D) Drip infusion

B) a mathematical method to estimate a missing value by taking an average of known values at neighboring points This method is used in helical scanning. The system takes the slanted helical data and estimates what its appearance would be if the slices were taken axially, with each slice parallel to the plane of the table. The greater the slant (i.e. the higher the pitch), the more interpolation is required. Choice A) is the definition of standard deviation.

Interpolation, as it is used in CT is... A) a statistical measure of the spread or dispersion of a set of data; it is the positive square root of the variance B) a mathematical method to estimate a missing value by taking an average of known values at neighboring points C) the method used to convert the electric signal to a digital format D) the method used to convert light levels into an electric current

Unstable atoms

Ionization may cause:

C. liver

It is imperative that all scans be completed before the equilibrium phase of contrast enhancement in studies of the: A) brain B) urinary tract C) liver D) postsurgical lumbar disk

D) mitral valve

Label #38 represents which of the following anatomical structures? a) tricuspid valve b) semilunar aortic valve c) semilunar pulmonary valve d) mitral valve

A. stochastic A stochastic, or random, effect of radiation exposure is one having no threshold dose. Common examples of stochastic radiation effects are genetic mutations and cancer.

Late effects of radiation, such as genetic mutations, may occur with even small doses of radiation and are termed: a. stochastic b. somatic c. nonstochastic d. chronic

B. number of pixels used to display image Consisting of an arrangement of pixels in rows and columns, the MATRIX is used to organize the attenuation information from the anatomic section into a digital image. The size of the matrix is given as the number of pixels across multiplied by the number of pixels down (length x width).

MATRIX SIZE describes which of the following? A) aperture size used during data acquisition B) number of pixels used to display image C) relationship between the field of view (FOV) and the algorithm D) the number of data channels available

A. 3 HU Each 1.0% difference in contrast between between adjacent objects amounts to a difference in pixel value of approximately 10 HU. MDCT systems are typically capable of capturing adjacent objects with attenuation differences AS SMALL AS 3 HU.

MDCT systems are typically capable of differentiating adjacent objects with attenuation differences as small as: a. 3 HU b. 10 HU c. 25 HU d. 60 HU

D. uniform arrays MDCT systems with detectors that contain thinner rows centrally are called adaptive arrays, nonuniform arrays, or hybrid arrays. Fourth generation scanners (in which detector are situated in a complete ring) are not used in MDCT systems because too many detector elements would be required.

MDCT systems that contain parallel rows of equal size detectors are called A) adaptive arrays B) hybrid arrays C) fourth-generation arrays D) uniform arrays

B. solid-state scintillation All modern MDCT systems utilize solid-state detectors consisting primarily of a scintillating crystal material. Solid-state detectors are preferred for MDCT because of their ability to accurately record incident x-ray energy from any angle. This flexibility is important when one considers the widened cone beam geometry inherent to MDCT systems.

MDCT systems typically employ which of the following types of detectors? a. tungsten ring b. solid-state scintillation c. charged-coupled device (CCD) d. gas ionization

Roentgen

Measurement of positive and negative particles created when radiation ionizes atoms in the air help define the:

Artificially produced radiation

Medical x-rays are an example of:

A. the shape of the x-ray beam before (pre) and after (post) the patient Model-based iterative reconstruction (MBIR) considers the shape of the CT x-ray beam both preparing and postpatient and uses complex statistical analyses in both a forward and backward series of reconstructions to arrive at an improved image with reduced noise.

Model-based iterative reconstruction (MBIR) considers which of the following during CT image formation? A) the shape of the x-ray beam before (pre) and after (post) the patient B) a master image stored in the CT system's computer C) the detector configuration used for the acquisition D) the age, gender and weight of the scanned patient

Silicon-based semiconductors

Modern rectifiers are made of:

C. third generation All MDCT systems utilize third-generation geometry and solid-state scintillation detectors.

Multidetector CT (MDCT) systems typically employ which of the following detector geometries? a. first generation b. second generation c. third generation d. fourth generation

D. left adrenal gland

Name the structure indicated by #5. A) superior mesenteric artery B) superior mesenteric vein C) splenic artery D) left adrenal

C. lateral ventricle

Name the structure indicated by arrow #1. A) corpus callosum B) caudate nucleus C) lateral ventricle D) external capsule

D. ascending colon (cecum)

Name the structure indicated by arrow #1. A) descending colon B) jejunum/ileum C) inflamed appendix D) ascending colon

C. right pulmonary artery

Name the structure indicated by arrow #1. A) left atrium B) aortic arch C) right pulmonary artery D) right brachiocephalic vein

B. incus

Name the structure indicated by arrow #1. A) mastoid air cells B) incus C) stapes D) cochlea

C. aorta

Name the structure indicated by arrow #1. A) portal vein B) inferior vena cava C) aorta D) left renal artery

B. calcaneus

Name the structure indicated by arrow #1. A) talus B) calcaneus C) navicular bone D) distal fibula (lateral malleolus)

D. superior vena cava

Name the structure indicated by arrow #2. A) brachiocephalic artery B) ascending aorta C) right pulmonary vein D) superior vena cava

B. duodenum

Name the structure indicated by arrow #2. A) hepatic flexure of colon B) duodenum C) gastroesophageal junction D) body of stomach

B. thyroid cartilage

Name the structure indicated by arrow #2. A) hyoid bone B) thyroid cartilage C) mandible D) cricoid cartilage

A. malleus

Name the structure indicated by arrow #2. A) malleus B) stapes C) trigeminal nerve branches D) sella turcica

A. right common iliac vein

Name the structure indicated by arrow #2. A) right common iliac vein B) right common iliac artery C) right ovary D) iliacus muscle

C. cuboid bone

Name the structure indicated by arrow #2. A) talus B) cuneiform (lateral or 1st) C) cuboid bone D) metatarsal bone

A. left common iliac vein

Name the structure indicated by arrow #3, A) left common iliac vein B) left common iliac artery C) celiac trunk D) azygos vein

B. epitympanic recess

Name the structure indicated by arrow #3. A) mastoid antrum B) epitympanic recess C) internal auditory canal D) vestibule

A. portal vein

Name the structure indicated by arrow #3. A) portal vein B) inferior vena cava C) superior mesenteric artery D) hepatic artery

B. caudate nucleus

Name the structure indicated by arrow #3. A) putamen B) caudate nucleus C) thalamus D) pons, medulla oblongata

B. ascending aorta

Name the structure indicated by arrow #3. A) right atrium B) ascending aorta C) right ventricle D) superior vena cava

B. cuneiform (intermedial or 2nd)

Name the structure indicated by arrow #3. A) talus B) cuneiform (intermedial or 2nd) C) navicular bone D) metatarsal bone

A. trachea

Name the structure indicated by arrow #3. A) trachea B) piriform recess C) laryngeal vestibule D) pharynx

D. calcified pineal body

Name the structure indicated by arrow #4. A) basilar artery B) sigmoid sinus C) anterior cerebral artery D) pineal body (calcified)

D. common carotid artery

Name the structure indicated by arrow #4. A) basilar artery B) vertebral artery C) internal carotid artery D) common carotid artery

A. head of pancreas

Name the structure indicated by arrow #4. A) head of pancreas B) tail of pancreas C) duodenum D) left renal artery

B. left common iliac artery

Name the structure indicated by arrow #4. A) left common iliac vein B) left common iliac artery C) celiac trunk D) azygos vein

D. metatarsal bone

Name the structure indicated by arrow #4. A) phalange (proximal) B) distal tibia (medial malleolus) C) navicular bone D) metatarsal bone

A. pulmonary trunk

Name the structure indicated by arrow #4. A) pulmonary trunk B) descending aorta C) left atrium D) superior vena cava

C. cochlea

Name the structure indicated by arrow #4. A) tympanic sinus B) internal jugular vein C) cochlea D) facial nerve

C. esophagus

Name the structure indicated by arrow #5. A) bronchus intermedius B) azygos vein C) esophagus D) right crus of diaphragm

A. cuneiform (medial or 1st)

Name the structure indicated by arrow #5. A) cuneiform bone (medial or 1st) B) cuboid bone C) calcaneus D) metatarsal bone

B. vertebral canal with spinal cord

Name the structure indicated by arrow #5. A) nucleus pulposus B) vertebral canal with spinal cord C) transverse ligament D) intervertebral foramen

C. Jugular vein

Name the structure indicated by arrow #5. A) pericallosal arteries B) thyroid gland C) jugular vein D) lingual artery

A. vestibule

Name the structure indicated by arrow #5. A) vestibule B) facial canal C) cochlea D) internal carotid artery canal

A. lateral semicircular canal

Name the structure indicated by arrow #6. A) lateral semicircular canal B) round window C) eustachian tube D) vestibular aqueduct

C. psoas muscle

Name the structure indicated by arrow #6. A) rectus abdominis muscle B) diaphragm C) psoas muscle D) erector spinae muscle

A. falx cerebri

Name the structure indicated by arrows number #2 and # 5. A) falx cerebri B) pericallosal arteries C) vermis of cerebellum D) central sulcus

C. thyroid gland

Name the structured indicated by arrow #1. A) sternocleidomastoid muscle B) scalenus muscle C) thyroid gland D) cricoid cartilage

Unintentional misconduct

Neglect or omission of reasonable care defines:

B. urinary tract lithiasis Lithiasis = stone Unenhanced helical CT of the urinary tract has become the standard for the investigation of urinary tract lithiasis/KIDNEY STONES.

Noncontrast CT of the urinary tract is a valuable tool in the investigation of: a. transitional cell carcinoma b. urinary tract lithiasis c. renal artery stenosis d. ureteral duplication

12 - 16 breaths per minute

Normal adult respiration is:

D. 95-100% Normal range of O2 saturation as measured by a pulse oximeter is between 95 and 100%. Saturation levels below 95% may indicate respiratory insufficiency.

Normal range for oxygen (O2) saturation is: a. 20-25% b. 50-60% c. 80-85% d. 95-100%

In the health care setting

Nosocomial infections are acquired:

C. inferior oblique muscle

Number 1 on the figure corresponds to which of the following? A) inferior rectus muscle B) inferior rectus muscle C) inferior oblique muscle D) levator palpebrae superioris

A. rectus abdominis

Number 1 on the figure corresponds to which of the following? A) rectus abdominis muscle B) external oblique muscle C) transverse abdominis muscle D) internal oblique muscle

B. right pulmonary artery

Number 1 on the figure corresponds to which of the following? A) right brachiocephalic artery B) right pulmonary artery C) right pericardiacophrenic artery D) right subclavian artery

C. left brachiocephalic vein

Number 1 on the figure corresponds to which of the following? A) right brachiocephalic vein B) left subclavian artery C) left brachiocephalic vein D) brachiocephalic artery

D. genu of the corpus callosum

Number 1 on the figure corresponds to which of the following? A) splenium of the corpus callosum B) internal capsule C) body of the corpus callosum D) genu of the corpus callosum

A. superior vena cava

Number 1 on the figure corresponds to which of the following? A) superior vena cava B) ascending aorta C) right brachiocephalic vein D) right brachiocephalic artery

A. capitate

Number 1 on the figure corresponds to which of the following? a. capitate b. lunate c. hamate d. scaphoid

A. common carotid artery

Number 1 on the figure corresponds to which of the following? a. common carotid artery b. external jugular vein c. internal jugular vein d. brachiocephalic artery

A. Left pulmonary artery

Number 1 on the figure corresponds to which of the following? a. left pulmonary artery b. ascending aorta c. inferior vena cava d. descending aorta

D. talus

Number 1 on the figure corresponds to which of the following? a. navicular b. cuboid c. calcaneus d. talus

D. rectus abdominis muscle

Number 1 on the figure corresponds to which of the following? a. psoas muscle b. gluteus medius muscle c. iliacus muscle d. rectus abdominis muscle

C. Ascending aorta

Number 1 on the figure corresponds to which of the following? a. right atrium b. pulmonary trunk c. ascending aorta d. right ventricle

B. Ethmoid sinus

Number 1 on the figure corresponds to which of the following? a. sphenoid sinus b. ethmoid sinus c. frontal sinus d. maxillary sinus

D. transverse foramen

Number 1 on the figure corresponds to which of the following? a. spinal root b. superior articular recess c. anterior arch d. transverse foramen

B. piriform sinus

Number 1 on the figure corresponds to which of the following? a. trachea b. piriform sinus c. aryepiglottic fold d. vocal cord

A. ureteropelvic junction

Number 1 on the figure corresponds to which of the following? a. ureteropelvic junction b. ureterovesical junction c. ureterocalyceal junction d. ureteropyramids junction

D. Epiglottis

Number 1 on the figure corresponds to which of the following? a. vocal cords b. uvula c. aryepiglottic fold d. epiglottis

C. Cochlea

Number 2 corresponds to which of the following? a. carotid canal b. incus c. cochlea d. carotid canal

C. portal vein

Number 2 in the figure corresponds to which of the following? a. inferior vena cava b. common bile duct c. portal vein d. superior mesenteric vein

B. pulmonary trunk

Number 2 on the figure corresponds to which of the following? A) aortic arch B) pulmonary trunk C) superior vena cava (SVC) D) left anterior descending artery

C. Left internal carotid artery

Number 2 on the figure corresponds to which of the following? A) left internal jugular vein B) left external carotid artery C) left internal carotid artery D) left external jugular vein

B. Anterior segment of left upper lobe bronchus

Number 2 on the figure corresponds to which of the following? A) left mainstem bronchus B) anterior segment of left upper lobe bronchus C) posterior segment of left upper lobe bronchus D) anterior segment of left lower lobe bronchus

B. left middle cerebral artery

Number 2 on the figure corresponds to which of the following? A) left posterior cerebral artery B) left middle cerebral artery C) left anterior cerebral artery D) left posterior communicating artery

A. left pulmonary artery

Number 2 on the figure corresponds to which of the following? A) left pulmonary artery B) descending aorta C) left brachiocephalic artery D) left subclavian artery

B. left subclavian artery

Number 2 on the figure corresponds to which of the following? A) right brachiocephalic vein B) left subclavian artery C) left brachiocephalic vein D) brachiocephalic artery

A. Superior mesenteric vein

Number 2 on the figure corresponds to which of the following? A) superior mesenteric vein B) superior mesenteric artery C) renal artery D) portal vein

D. scaphoid

Number 2 on the figure corresponds to which of the following? a. capitate b. lunate c. hamate d. scaphoid

D. scaphoid

Number 2 on the figure corresponds to which of the following? a. capitate b.. lunate c. hamate d. scaphoid

B. acromion

Number 2 on the figure corresponds to which of the following? a. coracoid process b. acromion c. glenoid process d. clavicle

C. Adrenal gland

Number 2 on the figure corresponds to which of the following? a. hepatic vein b. suprarenal lymph node c. adrenal gland d. common bile duct

C. thyroid cartilage

Number 2 on the figure corresponds to which of the following? a. hyoid bone b. cricoid cartilage c. thyroid cartilage d. mandible

B. sigmoid colon

Number 2 on the figure corresponds to which of the following? a. jejunum b. sigmoid colon c. ileum d. cecum

A. left ovary

Number 2 on the figure corresponds to which of the following? a. left ovary b. bladder c. sigmoid colon d. uterus

C. transverse process

Number 2 on the figure corresponds to which of the following? a. superior facet b. pedicle c. transverse process d. lamina

B. optic nerve

Number 2 on the figure corresponds tow hich of the following? a. medial rectus muscle b. optic nerve c. lateral rectus muscle d. superior oblique muscle

D. pancreas

Number 3 in the figure correspond to which of the following? a. duodenum b. splenic vein c. jejunum d. pancreas

A. left posterior cerebral artery

Number 3 on the figure corresponds to which of the following? A) left posterior cerebral artery B) left middle cerebral artery C) left anterior cerebral artery D) left posterior communicating artery

b. Thalamus

Number 3 on the figure corresponds to which of the following? a. caudate nucleus b. thalamus c. third ventricle d. pineal gland

C. glenoid process

Number 3 on the figure corresponds to which of the following? a. coracoid process b. acromion c. glenoid process d. clavicle

D. vocal cord

Number 3 on the figure corresponds to which of the following? a. cricoid cartilage b. piriform cartilage c. aryepiglottic fold d. vocal cord

D. left primary bronchus

Number 3 on the figure corresponds to which of the following? a. esophagus b. left pulmonary vein c. trachea d. left primary bronchus

C. abdominal aorta

Number 3 on the figure corresponds to which of the following? a. inferior vena cava b. common iliac artery c. abdominal aorta d. superior mesenteric artery

D. spinous process

Number 3 on the figure corresponds to which of the following? a. pedicle b. transverse process c. lamina d. spinous process

A. pineal gland

Number 3 on the figure corresponds to which of the following? a. pineal gland b. middle cerebral artery c. fourth ventricle d. aqueduct of Sylvius

A. Right ureter

Number 3 on the figure corresponds to which of the following? a. right ureter b. inferior vena cava c. renal calculi d. right renal artery

A.superior rectus muscle

Number 3 on the figure corresponds to which of the following? a. superior rectus muscle b. lateral rectus muscle c. medial rectus muscle d. inferior rectus muscle

C. seminal vesicles

Number 3 on the figure corresponds to which of the following? a. ureters b. cervix c. seminal vesicles d. vaginal cuff

A. Vestibule

Number 3 on the figure corresponds to which of the following? a. vestibule b. semicircular canal c. cochlea d. incus

D. Carotid canal

Number 4 corresponds to which of the following? a. external auditory meatus b. internal auditory canal c. vestibule d. carotid canal

D. large colon (splenic flexure)

Number 4 in the figure corresponds to which of the following? a. spleen b. duodenum c. left adrenal gland d. large colon

C. ureterovesical junction

Number 4 on the figure corresponds to whcih of the following? a. ureteropelvic junction b. phlebolith c. ureterovesical junction d. appendicolith

B. pectoralis minor

Number 4 on the figure corresponds to which of the following muscles? A) pectoralis major B) pectoralis minor C) intercostal D) serratus posterior superior

C. left common iliac vein

Number 4 on the figure corresponds to which of the following? A) left common iliac artery B) right common iliac artery C) left common iliac vein D) right common iliac artery

D. superior oblique muscle

Number 4 on the figure corresponds to which of the following? A) superior rectus muscle B) superior ophthalmic vein C) levator palpebrae superioris muscle D) superior oblique muscle

B. spleen

Number 4 on the figure corresponds to which of the following? a. descending colon b. spleen c. adrenal gland d. renal vein

D. levator scapulae muscle

Number 4 on the figure corresponds to which of the following? a. erector spinae muscle b. rhomboid major muscle c. deltoid muscle d. levator scapulae muscle

D. descending aorta

Number 4 on the figure corresponds to which of the following? a. gastric artery b. inferior vena cava (IVC) c. superior mesenteric artery (SMA) d. descending aorta

D. uterus

Number 4 on the figure corresponds to which of the following? a. left ovary b. bladder c. sigmoid colon d. uterus

B. cuboid

Number 4 on the figure corresponds to which of the following? a. navicular b. cuboid c. calcaneus d. talus

B. lamina

Number 4 on the figure corresponds to which of the following? a. spinous process b. lamina c. transverse process d. pedicle

D. lamina

Number 4 on the figure corresponds to which of the following? a. superior facet b. pedicle c. transverse process d. lamina

C. fourth ventricle

Number 4 on the figure corresponds to which of the following? a. third ventricle b. quadrigeminal cistern c. fourth ventricle d. sagittal sinus

D. basilar artery

Number 5 on the figure corresponds to which of the following? A) posterior communicating artery B) anterior communicating artery C) internal carotid artery D) basilar artery

A. right brachiocephalic vein

Number 5 on the figure corresponds to which of the following? A) right brachiocephalic vein B) left subclavian artery C) left brachiocephalic vein D) brachiocephalic vein

A. Right internal jugular vein

Number 5 on the figure corresponds to which of the following? A) right internal jugular vein B) right external carotid artery C) right internal carotid artery D) right external jugular vein

B. descending colon

Number 5 on the figure corresponds to which of the following? a. ascending colon b. descending colon c. sigmoid colon d. cecum

B. lunate

Number 5 on the figure corresponds to which of the following? a. capitate b. lunate c. hamate d. scaphoid

B. bladder

Number 5 on the figure corresponds to which of the following? a. left ovary b. bladder c. sigmoid colon d. uterus

B. superior ophthalmic vein

Number 5 on the figure corresponds to which of the following? a. optic nerve b. superior ophthalmic vein c. oculomotor nerve d. superior oblique muscle

B. right ureter

Number 5 on the figure corresponds to which of the following? a. right common iliac vein b. right ureter c. left common iliac vein d. inferior mesenteric vein

B. Stomach

Number 5 on the figure corresponds to which of the following? a. transverse colon b. stomach c. duodenum d. hepatic flexure

B. superior mesenteric artery

Number 6 in the figure correspond to which of the following? a. superior mesenteric vein b. superior mesenteric artery c. gastric artery d. right renal vein

B. falx cerebri

Number 6 on the figure corresponds to which of the following? a. anterior cerebral artery b. falx cerebri c. callosal marginal artery d. middle cerebral artery

B. azygos vein

Number 6 on the figure corresponds to which of the following? a. descending aorta b. azygos vein c. inferior vena cava d. brachiocephalic artery

D. pancreas

Number 6 on the figure corresponds to which of the following? a. duodenum b. terminal ileum c. appendix d. pancreas

A. navicular

Number 6 on the figure corresponds to which of the following? a. navicular b. cuboid c. calcaneous d. talus

C. lateral rectus muscle

Number 6 on the figure corresponds to which of the following? a. optic nerve b. medial rectus muscle c. lateral rectus muscle d. oculomotor nerve

A. Psoas msucle

Number 6 on the figure corresponds to which of the following? a. psoas muscle b. omentum c. piriformis muscle d. erector spinae muscle

C. renal calyx

Number 6 on the figure corresponds to which of the following? a. renal cortex b. renal pelvis c. renal calyx d. renal pyramid

C. ascending aorta

Number 6 on the figure corresponds to which of the following? a. right atrium b. pulmonary trunk c. ascending aorta d. descending aorta

C. bladder

Number 6 on the figure corresponds to which of the following? a. uterus b. rectum c. bladder d. sigmoid colon

B. Pancreatic head

Number 7 on the figure corresponds to which of the following? a. duodenum b. pancreatic head c. jejunum d. gallbladder

D. Ascending colon

Number 8 on the figure corresponds to which of the following? a. duodenum b. descending colon c. jejunum d. ascending colon

C. maxillary sinus

Number 9 on the figure corresponds to which of the following? a. ethmoid sinus b. nasal concha c. maxillary sinus d. sphenoid sinus

A) Windowing ***IS a computer visualization tool by which the image gray scale is manipulated with regard to the CT number of the structure of interest.***

Of the following, what is considered an image display technique used to optimize image viewing by means of gray scale manipulations? A) Windowing B) Cine mode C) Image magnification D) Multiplanar reformation

A) Windowing ***IS a computer visualization tool by which the image gray scale is manipulated with regard to the CT number of the structure of interest.***

Of the following, what is considered an image display technique used to optimize image viewing by means of gray scale manipulations?? A) Windowing B) Cine mode C) Image magnification D) Multiplanar reformation

C) ACR-NEMA ***DICOM was originally developed by the National Electrical Manufacturers Association (NEMA) and the American College of Radiology (ACR) in 1985 the ACR-NEMA committe was formed to explore ways of standardizing the interconnection of imaging devices and establishing a reasonable common ground for users and vendors to transmit digital data files***

Of the following, what organization was responsible for the development of the DICOM standard? A) ACRIN B) ASRT-ARRT C) ACR-NEMA D) NASA

C) ACR-NEMA ***DICOM was originally developed by the National Electrical Manufacturers Association (NEMA) and the American College of Radiology (ACR) in 1985 the ACR-NEMA committee was formed to explore ways of standardizing the interconnection of imaging devices and establishing a reasonable common ground for users and vendors to transmit digital data files***

Of the following, what organization was responsible for the development of the DICOM standard? A) ACRIN B) ASRT-ARRT C) ACR-NEMA D) NASA

C) 55 to 65 HU ***Which is slightly higher in attenuation when comparing to other soft tissue organs***

On an unenhanced abdominal CT, what is the range of CT attenuation values of a normal liver? A) -10 to 0 HU B) 10 to 40 HU C) 55 to 65 HU D) 125 to 165 HU

B. 38HU to 70HU The normal CT attenuation of the liver in unenhanced studies varies among individuals and ranges from 38 - 70HU.

On an unenhanced abdominal CT, what is the range of CT attenuation values of a normal liver? A) -10HU to 30HU B) 38HU to 70HU C) 85HU to 132HU D) 147HU to 165HU

C. the effective width of section B is greater than that of section A The section width (slice) for a volumetric acquisition may be graphically displayed as a slice sensitivity profile (SSP). The effective section width is defined as the full width at half maximum (FWHM) of the SSP. The FWHM is labeled in the figure as C. At this point of the graph, the section width of B is wider than that of A.

On the basis of the information provided on the graph in the figure, which of the following statements is true? A) the limiting resolution in section A is less than that in section B B) the signal-to-noise ratio (SNR) of section A is greater than that of section B C) the effective width of section B is greater than that of section A D) section B is isotropic in dimension

b) 2

On this axial CT of the abdomen, which arrow points to Morison's pouch? a) 1 b) 2 c) 3 d) 4

b) 2

On this axial CT of the abdomen, which arrow points to the splenic vein? a) 1 b) 2 c) 3 d) 4

a) 1

On this axial CT of the chest, which arrow points to the epicardial fat? a) 1 b) 2 c) 3 d) 4

c) 3

On this axial CT of the chest, which arrow points to the left subclavian vein? a) 1 b) 2 c) 3 d) 4

d) 4

On this axial CT of the chest, which arrow points to the right ventricle? a) 1 b) 2 c) 3 d) 4

b) 2

On this axial CT of the neck muscles, which arrow points to the levator scapulae muscle? a) 1 b) 2 c) 3 d) 4

a) 1

On this axial CT of the neck, which arrow points to the arytenoid cartilage? a) 1 b) 2 c) 3 d) 4

a) 1

On this axial CT of the neck, which arrow points to the internal jugular vein? a) 1 b) 2 c) 3 d) 4

c) 3

On this axial CT of the neck, which arrow points to the parotid gland? a) 1 b) 2 c) 3 d) 4

b) 2

On this axial CT of the neck, which arrow points to the thyroid cartilage? a) 1 b) 2 c) 3 d) 4

d. 4

On this axial CT slice of the abdomen, which arrow points to the common hepatic duct? a. 1 b. 2 c. 3 d. 4

b. 2

On this axial CT slice of the abdomen, which arrow points to the left renal veins? a. 1 b. 2 c. 3 d. 4

a. 1

On this axial CT slice of the male pelvis, which arrow points to the right external iliac artery? a. 1 b. 2 c. 3 d. 4

a. 1

On this axial CT slice of the male pelvis, which arrow points to the seminal vesicle? a. 1 b. 2 c. 3 d. 4

b) 2

On this axial CT slice of the orbit, which arrow points to the anterior clinoid process? a) 1 b) 2 c) 3 d) 4

d) 4 A) B) C) D) E) F)

On this axial CT slice of the orbit, which arrow points to the dorsum sellae? a) 1 b) 2 c) 3 d) 4

a) 1

On this axial CT slice of the orbit, which arrow points to the optic canal? a) 1 b) 2 c) 3 d) 4

a) 1 A) B) C) D) E) F)

On this axial CT slice of the temporal bone, which arrow points to the carotid canal? a) 1 b) 2 c) 3 d) 4

b) 2

On this axial CT slice of the temporal bone, which arrow points to the jugular foramen? a) 1 b) 2 c) 3 d) 4

c) 3

On this axial CT slice of the temporal bone, which arrow points to the sphenoid sinus? a) 1 b) 2 c) 3 d) 4

a) 1

On this axial CT slice of the thoracic vertebra, which arrow points to the costovertebral joint? a) 1 b) 2 c) 3 d) 4

b) 2

On this axial CT slice of the thoracic vertebra, which arrow points to the lamina? a) 1 b) 2 c) 3 d) 4

d) 4 A) B) C) D) E) F)

On this axial CT slice of the thoracic vertebra, which arrow points to the pedicle? a) 1 b) 2 c) 3 d) 4

a) 1

On this coronal CT slice of the ethmoid bone, which arrow points to the cribriform plate? a) 1 b) 2 c) 3 d) 4

c) 3

On this coronal CT slice of the ethmoid bone, which arrow points to the ethmoid sinus? a) 1 b) 2 c) 3 d) 4

b) 2

On this coronal CT slice of the ethmoid bone, which arrow points to the vomer? a) 1 b) 2 c) 3 d) 4

C) Glomerular filtration rate

One of the following laboratory test listed below, which test can be utilized to evaluate the renal functions in a patient? A) D-Dimer B) INR C) Glomerular filtration rate D) PTT

D) Reduced noise ***The major advantages of 180 degree z-interpolation include: improved z-axis resolution, the generation of thinner slices, and scanning at higher pitches. These advantages are accomplished, since two data sets (spiral and virtual) are generated in opposite direction of each other***

One of the new z-interpolation process is 180 degrees linear interpolation. Which of the following is NOT an advantage of 180 degree linear interpolation? A) Scanning at pitches grater than 2 B) Narrower slices sensitivity profile C) Improved z-axis resolution D) Reduced noise

D) Reduced noise ***The major advantages of 180 degree z-interpolation include: improved z-axis resolution, the generation of thinner slices, and scanning at higher pitches. these advantages are accomplished, since two data sets (spiral and virtual) are generated in opposite direction of each other***

One of the new z-interpolation process is 180 degrees linear interpolation. Which of the following is NOT an advantage of 180 degree linear interpolation? A) Scanning at pitches grater than 2 B) Narrower slices sensitivity profile C) Improved z-axis resolution D) Reduced noise

D. 1, 2 and 3 -- restricting the acquired acquisition volume to the area of clinical interest, controlling access to the CT imaging room to essential personnel only and adjusting technical parameters (mA, kVp) to the size of the patient A comprehensive CT dose reduction program includes the minimization of acquisition volumes, tailoring the dose to the size of the patient, and restricting public and nonessential personnel access to the CT scan room.

Optimization of CT techniques and procedures for the purpose of radiation dose reduction for patients and personnel should include: 1. restricting the acquired acquisition volume to the area of clinical interest 2. controlling access to the CT imaging room to essential personnel only 3. adjusting technical parameters (e.g. mA, kVp) to the size of the patient a. 2 only b. 1 and 3 only c. 2 and 3 only d. 1, 2 and 3

A. seizures The use of iodinated contrast media has been shown to provoke seizures in patients who have diseases that disrupt the blood-brain barrier. In these patients, the risk of seizure can be reduced by a one-time dose of diazepam, 30 minutes before contrast administration. Seizures that occur can also be controlled with diazepam.

Patients with metastasis to the brain are at greater risk for _____________ after the administration of an IV contrast agent A) seizures B) contrast-induced nephropathy C) anaphylactoid reaction D) pulmonary effects, such as bronchospasm

C. abdominal abscess Computed tomography is commonly used during percutaneous aspiration and drainage procedures. CT images enable the radiologist to view the precise location of abnormal fluid collections and allow for accurate planning of a safe access route for the aspiration procedure. When performed properly, CT-guided aspiration of an abdominal abscess can be a valuable nonsurgical therapeutic technique. This type of procedures can also be used to reduce other types of abnormal fluid collections including cysts, bilomas, urinomas and lymphoceles.

Percutaneous drainage under CT guidance may be used for the aspiration of which of the following pathologic processes? a. chronic subdural hematoma b. hydrocephalus c. abdominal abscess d. dissecting aorta aneurysm

C. table speed Pitch is most commonly defined as the travel distance of the CT scan table per 360 degrees rotation of the x-ray tube, divided by the x-ray beam width. In the case of MDCT, the beam width can be determined by multiplying the number of slices by slice thickness.

Pitch is a parameter that is commonly used in helical CT to describe: A) the number of detectors in the z axis B) tube heat capacity C) table speed D) different types of interpolation methods

Picture element

Pixel is an acronym meaning:

B. positive When calculated by a formula comparing the attenuation coefficient of tissue with that of water, materials whose coefficients are greater than that of water are assigned positive CT numbers.

Pixels representing tissues with average attenuation coefficients greater than that of water have which of the following types of values? a. extremely small b. positive c. negative d. low contrast

C. negative When a formula comparing the attenuation coefficient of tissue with that of water is used, materials whose coefficients are less than that of water are assigned negative CT numbers.

Pixels whose average attenuation coefficients are less than the coefficient of water have which of the following types of CT number values? a. extremely large b. highly positive c. negative d. high contrast

D. 2 and 3 only -- making the patient more comfortable throughout the exam, and decreasing the lordotic curvature of the lumbar spine During a CT lumbar spine exam, a foam cushion should be placed under the patient's flexed knees. This maneuver reduces strain on the lower back, making the patient more comfortable and cooperative. It also reduces the lordotic curve of the lumbar spine, allowing for more accurate imaging of the intervertebral disc spaces.

Positioning the patient with the knees flexed over a foam cushion during a CT examination of the lumbar spine assists in: 1. decreasing the kyphotic curvature of the lumbar spine 2. making the patient more comfortable throughout the examination 3. decreasing the lordotic curvature of the lumbar spine a. 1 only b. 2 only c. 3 only d. 2 and 3 only

D. 60-100 kilowatts (kW) The power output of modern multidetector CT (MDCT) systems is vendor specific, within a typical range of 60 to 100 kilowatts (kW).

Power output for a modern multidetector CT (MDCT) x-ray tube has an approximate range of: a. 3-5 kilowatts (kW) b. 15-20 kilowatts (kW) c. 40-48 kilowatts (kW) d. 60-100 kilowatts (kW)

Poly-

Prefix that means "many" is:

C. 1 and 3 only -- soft hook-and-loop (Velcro) immobilization straps and good patient communication CT manufacturers supply comfortable, nonadhesive straps and cushions to help maintain motion-free patient positioning. Good patient communication is a key immobilization technique. Proper instructions for remaining still, suspending respiration, not swallowing, and so on help ensure patient cooperation and yield higher-quality CT examinations. Adhesive tape should NOT be used because of the abrasive effects of tape removal.

Proper immobilization during a CT procedure may involve the use of: 1. soft, hook-and-loop (e.g. Velcro) immobilization straps 2. adhesive medical tape 3. good patient communication a. 1 only b. 1 and 2 only c. 1 and 3 only d. 1, 2 and 3

C. R wave

Prospective ECG gating methods for imaging the heart use a signal, usually derived from the ___________ of the patient's ECG, to trigger image acquisition. A) ST segment B) PR segment C) R wave D) T wave

A. daily CT units should be calibrated with their reference CT number for water at approximately 0 daily. The CT number for air should be at approximately -1000.

Quality control measurements to test the accuracy of the CT scanner's calibration should be performed: a. daily b. weekly c. monthly d. annually

Genetic

Radiation effects that show up in the next generation are called:

Linear-nonthreshold

Radiation protection is based on which dose-response relationship?

Is highly ionizing

Radiation with a high LET:

C. coronal multiplanar reformation (MPR) images built from an axial acquisition An isotropic axial acquisition of the paranasal sinuses from the hard palate superiorly through the frontal sinus avoids artifact from dental fillings while providing excellent multiplanar reformations (MPR) in any plane. Direct coronal acquisition may not be required because the detail provided by the reformatted images is usually sufficient. This technique also aids in reducing patient radiation dose.

Reduction of the type of artifact seen in the figure on a coronal multidetector CT (MDCT) view of the sinuses may be accomplished through: A) an increase in the mA B) a decrease in the kVp C) coronal multiplanar reformation (MPR) images built from an axial acquisition D) a reduction in the scan field of view (SFOV)

B. orthogonal Orthogonal planes are at right angles at each other. Orthogonal reformations are perpendicular to the original plane of data acquisition.

Reformatted CT image planes that lie perpendicular to the original plane of acquisition may be described as: a. oblique b. orthogonal c. obtuse d. orthographic

B. the spleen is absent The gallbladder is present. A Whipple procedure consists of removal of the gallbladder (along with the distal portion of the stomach, the distal portion of the common bile duct, the head of the pancreas, the duodenum, and the proximal jejunum). The high-density objects on the anterior chest wall are breast shields, intended to reduce radiation to the breast.

Regarding this CT slice, which is a true statement? A) the gallbladder is absent B) the spleen is absent C) the patient has breast implants D) the patient had a Whipple procedure

C. rotate-rotate Ring artifacts are associated with the use of third-generation CT scanners. Both the x-ray tube and the detector array rotate around the patient with third-generation scanners. A malfunctioning detector or series of detectors in a third-generation CT scanner causes a ring artifact to appear on the image as a result of the rotational nature of the detector array.

Ring artifacts on the CT image are associated with which of the following tube-detector geometries? a. rotate-nutate b. rotate-stationary c. rotate-rotate d. translate-rotate

C. 1 and 2 only -- diabetes and advanced age Risk factors for contrast-induced nephrotoxicity: preexisting compromise of renal function, dehydration, diabetes, myeloma, advanced age and cardiovascular disease.

Risk factors for contrast-induced nephrotoxicity (CIN) include: 1. diabetes 2. advanced age 3. hematuria a. 1 only b. 3 only c. 1 and 2 only d. 1, 2 and 3

B. 3 only -- dehydration Risk factors for contrast-induced nephrotoxicity (CIN) include preexisting renal function compromise, dehydration, diabetes, myeloma, advanced age, and cardiovascular disease.

Risk factors for contrast-induced nephrotoxicity (CIN) include: 1. pheochromocytoma 2. allergy to shellfish 3. dehydration a. 2 only b. 3 only c. 1 and 2 only d. 1, 2 and 3

D. lung cancer may metastasize to the adrenal glands In many institutions the protocol for a thoracic CT extends to the adrenal glands when patients have a history of lung cancer.

Scanning to the adrenal glands in a CT study of the thorax should be performed because: A) it ensures that the technologist has scanned the entire lung field B) kidney function can be assessed C) the adrenal glands are part of the respiratory system D) lung cancer may metastasize to the adrenal glands

A. temporal resolution Controlling factors of a CT system's temporal resolution: gantry rotation speed and reconstruction method. Temporal resolution can be improved by segmenting the data acquisition process into separate components of smaller rotation angles. Single-segment or half-scan acquisitions reconstruct data obtained from half of the rotation time. For example, this type of segmentation would yield a 250-msec temporal resolution from a 0.5-sec gantry rotation time.

Segmenting the data acquisition process into separate components of smaller rotation angles may improve which of the following components of CT image quality? a. temporal resolution b. longitudinal spatial resolution c. contrast resolution d. in-plane spatial resolution

Less than 50

Shock is indicated when the diastolic pressure is:

B. 0 to +20 Hounsfield units Simple cysts contain primarily water and therefore exhibit CT numbers ranging from approximately 0 (or slightly below) to +20 Hounsfield units.

Simple cysts of the kidney have average attenuation values in the range of: a. -40 to 0 Hounsfield units b. 0 to +20 Hounsfield units c. +30 to +50 Hounsfield units d. above +60 Hounsfield units

B) X-ray tube and detector acquisition system ***The purpose of slip-ring technology was to first eliminate the cable needed to transmit voltage from the generator to the gantry.***

Slip rings are employed in helical CT to conduct high voltage power to what components in the gantry? A) X-ray tube and couch B) X-ray tube and detector acquisition system C) Cooling and detector acquisition system D) Generator and the cooling system

Equalization

Smoothing provides:

Are caused when a large dose of high-LET radiation is received by a large area of the body.

Somatic effects of radiation:

A. using a line/pairs phantom A typical line/pairs phantom contains groups of lead strips having different strip width and spacing. In each group, the lead width is equal to the lead spacing. The spatial resolution is given as the maximum number of visible line pairs (lead strip and space) per millimeter.

Spatial resolution can be calculated fromthe analysis of the spread ofinformation within the system using the modulation transfer function. What is a simpler and more direct way to measure spatial resolution? A) using a line/pairs phantom B) using a water phantom C) using the readout form the scanner's display D) calculating it from the radiation dose delivered

D. graininess Statistical noise = quantum noise/mottle Caused by an insufficient number of photons being detected, creating graininess on the CT image.

Statistical noise appears as _________ on a CT image. a. decreased contrast b. increased brightness c. concentric circles d. graininess

B. how rapidly the data are acquired by the CT system In the context of image quality, the word "temporal" refers to the characteristic of being limited by time and *relates to a scanner's data acquisition speed.* The temporal resolution of a system is typically reported in milliseconds.

Temporal resolution refers to... A) the spatial resolution measurable in images of the temporal bone B) how rapidly the data are acquired by the CT system C) the ability to differentiate between objects with very similar densities as their backgrounds D) the level of noise apparent on an image

D. volume rendering Adjusts the opacity of voxels included in the 3D model according to their tissue characteristics. Unlike the thresholding concept used for shaded-surface display (SSD), or SURFACE RENDERING, volume rendering does not exclude voxels, but instead alters their appearance so that the 3D model contains the entire volume data set.

The 3D CT technique that includes all of the acquired voxel information in the reconstructed model with adjustments to its opacity is termed: a. surface rendering b. maximum intensity projection (MIP) c. curved multiplanar reformation d. volume rendering

C. contrast and brightness As a form of grayscale mapping, the window determines the pixels' shades of gray based on their CT numbers.

The CT "window" controls the ____________ of the CT image as it appears in the viewer. a. density and detail b. spatial and contrast resolution c. contrast and brightness d. attenuation coefficient and Hounsfield value

A) traumatic type 4 spondylolisthesis

The CT sagittal image in the figure demonstrates a greater than 75% slipping of C6 over C7. Which of the following pathological conditions does this represent? a) traumatic type 4 spondylolisthesis b) traumatic type 2 spondylolisthesis c) spondylosis d) traumatic type 1 spondylolisthesis

D. beam divergence The beam divergence inherent to the cone beam geometry of MDCT can cause difficulty during the image reconstruction process. Specific cone beam algorithms such as the Feldkamp-Davis-Kress (FDK) and advanced single-slice rebinning (ASSR) algorithms attempt to overcome the issues resulting from the divergent path of the x-ray beam from the tube and to the widened detector array.

The Feldkamp-Davis-Kress (FDK) algorithm may be applied to MDCT acquisition data to overcome image artifacts from: a. patient motion b. beam hardening c. partial volume averaging d. beam divergence

A) 2,000

The HRCT of the chest in the figure demonstrates increased opacification in the mid lung and atelectasis in the middle lobe and lingual. Which of the following window widths was most likely used to display the image? a) 2,000 b) -800 c) 450 d) 80

A. tarsals and metatarsals Lisfranc joint: the articulation between the tarsals and metatarsals; it can be the site of complex fractures and dislocations of the foot.

The Lisfranc joint is the articulation between the: a. tarsals and metatarsals b. tarsals and lower leg c. metatarsals and phalanges d. proximal and middle phalanges

B) Noise ***the noise value can be measured by means of the standard deviation indicating the amount of variance among pixel values in a desired ROI***

The QA test for standard deviation employs a water filled 20-cm phantom with an ROI placed in the center of the images, what is the general purpose of this QA test? A) Spatial resolution B) Noise C) Contrast resolution D) Temporal resolution

D. D. L4 At the level of the fourth lumbar vertebra, the aorta bifurcates into the left and right common iliac arteries.

The abdominal aorta bifurcates at the level of: a. T10 b. T12 c. L2 d. L4

B) L4 ***Common Liliac arteries are bilateral arising from the abdominal aorta at the leve of L4. They divide laterally as the enter the pelvis***

The abdominal aorta divides into the right and left common iliac arteries at what lumbar vertebrae? A) L2 B) L4 C) S1 D) S2

A. Morison pouch The common name for the hepatorenal recess, which is indicated by number 7 in the figure. This space between the liver and right kidney is a common location for the abnormal collection of fluid known as ascites.

The abdominal space indicated by number 7 in the figure is commonly referred to as: a. Morison pouch b. cul-de-sac c. splenorenal recess d. subphrenic space

C. temporal resolution The stop-motion capability of a CT system is referred to as TEMPORAL RESOLUTION. Temporal resolution quantifies the CT system's ability to freeze motion and provide an image free of blurring.

The ability of a MDCT system to freeze motion and provide an image free of blurring is called: a. in-plane spatial resolution b. longitudinal spatial resolution c. temporal resolution d. contrast resolution

D. pleural effusion Pleural effusions are commonly seen in the posterior portion of the lung field on images obtained with the patient in a supine position on the CT table. Differentiation between pleural effusion and pleural thickening is made when region of interest (ROI) measurements reveal fluid with density readings at or slightly above zero. Pleural effusion may be caused by multiple pathologic processes, including infection, neoplasm, and congestive heart failure.

The abnormal density located in the posterior portion of the left lung field on the figure has an average attenuation value of +5.0 Hounsfield units. This density most likely represents: a. pneumothorax b. hemothorax c. atelectasis d. pleural effusion

B. vacuum phenomenon Small amounts of gas may appear in the areas of degenerated intervertebral discs. The accumulation of gases such as nitrogen occurs as a by-product of the physical breakdown of the disc material.

The accumulation of gas within a degenerating intervertebral disc is called the: a. aeration effect b. vacuum phenomenon c. oxygen saturation point d. carbonization sign

B. cine CT Cine CT acquisition involved multiple axial scans obtained at a single anatomic level over a predetermined period. Clinical applications of cine CT acquisition include contrast bolus tracking; dynamic imaging of physiologic processes such as respiration, swallowing and the cardiac cycle; and for CT perfusion studies.

The acquisition of a series of CT images at a single anatomic location over a set period is referred to as: a. ultrafast CT b. cine CT c. conventional CT d. temporal CT

C. The radiation dose to the patient during a CT scan CT Dose Index: used to quantify the radiation dose received by the patient during a CT scan. It involves the use of an ionization chamber to accurately measure radiation exposure for a given set of technical factors.

The acronym CTDI is used to describe which of the following? A) a specialized CT imaging technique used to measure bone mineral density B) a quality control test that measures the accuracy of the laser lighting system C) the radiation dose to the patient during a CT scan D) a high-speed CT scanner used for cardiac imaging

B. L1 - L2 The spinal cord extends to the lower margin of the first or upper margin of the second lumbar vertebra. At this level, the spinal cord tapers to a point known as the CONUS MEDULLARIS.

The adult spinal cord ends at what vertebral level? a. T11 - T12 B. L1 - L2 C. L3 - L4 D. Superior portion of the coccyx

Linear energy transfer

The amount of of radiation deposited per unit length of tissue transversed by incoming photons is called:

Doubling dose

The amount of radiation that causes the number of genetic mutations in a population to double is called the:

B. Kerma May be used to describe absorbed dose. Air Kerma is the amount of radiation absorbed in a quantity of air. Kerma is an acronym for: Kinetic Energy Released per unit MAss.

The amount of x-ray energy absorbed in a quantity of air is termed: a. dose profile b. kerma c. becquerel d. multiple scan average dose (MSAD)

A. coronal plane The image in the figure is a multiplanar reformation (MPR) of the wrist in the coronal plane. MPR images are particularly helpful for trauma indications, such as fracture of the distal radius, labeled number 3 in the figure.

The anatomic plane of the image in the figure can be best described as the: a. coronal plane b. sagittal plane c. axial plane d. oblique axial plane

A. coronal plane The image is a multiplanar reformation (MPR) of the right shoulder in the coronal plane. This perspective demonstrates the relationship between the humeral head and they glenoid fossa.

The anatomic plane of the image in the figure can be best described as the: a. coronal plane b. sagittal plane c. axial plane d. oblique axial plane

D. distal lower extremity The angiographic assessment of the peripheral arterial tree form the renal arteries through the feet is commonly referred to as a CT runoff.

The angiographic assessment known as a CT runoff evaluates the peripheral arterial tree from the renal arteries through the: a. aortic arch b. superior mesenteric artery c. aortic bifurcation d. distal lower extremity

0.1 REM

The annual effective dose limit for the general public, assuming frequent exposure is:

B. focal spot The focal spot is also called the ANODE TARGET.

The area of the anode where the electrons strike and the x-ray beam is produced is the A) filament B) focal spot C) generator D) detector

C. pulsation artifact Pulsation artifacts are unique to cardiac imaging and CTA procedures of the mediastinum. They appear as a type of stair-step artifact and result from the diastolic heart motion occurring during relatively long exposure times. Improving the temporal resolution of the scan and ECG gating are methods employed to reduce pulsation artifact.

The areas of image quality degradation indicated by number 3 on the figure are an example of: a. aliasing artifact b windmill artifact c pulsastion artifact d. beam-hardening artifact

B) inconstancies of the projected data on the reconstructed image due to the high density of the metal

The artifact depicted in the figure arises due to which of the following reasons? a) patient involuntary motion b) inconstancies of the projected data on the reconstructed image due to the high density of the metal c) object is partially lying within an individual slice d) insufficient projection sampling

B. Out-of-field artifact The artifact present in the figure most likely represents an out-of-field artifact. This relatively large patient was incorrectly positioned, and a portion of the anatomy lies outside the scanned field of view. This improperly centered anatomy interferes with the reference detectors, thus causing a streak artifact near the unscanned area.

The artifact present on the lateral borders of the image in the figure most likely represents which of the following? a. edge gradient b. out-of-field artifact c. tube arcing d. beam hardening

D) The acquisition geometry ***The problem in CT is to determine the attenuation of the tissue and use the information to reconstructed an image of the slice of tissue.***

The attenuation of a beam of radiation depends on several factors. Which of the following factors does NOT influence attenuation? A) The atomic number of the material B) The electrons per gram of tissue C) The energy of the radiation beam D) The acquisition geometry

B. lipoma The density of the mass in question indicates that it consists of fatty tissue. A LIPOMA is a benign mass consisting of fat cells. An ANGIOMYOLIPOMA, a common benign mass found in the kidney, consists of muscle cells, blood vessels, and fat. The average CT value range for fat is -50 to -100 Hounsfield units.

The average density of a mass within the kidney measure -75 Hounsfield units (HU). The mass is most likely a: a. cyst b. lipoma c. stone d. hydrocele

Mean marrow dose

The average dose to active bone marrow as an indicator of somatic effects on a population is called:

B. increasing filtration Increases in filtration causes a greater amount of low-energy x-ray photons to be absorbed, thereby increasing the average photon energy of the beam. Higher mAs values increase the intensity of the beam but do not affect average photon energy.

The average photon energy of the CT x-ray beam can be increased by: a. increasing mAs b. increasing filtration c. increasing collimation d. all of the above

C. 100 keV The average photon energy of the primary beam used in CT is approximately 70 keV. The photon energy of any radiographic primary beam is typically 30-40% of the applied kilovoltage. The average photon energy of the CT beam is increased through beam filtration.

The average photon energy of the primary beam of a CT scanner operating at a tube potential of 120 kVp is approximately: a. 50 keV b. 70 keV c. 100 keV d. 120 keV

A. acoustic neuroma The eighth cranial nerve is the vestibulocochlear nerve responsible for both hearing and balance. VESTIBULAR SCHWANNOMA may also be referred to as ACOUSTIC NEUROMA. This type of mass most often involves the vestibular portion of the nerve bundle, with deleterious effects on balance and equilibrium.

The benign mass of the eighth cranial nerve known as a VESTIBULAR SCHWANNOMA may also be termed a(n): a. acoustic neuroma b. hypoglossal adenoma c. olfactory neuroblastoma d. optic nerve glioma

C. retrospectively reconstruct the image using a small display field of view (DFOV) Portions of the CT image may be enlarged on the display screen by either of two methods. The image may be magnified to offer the viewer a closer, enlarged look at a specific area. A CT image may also be "targeted" through a decrease in the display field of view (DFOV) size. A targeted image places the area of interest over the entire display matrix, providing greater resolution and more detail than a magnified image.

The best method for targeting the sacrum in the figure for detailed examination would be to: A) magnify the image 2x B) rescan the patient using a small scan field of view (SFOV) C) retrospectively reconstruct the image using a small display field of view (DFOV) D) decrease the matrix dimension

A. pixel The bit depth of a digital imaging system defines the number of information bits obtained within each pixel. This parameter ultimately controls the total range of CT values that may be assigned to a given pixel. A bit depth of 12 results in 4096 possible CT values (2^12 = 4096).

The bit depth of a digital imaging system defines the number of information bits contained within each: a. pixel b matrix c. axial image d. multiplanar reformation (MPR)

D. contrast agent volume Bolus duration, or the time required to inject a specific volume of contrast agent, may be calculated as the product of injection flow rate and total contrast volume.

The bolus duration of any IV contrast agent administration can be calculated as the product of injection flow rate and: a. scan time b. osmolality c. cardiac output d. contrast agent volume

C) T5 ***The trachea bifurcates into the left and right main stem bronchi at an area called the carina.***

The carina of the trachea is where the trachea bifurcates into the right and left main stem bronchi, what is the vertebral location of this bifurcation? A) C6 B) T1 C) T5 D) T12

C) T5 ***The treacha bifurcates into the left and right main stem bronchi at an area called the carina.***

The carina of the trachea is where the trachea bifurcates into the right and left main stem bronchi, what is the vertebral location of this bifurcation? A) C6 B) T1 C) T5 D) T12

Proximal row (scaphoid, lunate triquetral, pisiform) Distal row (trapezium, trapezoid, capitate, hamate)

The carpal bones are arranged in two rows as follows:

C. 1 and 3 only -- the allergic history, age and physical condition of the patient Patients may be assigned to a higher-risk group for an adverse reaction to iodinated contrast material on the basis of several factors, including previous allergic history and physical conditions. Patients considered to be at high risk for adverse reactions should be administered nonionic contrast material. Iso-osmolar contrast material is an alternative choice that may also reduce the risk of adverse reactions.

The choice between ionic and nonionic contrast media should be based on: 1. the allergic history of the patient 2. the cost of the contrast material 3. the age and physical condition of the patient a. 1 only b. 1 and 2 only c. 1 and 3 only d. 1, 2 and 3

C. perform a region of interest (ROI) measurement ROI measurements may be made by superimposing a cursor over an area and instructing the computer to average the CT numbers included within the region.

The circle shown on the image in the figure is used to: A) magnify a portion of the image B) localize an area for percutaneous biopsy C) perform a region of interest (ROI) measurement D) produce a multiplanar reformation (MPR) image

A) Common hepatic duct and the cystic duct ***The common bile duct transports bile from the gall bladder via the cystic duct and the liver via the hepatic duct to the duodenum***

The common bile duct is formed by the union of which two duct? A) Common hepatic duct and the cystic duct B) Intrahepatic duct and the pancreatic duct C) Cystic duct and the left intrahepatic ducts D) Porta hepatic duct and the cystic duct

B. array processor The primary data-processing component of the CT system. It is responsible for receiving scan data from the host computer, performing all of the major processing of the CT image, and returning the reconstructed image to the storage memory of the host computer.

The component of the CT computer system responsible for the data processing of image reconstruction is the: a. pipeline memory b. array processor c. hard disk drive d. RAM microprocessor

D. array processor A specialized component of the CT computer system. It is capable of performing the massive calculations required for CT image reconstructions.

The component of the CT scanner responsible for the mathematical calculations of the image reconstruction process is the: a. data acquisition system (DAS) b. analog-to-digital converter (ADC) c. digital-to-analog converter (DAC) d. array processor

B. claudication Refers to the condition of intermittent cramping pain in the legs resulting from poor circulation.

The condition of intermittent cramping pain in the legs due to poor circulation is called: a. varices b. claudication c. stenosis d. thrombosis

A. Hypoxemia Hypoxemia: low concentration of oxygen in blood Hypotension: low blood pressure hypoxia: low oxygen in tissue hyponatremia: low sodium in blood

The condition whereby the patient suffers from a low concentration of oxygen is called: a. hypoxemia b. hypotension c. hypoxia d. hyponatremia

C. generation

The configuration of the x-ray tube to the detectors determines a scanner's a. heat dissipation rate b. detector efficiency c. generation d. speed

C. slip rings Slip-ring technology eliminates the need for cables and allows for continuous gantry rotation by utilizing a system of contact brushes that supply electricity to power the system, enabling the passage of transmissions data to the computer system.

The continuous gantry rotation required by helical CT acquisition is made possible by the application of: a. electromagnetic bushings b. titanium bearings c. slip rings d. oil cooled couplings

A. focal spot size Focal spot size is a geometric factor that influence the spatial resolution of a CT scanner. Contrast resolution of a CT scanner depends on several factors, including section width, algorithm section, detector sensitivity and noise.

The contrast resolution of a CT scanner is not related to which of the following? a. focal spot size b. section width c. reconstruction algorithm d. signal-to-noise ratio

22 REM

The cumulative occupational exposure for a 22-year-old radiographer is:

D. 1, 2 and 3 -- measuring transmitted intensity, converging the transmission data into a digital signal and sending the digital information to the computer for processing The data acquisition system (DAS) of a CT scanner consists of the detector array, the analog-to-digital converter, and a transmission device used to send the converted digital information to the computer for image reconstruction.

The data acquisition system (DAS) of a CT scanner is responsible for: 1. measuring transmitted intensity 2. converging the transmission data into a digital signal 3. sending the digital information to the computer for processing a. 1 only b. 2 only c. 1 and 3 only d. 1, 2 and 3

B. Corticomedullary Beginning 30 - 40 seconds after the initiation of contrast agent administration, the corticomedullary phase demonstrates optimal enhancement of the renal cortex with maximum differentiation of the cortex from the renal medulla.

The data for this reformatted image of the abdomen was most likely acquired in which of the following renal enhancement phases? a. precontrast b. corticomedullary c. nephrographic d. excretory

A. spondylolisthesis Refers to the forward "slipping" of an upper vertebral body over the lower as a result of degenerative changes of the facet joints.

The degenerative pathologic process involving the forward slipping of an upper vertebral body over a lower vertebral body is called: a. spondylolisthesis b. spondylolysis c. herniated nucleus pulposus d. spinal stenosis

C. water The CT number for water has an average value of zero

The density measurement performed in the figure yielded an average CT number of zero. This area consists of: a. fat b. blood c. water d. air

D. gantry The x-ray tube and data acquisition system (DAS) are house within the gantry of a CT scanner. The gantry aperture is the circular opening through which the patient moves during scanning.

The device constructed to house the x-ray tube and data acquisition system (DAS) for a CT scanner is called the: a. central processing unit (CPU) b. generator c. array processor d. gantry

Rectifier

The device in the x-ray circuit that changes AC to DC is the:

B. injection rate and scan delay

The difference among the bolus phase, the nonequilibrium phase, and the equilibrium phase of contrast enhancement is primarily determinedby the: A) brand of iodinated contrast agent that is used B) injection rate and scan delay C) type of pathologic disease present D) film processing time

C. beam hardening Occurs as low-energy x-ray photons are absorbed while the beam passes through the patient. The average photon energy of the beam increases along the path and may result in a loss of system uniformity.

The differences in Hounsfield value exhibited in the regions of interest (ROI) measured on this CT image of a water-filled phantom are mots likely due to the phenomenon referred to as: a. partial volume averaging b. noise c. beam hardening d. detector drift

C. histogram

The display function that creates a bar graph to show how frequently a range of CT numbers occurs within a specified ROI is called a: A) gray scale B) localizer image C) histogram D) beam attenuation chart

C. section interval Describes the spacing between two adjacent CT images. It is measured as the distance between the center of one section and the center of the adjacent section.

The distance between the centers of two adjacent reconstructed CT images is termed the: a. section width b. interpolation degree c. section interval d. sampling rate

B. focus-to-isocenter distance Describes the distance from the x-ray source (CT tube) to the center of the gantry opening, where the patient should be positioned. A structural decrease in this distance would result in a decrease in patient dose, based upon the inverse square law.

The distance between the x-ray source (CT tube) and the center of the gantry may be referred to as the: a. focus-to-detector distance b. focus--to-isocenter distance c. focus-to-image receptor distance d. source-to-image receptor distance

C. the absorbed dose is greater in the patient with a smaller body habitus It is important to remember that CTDIvol and dose length product (DLP) do not account for patient size and so overestimate the radiation dose to the larger patient and underestimate the dose to the smaller patient. For a given DLP, absorbed dose will be less in the larger patient, as the radiation is distributed over a larger mass than in the smaller patient. This concept forms the basis of why customized, size-based protocols are an important component of dose-reduction efforts.

The dose length product (DLP) is identical for a given CT scan acquired on two patients, each with a significantly different body habitus Which of the following statements is correct? A) the absorbed dose is identical in each patient B) the absorbed dose is greater in the patient with a larger body habitus C) the absorbed dose is greater in the patient with a smaller body habitus D) dose length product is unrelated to absorbed dose in the patient

B. 1 and 2 only -- MSAD X slice width (cm) X number of slices in scan volume and CTDIvol X scan length (cm) Dose length product (DLP) is used to estimate the radiation dose to the patient. During helical scanning, it can be calculated as the product of the CTDIvol and the acquisition (scan) length in centimeters. For an axial (nonhelical) acquisition, DLP is calculated as the product of the multiple scan average dose (MSAD), the section width (cm), and the total number of acquired sections.

The dose length product (DLP) of a given CT acquisition may be calculated according to which of the following? 1. MSAD x slice width (cm) x number of slices in scan volume 2. CTDIvol x scan length (cm) 3. pitch x CTDIw a. 3 only b. 1 and 2 only c. 1 and 3 only d. 1, 2 and 3

B. 1 and 2 only -- tube current (milliamperage) and tube potential (applied kilovoltage) Modern CT systems employ automated systems controlling both tube milliamperage (mA) and applied potential (kVp) for the purpose of patient dose reduction. These systems act as a means of automatic exposure control (AEC) for CT. The focus-to-isocenter distance of a given CT system is determined in the system design and cannot be adjusted. A manufactured decrease in focus-to-isocenter distance will result in an increase in patient dose, according to the inverse square law.

The dose modulation capabilities of a CT scanner may include automatic control of: 1. tube current (milliamperage) 2. tube potential (applied kilovoltage) 3. focus-to-isocenter distance (centimeters) a. 1 only b. 1 and 2 only c. 2 and 3 only d. 1, 2 and 3

A) MSAD *** The value of MSAD is the average value of all profiles over one scan interval in the central portion of the profile***

The dose to the central slice plus the dose from the scatter into nearby slices equals which of the following doe descriptors? A) MSAD B) SV C) mAs Setting D) R

C. CTDIvol CTDIvol is used to approximate the dose from a helical scan. It corresponds to the axially acquired CTDIw divided by the helical pitch. Whereas CTDIw approximates the dose along the x and y axes of the acquired CT image, CTDIvol also includes the dose along the z axis of the scan acquisition. AS PITCH INCREASES, THE DOSE PER SCAN, OR CTDIvol, DECREASES.

The dosimetry index used to approximate the radiation dose for CT sections acquired during a helical scan is called the: a. CTDIw b. effective dose c. CTDIvol d. equivalent dose

D. Corticosteroid SOLU-CORTEF is a brand name for hydrocortisone, which is a type of corticosteroid. Corticosteroids may be used during the treatment of anaphylactoid reactions to iodinated contrast materials.

The drug SOLU-CORTEF may be classified as which of the following? a. anticholinergic b. bronchodilator c. antihistamine d. corticosteroid

5000 mRem per year

The effective dose limit for radiographers is:

A. binning Refers to the electronic combination of signal from adjaccent detectors to form a reconstructed slice that is thicker than the individual detector width. For example, transmission data from four adjacent 1.25-mm detectors is combined to produce a CT section 5.0 mm thick.

The electronic combination of signals from adjacent detectors to form a CT image is called: a. binning c. interpolation c. z-filtering d. partial volume averaging

A. first trimester

The embryo or fetus is most sensitive to ionizing radiation during which portion of gestation? A) first trimester B) second trimester C) third trimester D) the fetus is equally radiosensitive during all trimesters

B. 1 and 3 only -- thin-sections precontrast acquisition through the adrenals and 10-15 minute delayed precontrast acquisition through the adrenals Differentiation between a metastatic lesion and a benign adenoma of the adrenal masses can be accomplished by calculating the percentage of enhancement washout after a 10-15 minute delay. Precontrast, portal venous phase, and delayed acquisitions are obtained. On delayed postcontrast imaging, metastatic lesions of the adrenal gland remain enhanced longer than adrenal adenomas.

The enhancement washout technique for CT characterization of adrenal masses include: 1. thin-section precontrast acquisition through the adrenals 2. arterial phase postcontrast acquisition through the adrenals 3. 10-15 minute delayed precontrast acquisition through the adrenals a. 1 and 2 only b. 1 and 3 only c. 2 and 3 only d. 1, 2 and 3

C. urothelium Lines most of the urinary tract, including the renal pelvis, ureters and bladder. The transitional portion of the urothelium can be the site of transitional cell carcinoma (TCC), a common malignancy of the bladder, ureters and kidneys.

The epithelial lining of the urinary tract is called the: a. omentum b. haustrum c. urothelium d. pyelocalyx

500 mRem during gestation

The equivalent dose limit for an embryo or fetus is:

D. extravasation Sometimes referred to as an infiltration.

The escape of contrast material from a needle or blood vessel into the subcutaneous tissues is called: a. infusion b. extraversion c. influxation d. extravasation

B. Maximum intensity projection (MIP)

The figure above demonstrates which of the following 3D techniques? a) volume rendered (VRT) b) maximum intensity projection (MIP) c) minimum intensity projection (minIP) d) mulitplanar reformation (MPR)

A) renal cell carcinoma Demonstrates low-density areas within the mass consistent with necrosis. It is not uncommon for the renal cell tumors to contain internal hemorrhage, cystic necrosis, and coarse and irregular calcifications.

The figure demonstrates a large solid mass in the posterior aspect of the right kidney containing low-density area within the mass consistent with necrosis. What pathological condition does this image demonstrate? a) renal cell carcinoma b) polycystic kidney disease c) renal calculi d) horseshoe kidney

D) sarcoidosis A systemic disease primarily affecting the lungs. High-resolution CT imaging is the best modality for differentiating this disease, as it is interpreted as hilar and mediastinal lymphadenopathy and multiple round mass-like consolidations.

The figure demonstrates increasing opacification involving the mid-lung with marginal lung nodules and atelectasis. Which of the following disease does this represent? a) pulmonary embolism b) pulmonary stenosis c) pulmonary thrombosis d) sarcoidosis

C. annulus fibrosus The outer portion of an intervertebral disc

The firm, outer portion of each intervertebral disc is called the: a. nucleus pulposus b. nucleus prepositus c. annulus fibrosus d. annulus stapedius

B. noise Noise appears on the CT image as an inaccurate CT number. The noise of a CT scanner may be measured by scanning a homogeneous object such as a water phantom. Fluctuations in CT number from pixel to pixel indicate the presence of noise.

The fluctuation of CT numbers in an image of uniform, homogeneous material may occur because of: a. linearity b. noise c. aliasing d. partial volume effect

B. thrombosis A thrombosis is a blood clot. If the thrombus detaches from its original site it is referred to as an embolus. If the embolus occludes a vessel in the brain and disrupts the blood supply, it results in an ischemic stroke. A myocardial infarction (heart attack) occurs when the heart does not receive an adequate blood supply.

The formation, development or existence of a clot within the vascular system is referred to as a(n): A) embolus B) thrombosis C) ischemic stroke D) myocardial infarction

A. attenuation coefficient The attenuation coefficient is a number derived from a specific strength beam as it travels through a specific substance. For example, with a CT scanner that is operating in the typical range of 120 kVp, the linear attenuation coefficient for water is approximately 0.18 cm^-1. This means that when the beam passes through 1.0 cm of water, about 18% of the photons are either absorbed or scattered.

The fraction of a beam of x-rays that is absorbed or scattered per unit thickness of the absorber is referred to as the A) attenuation coefficient B) contrast transfer function C) spatial resolution D) modulation transfer function

A. spatial resolution The full width at half maximum (FWHM) is used to illustrate the spatial resolution of a CT scanner. The spatial resolution in CT scanners can be measured by studying the amount of blurring that occurs around a point with the CT image. This blurring/unsharpness in a CT image, known as POINT SPREAD FUNCTION (PSF), may be graphically represented. When PSF/blurring/unsharpness is graphed, spatial resolution can be quantified by measuring the graph at half its maximum value. This measurement is called FULL WIDTH AT HALF MAXIMUM (FWHM).

The full width at half maximum (FWHM) of a CT scanner is used to describe: a. spatial resolution b. contrast resolution c. noise d. calibration accuracy

D. amount of interspace material between detectors The geometric efficiency of a CT detector is influenced primarily by the amount of interspace material necessary between adjacent detectors. Although valuable in limiting interference (crosstalk) between adjacent detectors, the interspace material reduces geometric efficiency by absorbing transmitted x-ray energy.

The geometric efficiency of a CT detector is influenced primarily by the: A) atomic number of the detector material B) size of the detector element C) size of the tube filament D) amount of interspace material between detectors

B. longitudinal spatial resolution Is usually qualified by the extent of broadening that occurs to the slice sensitivity profile (SSP) during volumetric acquisition. The section width (slice) for a volumetric acquisition may be graphically represented as an SSP.

The graph in the figure is used to evaluate which of the following components of CT image quality? A) in-plane spatial resolution B) longitudinal spatial resolution C) temporal resolution D) contrast resolution

Radon

The greatest source of natural background radiation to humans is:

Amplitude

The height of a sine wave is called:

Step-up

The high-voltage section of the x-ray circuit makes use of what type of transformer?

Ionic iodinated contrast media

The highest incidence of contrast agent reactions occurs with the use of:

B. low-contrast resolution

The image features a series of cylinders with different diameters, all at 0.6% (6HU) difference from the background material. This phantom is used to measure a. slice thickness accuracy b. low-contrast resolution c. high-contrast resolution d. CT number accuracy

C) tibial plateau fracture

The image in the figure depicts which of the following pathological conditions? a) bone contusion b) lateral collateral ligament tear c) tibial plateau fracture d) baker cyst

B. arterial phase Dense contrast opacification of the aorta and pulmonary arteries indicates that this image was obtained during the arterial phase, most likely within the first 20 - 30 seconds after bolus injection of contrast material.

The image in the figure was most likely acquired during which of the following phases of contrast enhancement? a. precontrast b. arterial phase c. venous phase d. delayed phase

C. +50 The level chosen for a given window setting should correspond to the average density value of the tissue(s) of interest areas of soft tissue, such as the brain, are often displayed at window levels of approximately +50 HU.

The image in the figure was most likely displayed in a window with a level of: a. -150 b. 0 c. +50 d. +400

B. 400 The width of a window used to display a CT image of the abdomen should be within the range of 350 - 600 HU. This range allows for excellent visualization of the soft tissue structures.

The image in the figure was most likely displayed with the use of which of the following window widths? a. 70 b. 400 c. 1300 d. 3800

D. centering the patient within the SFOV The out-of-field artifact present in this image could be easily reduced by properly centering the patient within the scan field of view.

The image in the figure was produced using the following parameters: large (full) scan field of view (SFOV); maximum display field of view (DFOV) (48 cm); 200 mA, 120 kVp; soft tissue algorithm. Which of the following technical adjustments would serve to reduce the artifact present? A) switching to a detail or bone algorithm B) using a smaller SFOV C) increasing the mA to 240 and the kVp to 140 D) centering the patient within the SFOV

C. axial plane The axial plane is parallel to the foot's plantar surface. At approximately 20-30 degrees from the axial plane, the oblique axial plane is parallel to the metatarsals.

The imaging plane that is parallel to the foot's plantar surface is called the: a. coronal plane b. sagittal plane c. axial plane d. oblique axial plane

A. quality control Refers to the implementation of corrective actions to improve any identified performance inadequacies of the CT system found through quality assurance procedures.

The implementation of corrective actions to improve any identified performance inadequacies of the CT system is referred to as: a. quality control b. uniformity c. quality assurance d. linearity

C. threshold setting Used to include and exclude information during the 3D reconstruction of a CT scan. For example, a high threshold (+150 HU) may be set to produce a 3D model of a bony structure. This threshold eliminates any density value betow +150 HU from the data set. The reconstructed 3D model would include only bone tissue or any other substance with a Hounsfield value above +150.

The information included during the 3D reconstruction of a CT scan is controlled by the: a. algorithm b. window setting c. threshold setting d. grayscale map

B. 3.0 mm Al The CT x-ray beam is heavily filtered to remove low energy x-ray photons, hardening the beam to reduce patient dose and to improve image quality. The inherent structure of the CT x-ray tube (house, cooling oil, etc.) typically amounts to 3.0 mm of Aluminum equivalent filtration.

The inherent filtration of the x-ray tube used in a CT system typically amounts to an equivalent thickness of: a. 0.5 mm Al b. 3.0 mm Al c. 10.0 mm Al d. 15. 0 mm Al

C. transmitted intensity The amount of energy that passes through the patient onto a detector.

The intensity of the x-ray beam after it passes through an object to a detector is called the: a. incident intensity b. ray c. transmitted intensity d. primary beam

D. photoelectric effect Photoelectric (effect) absorption occurs when the energy of an incoming (incident) x-ray photon is completely absorbed through the ionization of an inner-shell electron of the target atom.

The interaction between radiation and matter that results in the complete attenuation of an x-ray photon due to ionization of a target atom's inner shell electron is termed: a. photodisintegration b. Compton scatter c. Bremsstrahlung d. photoelectric effect

A. prothrombin time (PT) the international normalized ratio (INR) is calculated to standardize prothrombin time (PT) results. The INR compares a patient's PT with a control sample for a more accurate result.

The international normalized ratio (INR) is calculated to standardize which of the following laboratory values? a. prothrombin time (PT) b. partial thromboplastin time (PTT) c. blood urea nitrogen (BUN) d. glomerular filtration rate (GFR)

B. 1 and 2 only Intrathecal administration of an iodinated contrast agent allows for greater visualization of the spinal cord and nerve roots after its introduction into the subarachnoid space.

The intrathecal administration of an iodinated contrast agent in the patient in the figure would allow for greater visualization of the: 1. spinal cord 2. nerve roots 3. anulus fibrosis a. 1 only b. 1 and 2 only c. 1 and 3 only d. 1, 2 and 3

D. Gerota's fascia The kidneys are located in a portion of the retroperitoneum known as the perirenal space. The kidneys are held in place by fibrous connective tissue commonly referred to as GEROTA'S FASCIA.

The kidneys are located in the retroperitoneum and are bound by a band of fibrous connective tissue called: a. Cooper ligament b. Camper fascia c. linea alba d. Gerota's fascia

B. 1,000 The latency time for cancer induction at the dose ranges used in CT is 10 to 30 years. However, because of their younger age, children have more time to express a cancer, and more time to undergo more CT examinations, than do adults. In addition, because of their youth they have more rapidly dividing cells, which are more radiosensitive.

The lifetime risk of an eventual cancer-related death as a result of pediatric CT is approximately 1 in a. 100 b. 1,000 c. 10,000 d. 100,000

C. 14.6 lp/cm The limiting resolution of a particular CT scan is determined at a point on the graph where the signal frequency corresponding to a particular object has reached 10% (modulation transfer function [MTF] = 0.1).

The limiting resolution of the bone algorithm in the figure is: a. 6.1 lp/cm b. 9.5 lp/cm c. 14.6 lp/cm d. 15.5 lp/cm

C. portal vein The liver has a dual blood supply, receiving 75% of its supply from the portal vein and the remaining 25% from the hepatic artery.

The liver has a dual blood supply and receives 75% of its blood from the: a. hepatic artery b. superior mesenteric artery c. portal vein d. superior mesenteric vein

A) 25% ***The liver has a dual blood supply from the hepatic portal vein and the hepatic arteries. The hepatic arteries supply arterial blood to the liver***

The liver has a dual blood supply. What percent of the hepatic artery supplies blood to the liver? A) 25% B) 35% C) 50% D) 80%

A) 25% ***The liver has a dual blood supply from the hepatic portal vein and the hepatic arteries. The hepatic arteries supply arterial blood to the liver***

The liver has a dual blood supply. What percent of the hepatic artery supplies blood to the liver? A) 25% B) 35% C) 50% D) 80%

B. misregistration An artifact that occurs when a patient suspends respiration at different depths during consecutive scans. Results in the loss of anatomic information.

The loss of anatomic information between contiguous sections due to inconsistent patient breathing is called: a. cupping artifact b. miregistration c. overshoot artifact d. out of field artifact

B) Increases *** Pitch and dose are inversely proportional to each other. As pitch increase the CTDI volume decreases and the pitch decreases. CTDI volume increases***

The mAs for a particular procedure remains constant and the pitch changes from 2 to 1; what change does it have on the dose? A) Decrease B) Increase C) Remains the same D) There is no relationship between pitch and dose

D. star artifact An unwanted by-product of the back-projection method of image reconstruction used in older CT scanners. It is now removed by the process of convolution used in the modern reconstruction method known as FILTERED BACK-PROJECTION.

The major disadvantage of the back-projection method of image reconstruction is the appearance of the: a. partial volume effect b. ring artifact c. Gibb phenomenon d. star artifact

B. greater patient radiation dose The use of a fan or cone-shaped x-ray beam during CT increases the total volume of tissue irradiated, thereby increasing the amount of scatter radiation produced. This change results in a significantly higher patient radiation dose than the "pencil beam" radiation used in older CT scanners.

The major disadvantage of the fan or cone-shaped x-ray beams used in modern CT units in comparison with the "pencil-thin" beams of older units is: a. increased transmission measurements b. greater patient radiation dose c. decreased transmission measurements d. excess tube wear

C. 0.206 At the range of photon energies employed during most CT examinations, water exhibits an approximate attenuation coefficient value of 0.206.

The material within the region of interest (ROI) measurement in the figure has a linear attenuation coefficient of: a 0.0007 b. 0.185 c. 0.206 d. 0.530

D. 1, 2 and 3 -- algorithm, kernel and mathematical filter function The terms ALGORITHM, KERNEL and MATHEMATICAL FILTER FUNCTION may all be used interchangeably in describing the mathematical process used for the complex calculations required during CT image reconstruction.

The mathematical manipulations required during the reconstruction of a CT image are accomplished a(n): 1. algorithm 2. kernel 3. mathematical filter function a. 1 only b. 1 and 2 only c. 1 and 3 only d. 1, 2 and 3

D. z-filtering Multiple-detector row CT systems utilize the process called z-filtering during image reconstruction. Z-filtering allows for thin sections to be reconstructed at any point along the acquired z-axis volume. This interpolation technique utilizes multiple complementary rays beyond those immediately above and below the particular slice plane.

The mathematical process that allows multidetector CT (MDCT) images to be reconstructed at any point along the acquired volume is commonly referred to as: a. iteration b. filtered back-projection c. Fourier reconstruction d. z-filtering

C. interpolation The mathematical process whereby data from tube rootations just above and just below a given slice position are used for image reconstruction. Interpolation allows for the reconstruction of a thin, motion-free image from a volumetric data set acquired from a moving patient.

The mathematical technique allowing the reconstruction of motion-free images from helically acquired CT data is called: a. convolution b. radon transformation c. interpolation d. Fourier reconstruction

B. interpolation A mathematical technique used in the reconstruction process of the spiral CT image. It involves the estimation of an unknown value from information above and below it.

The mathematical technique that involves the estimation of an unknown value from values on either side of it is known as: a. filtering b. interpolation c. convolution d. summation

D. 40.00 mm At a maximum, MDCT cone beam may be collimated to a dimension equal to the entire multirow detector array. In the example, the total dimension of the detector array along the z-axis is 40.00 mm, or 64 x 0.625 mm.

The maximum beam collimation for a MDCT system with a detector array of 64 detectors, each 0.625 mm wide, is: a. 1.25 mm b. 5.00 mm c. 10.00 mm d. 40.00 mm

B. data channels The number of data channels controls the number of sections the scanner can simultaneously acquire with each gantry rotation. A four-slice multidetector CT (MDCT) system has sixten 1.25 mm detectors in its array. However, with only four data channels, the maximum number of sections that may be reconstructed from each gantry rotation is four. The system combines transmission information from the individual detectors through its four data channels, resulting in the acquisition of four sections per gantry rotation.

The maximum number of simultaneous sections a MDCT system can acquire per gantry rotation is controlled by the number of: a. detector rows b. data channels c. focal spots d. x-ray tubes

C. quality ASSURANCE Refers to the measurement of the scanner's performance through quality testing procedures and evaluation of the test results. Quality CONTROL refers to the implementation of corrective actions to improve any identified performance inadequacies of the CT system found through quality assurance procedures.

The measurement of CT system performance through quality testing procedures and evaluation of the test results is referred to as: a. calibration b. uniformity c. quality assurance d. linearity

D. ray sum The measurement of transmitted radiation made by an individual detector.

The measurement of transmitted radiation made by an individual detector is called a(n): a. attenuation coefficient b. Hounsfield value c. CT number d. ray sum

C. 1 and 3 only -- superior vena cava and ascending aorta The mediastinum is a potential space located between the two lungs. It contains the heart, great blood vessels, thymus and portions of the trachea and esophagus.

The mediastinum includes which of the following anatomic structures? 1. superior vena cava 2. stomach 3. ascending aorta a. 1 only b. 1 and 2 only c. 1 and 3 only d. 1, 2 and 3

C. spleen Hematoma, hemorrhage and laceration are typical signs of traumatic splenic injuries from blunt abdominal trauma.

The most common site of organ injury due to blunt abdominal trauma is the: a. liver b. kidney c. spleen d. pancreas

C. reduced scan times Involuntary motion is beyond control of the patient and include peristalsis, cardiac contraction, and tremors. *The most effective method of reducing involuntary motion on a CT scan is reduction in scan times.* Many scanners offer segmenting, or "halfs-scan," options whereby images may be reconstructed after a partial revolution of the tube-detector system. In the case of digestive involuntary motion, glucagon administration is an additional option.

The most effective method of reducing involuntary motion on a CT image is: a. immobilization b. thorough explanation of the examination to the patient c. reduced scan times d. physical restraint

C. 70 +/- 14 mL/min/m^2 Normal range of glomerular filtration rate (GFR) is 70 +/- 14 mL/min/m^2 for males, and 60+/- mL/min/m^2 for females.

The normal range of glomerular filtration rate (GFR) for men is: a. 50 +/- 14 mL/min/m^2 b. 60 +/- 10 mL/min/m^2 c. 70 +/- 14 mL/min/m^2 d. 80 +/- 10 mL/min/m^2

Frequency

The number of x-ray waves passing a given point per unit time is called:

D. 1, 2 and 3 -- reducing the kVp for an increase in displayed vessel opacification and contrast, retrospective reconstructions with a 50% overlap in section increment and bolus-tracking software to maximize contrast enhancement Reductions in kVp, overlapping axial sections and precise bolus timing are all important factors in the production of high-quality CTA examinations.

The overall quality of multiplanar reformation (MPR) and volume-rendered 3D images for a carotid artery CTA may be improved by: 1. reducing the kVp for an increase in displayed vessel opacification and contrast 2. retrospective reconstructions with a 50% overlap in section increment 3. bolus-tracking software to maximize contrast enhancement a. 2 only b. 1 and 2 only c. 2 and 3 only d. 1, 2 and 3

C. sampling each of the detector cells many times per second

The part of the CT system referred to as the DAS is responsible for: A) converting the light emitted from the detector to an electric current B) projecting the data from the attenuation profile onto a matrix C) sampling each of the detector cells many times per second D) converting digitized data to shades of gray to be displayed

D. Ground-glass opacity Ground-glass opacities are hazy areas of increased attenuation in the lungs and are often associated with the interstitial lung disease evaluated by high-resolution CT (HRCT).

The pathologic process indicated by number 3 on the figure most likely corresponds to: a. atelectasis b. lymphadenopathy c. pleural effusion d. ground-glass opacity

Rem or mRem

The personal dosimeter report reads in what unit of measurement?

B. low-contrast resolution

The phantom features a series of cylinders with different diameters, all at 0.6% (6HU) difference from the background material. This phantom is used to measure: A) slice thickness accuracy B) low-contrast resolution C) high-contrast resolution D) CT number accuracy

Differential absorption of the x-ray beam by the body

The photoelectric effect is responsible for:

Results in absorption of the incident photon

The photoelectric effect:

B. Coronal The pituitary and other structures involving the sella turcica are usually imaged in the coronal plane with CT. The coronal plane provides the best visualization of the pituitary gland with regard to its position within the sella turcica and involves less partial volume averaging of the pituitary with surrounding structures than scans obtained in the axial plane.

The pituitary gland is best demonstrated during CT in which of the following imaging planes? a. axial b. coronal c. sagittal d. transaxial

A. beam hardening The polyenergetic CT x-ray beam consists of photons of varying energy. As the beam traverses the patient, low-energy photons are absorbed first, increasing the average intensity of the beam as it travels along its path. This change, referred to as BEAM HARDENING, can have an artifactual result on the CT image.

The polyenergetic x-ray beam emitted from a CT x-ray tube is susceptible to artifacts resulting from the phenomenon known as: a. beam hardening b. density bloom c. kerma d. partial volume

C. contrast-induced nephrotoxicity (CIN) A potentially serious delayed effect of contrast agent administration. CIN is typically marked by an increase in serum creatinine concentration in comparison with a baseline measurement obtained before contrast agent administration.

The potentially serious decline in renal function after the IV administration of contrast material is called: a. anaphylaxis b. bronchospasm c. contrast-induced nephrotoxicity (CIN) d. urticaria

Peri-

The prefix that means "around" is:

A. low-contrast resolution We can recognize noise as the grainy appearance or "salt-and-pepper" look on an image. Contrast resolution involves differentiating an object from its very similar density background. Because the difference between object and background is small, *noise plays an important role in low-contrast resolution.*

The presence of noise on an image degrades its quality, particularly its A) low-contrast resolution B) longitudinal resolution C) spatial resolution D) temporal resolution

D. ostiomeatal complex An important sinus opening that allows for drainage of the frontal, ethmoid and maxillary sinuses. It is a common are for sinusitis, or inflammation of the sinuses.

The primary drainage opening within the sinus cavities, which is a common area for inflammation, is referred to as the: a. mucosal fossa b. adenovestibular complex c. external olfactory canal d. ostiomeatal complex

D. filtering the x-ray beam As x-ray beams pass through an object, lower-energy photons are preferentially absorbed, creating a "harder" beam. Individual rays are hardened to differing degrees, and this variation cannot be adjusted for by the reconstruction algorithm. *CT systems use three features to minimize beam hardening: filtration, calibration correction, and beam-hardening correction software.* Filtering the beam with a material such as aluminum filters out the lower-energy components of the beam before they pass through the patient.

The primary method of reducing beam-hardening artifacts is: A) increasing mAs B) decreasing slice thickness C) using a helical technique D) filtering the x-ray beam

A. partial volume effect The partial volume effect occurs when more than one type of tissue is contained within a voxel.

The process in CT by which different tissue attenuations are averaged to produce one less accurate pixel reading is called: A) the partial volume effect B) beam hardening C) photon deprivation D) ring artifacts

A. convolution This process is applied to reduce image unsharpness. An algorithm, or convolution kernel, acts as a mathematical filter, modifying the ray sum data and removing the unwanted blurring effect of the back-projection.

The process of applying a mathematical filter to remove blurring from the reconstructed CT image is termed: a. convolution b. interpolation c. iteration d. z filtration

C. multiplanar reformation (MPR) MPR describes the process of displaying CT images in a different orientation from the one used in the original reconstruction process. Unlike retrospective reconstruction, reformation does not change the makeup of the image voxels. Reformation merely alters the viewing perspective of the images to a different anatomic plane.

The process of displaying CT images in a different orientation from the one used in the original reconstruction process is called: a. retrospective reconstruction b. prospective reconstruction c. multiplanar reformation d. multisegment reconstruction

D. windowing Windowing is used to describe the process of grayscale mapping in the CT image, during which the display system assigned a shade of gray to an individual pixel on the basis of its CT number (HU).

The process of grayscale mapping of the CT image may be referred to as: a. analog-to-digital conversion b. retrospective reconstruction c. prospective reconstruction d. windowing

C. retrospective reconstruction Prospective reconstruction: occurs automatically with scanning. Reformatting: includes only image data step-and-shoot scanning: also called axial scanning and refers to a nonhelical scan process in which the table movement is stopped during each scan acquisition.

The process of using raw data to create a new image after the initial image has been generated is typically referred to as: a) prospective reconstruction b) reformatting c) retrospective reconstruction d) step-and-shoot scanning

C. reduce metrizamide pooling The prone position during a postmyelogram CT study of the lumbar spine reduces pooling or layering of the intrathecal contrast material.

The prone position may be used for a postmyelogram CT examination of the lumbar spine in an effort to: a. reduce the lordotic curve b. decrease patient gonadal radiation dose c. reduce metrizamide pooling d. increase the lordotic curve

D. large (48 cm) The scan field of view (SFOV) chosen must be larger than the width of the patient in order to eliminate out-of-field artifacts. Some CT scanners may have limited choices for SFOV values. Other common terminology includes FULL-FIELD or HALF-FIELD, and BODY-CAL and HEAD-CAL.

The proper scan field of view (SFOV) for a CT of the abdomen of a patient who measures 42 cm is: a. head (25 cm) b. small (25 cm) c. medium (35 cm) d. large (48 cm)

mAs

The quantity of x-rays produced is directly contolled by:

A. CTDIw Calculated from measurements made with dosimeters positioned a the center and periphery of a phantom and accounts for the variance in dose distribution resulting from the effects of beam hardening.

The radiation dose index calculation that takes into account the variations in absorption across the field of view due to beam hardening is termed: a. CTDIw b. CTDI100 c. CTDIvol d. MSAD

C. 1, 3, 4 and 5 -- tube current (mA), tube voltage (kVp), acquisition time (seconds) and acquisition length (millimeters) The radiation dose structured report (RDSR) must include all pertinent information related to the x-ray tube's output during CT data acquisition. This includes the tube current and voltage, scan time, acquisition length, pitch and collimation width.

The radiation dose structured report (RDSR) must include which of the following details regarding a CT acquisition: 1. tube current (mA) 2. display field of view (DFOV) 3. tube voltage (kVp) 4. acquisition time (seconds) 5. acquisition length (millimeters) a. 1 only b. 1, 2 and 4 c. 1, 3, 4 and 5 d. All (1 - 5)

A. from conception to 3 months' gestation This is the time of organ and neural crest development.

The radiosensitivity of a developing fetus is greatest a) from conception to 3 months' gestation b) from 3 months' to 5 months' gestation c) from 5 months' to 7 months' gestation d) from 7 months' to 7 months' gestation

A. 2-8 mg/mL Serum iodine concentration is a measure of the amount of iodine within the bloodstream. The specific dose depends on the patient's age, weight, and renal function, as well as on the clinical indication for the CT procedure.

The range of serum iodine concentration for adequate tissue opacification during contrast-enhanced CT examination is: a. 2-8 mg/mL b. 12-20 mg/mL c. 30-42 mg/mL d. 75-105 mg/mL

B. photon flux The rate at which a quantity of radiation (photon fluence) passes through a unit area over unit time. It may also be referred to as the FLUENCE RATE.

The rate at which a quantity of x-radiation emitted from a CT tube passes through a unit area over a unity of time is called the: a. effective mAs b. photon flux c. constant mAx d. photon fluence

B. attenuation The reduction in intensity of an x-ray beam as it interacts with matter. Several interactions are responsible for the attenuation of primary radiation, including Compton scatter and photoelectric effect. Each of these interactions reduce the energy and/or number of the x-ray photons in a primary beam.

The reduction in intensity of an x-ray beam as it interacts with matter is called: a. scatter b. attenuation c. transmission d. luminescence

C. Attenuation The reduction in x-ray intensity as the beam traverses the patient. The overall attenuation of the beam results from a combination of photon absorption and scatter.

The reduction in intensity of the CT x-ray beam as it passes through the patient's tissue may be generally termed: a. isotropy b. photodisintegration c. attenuation d. absorption

A. medical asepsis The reduction in number of infectious agents without the complete elimination of the organisms. Medical asepsis is accomplished through the use of soap, water and many other types of disinfectant materials.

The reduction in number of infectious organisms without a complete elimination is termed: a. medical asepsis b. sterilization c. surgical asepsis d. immunization

D. water Water has a CT number at or near zero. This are in the region of interest (ROI) in the figure most likely represents a hepatic cyst.

The region of interest (ROI) measurement in the figure provides an average density of +1.9 HU. This material is most likely: a. fat b. bood c. tumor d. water

A. mAs If the mAs setting is doubled the radiation dose is also doubled. Likewise, if the mAs setting is halved, the dose to the patient is also halved. Although kVp does affect the patient radiation dose, the effect is not linear. Changing from 120 to 140 kVp increases the radiation dose about 30% to 45%. Pitch also affects the radiation dose to the patient, but the effect is not linear.

The relationship between __________ and radiation dose to the patient is linear. a. mAs b. kVp c display field size d. pitch

Direct, although not proportional

The relationship between kVp and density is:

A. linearity The relative accuracy between calculated CT numbers and their respective linear attenuation coefficients is termed LINEARITY. Calibration and routine vendor maintenance procedures work to establish and maintain CT system linearity.

The relative accuracy between calculated CT numbers and their respective linear attenuation coefficients is referred to as: a. linearity b. calibration c. uniformity d. contrast resolution

Electron cloud

The result of thermionic emission is a/an:

15% rule

The rule or law that governs changing technique using kVp is the:

A. 3 mm Section thickness of a CT scan directly affects the detail and spatial resolution of the image. Narrow section widths result in greater detail for imaging of small anatomical structures and sections, such as the sinuses.

The section thickness that would demonstrate the greatest detail of the paranasal sinuses is: a. 3mm b. 5mm c. 7mm d. 10mm

B. 3 only -- Image magnification Increasing the display field of view (DFOV) for a reconstructed CT image decreases its displayed size. Electronically magnifying the image on the display system results in a larger displayed image but has no effect on pixel or voxel dimensions.

The size of the CT image displayed on a computer monitor can be enlarged by increasing the: 1. display field of view (DFOV) 2. scan field of view (SFOV) 3. image magnification a. 1 only b. 3 only c. 1 and 2 only d. 1 and 3 only

A) Ileum ***Ileum is the distal portion of the small intestine approximately 3.5 m in length and located in the RLQ***

The small intestine is divided into 3 parts, what is the name of the distal portion? A) Ileum B) Duodenum C) Jejunum D) Cecum

Atom

The smallest particle of an element that retains the characteristics of the element is a/an:

A. bit Bit is derived from BInary digiT and can appear as either a number 1 or 0. A sequence of eight bits constitutes a byte.

The smallest unit of information used in the binary language of computers is the: a. bit b. chip c. base d. byte

D. CT cystography A CT cystogram (not urogram) involves administering iodine directly into the bladder via a Foley catheter.

The specialized CT examination of the bladder in which an iodinated contrast agent is directly administered under gravity into the bladder via Foley catheter is termed: a. CT intravenous pyelogram (IVP) b. CT urogram c. CT enterolysis d. CT cystography

C. ischemic penumbra Penumbra is the ischemic, yet still viable, tissue immediately surrounding the infarct core. It can be described as the region of ischemic brain parenchyma where cerebral blood volume (CBV) is compromised but still higher than 2.5 mL per 100 g of tissue.

The still-viable ischemic cerebral tissue that surrounds the infarct core in a patient with an acute stroke is termed the: a. thrombolytic zone b. hemorrhage focus c. ischemic penumbra d. stroke volume

D. The Hounsfield bar The dense bony ares of the posterior fossa of the skull are especially prone to beam-hardening artifacts. HOUNSFIELD BAR is commonly used to describe this particular artifact.

The streaking artifact that occurs in the area of the posterior fossa during a CT examination of the brain may be referred to as: a. cupping b. the "boiled egg" artifact c. stairstep d. the Hounsfield bar

C) middle cerebral artery

The structure labeled #4 in the figure is: a) posterior cerebral artery b) vertebral artery c) middle cerebral artery d) common carotid artery

C) terminal ileum

The structure labeled #9 in the figure is: a) descending colon b) ascending colon c) terminal ileum d) transverse colon

-osis

The suffix that means "condition" is:

-pathy

The suffix that means "disease" is:

B. 1 and 3 only -- thin detector collimation and small DFOV Isotropic voxels have equal dimensions along the x, y and z-axes. Use of a thin reconstructed section width and small display field of view (DFOV) minimizes each voxel dimension.

The technical factor(s) necessary for the acquisition of an isotropic data set is/are: 1. thin detector collimation 2. high mA setting 3. small DFOV a. 1 only b. 1 and 3 only c. 2 and 3 only d. 1, 2 and 3

A. linearity The relationship between the CT number and actual linear attenuation coefficients of an object. It is used to measure the accuracy of a CT scanner.

The term used to describe the relationship between the linear attenuation coefficient of an object and the calculated CT number is: a. linearity b. mottle c. quantum noise d. spatial resolution

C. mesentery The mesentery is a double fold of the peritoneum that attaches the jejunum and ileum of the small bowel to the abdominal wall.

The thick, layered portion of the peritoneum responsible for attaching portions of the intestines to the bowel wall is called the: a. Gerota fascia b. falciform ligament c. mesentery d. ligamentum teres

C. cerebral aqueduct The third and fourth ventricles communicate through the CEREBRAL AQUEDUCT, which is commonly referred to as the AQUEDUCT OF SYLVIUS.

The third ventricle of the brain communicates with the fourth ventricle through the: a. anterior commissure b. septum pellucidum c. cerebral aqueduct d. fornix

B. T4 to T5

The trachea bifurcates at the level of: A) C6 to C7 B) T4 to T5 C) T7 - T8 D) T10 to T11

RAD

The traditional unit of absorbed dose is:

Curie

The traditional unit of activity is:

Autotransformer

The transformer that operates on the principle of self-induction is the:

A. uniform matrix array Consists of multiple detectors in the longitudinal direction, each with the same dimensions.

The type of multidetector CT (MDCT) detector array that contains multiple rows of detector elements, each of the same length, is called a(n): a. uniform matrix array b. adaptive array c. hybrid array d stationary array

Coulombs per kilogram

The unit of measurement that is determined by measuring ions produced in air as radiation passes through it is the:

Rem, curie, roetgen, rad

The units of equivalent dose, activity, in-air exposure, and absorbed dose are:

A. to reduce or eliminate beam-hardening artifacts from dense contrast media within the superior vena cava that may obscure small emboli in adjacent vessels

The use of a saline flush after the injection of the iodinated contrast is recommended for CTA pulmonary studies. Why? A) to reduce or eliminate beam-hardening artifacts from dense contrast media within the superior vena cava that may obscure small emboli in adjacent vessels B) to hydrate the patient and reduce the risk of contrast-induced nephropathy C) so that additional images can be acquired with a negative contrast agent D) to flush any emboli out of the patient's pulmonary arteries

C) Contrast media dose and timing can be easily regulated and reproduced in subsequent studies ***you can set the flow rate, amount of contrast and delay time for individual people***

The use of flow control mechanical pressure injectors are recommended for CT studies of the abdomen and pelvis due to which of the following characteristics? A) There is a reduced risk of extravasation of the contrast media into the soft tissue B) Side effects such as heat and nausea are reduced C) Contrast media dose and timing can be easily regulated and reproduced in subsequent studies D) The risk of air embolism is eliminated

C) Contrast media dose and timing can be easily regulated and reproduced in subsequent studies ***you can set the flow rate, amount of contrast and delay time for individual people***

The use of flow control mechanical pressure injectors are recommended for CT studies of the abdomen and pelvis due to which of the following characteristics? A) There is a reduced risk of extravasation of the contrast media into the soft tissue B) Side effects such as heat and nausea re-reduced C) Contrast media dose and timing can be easily regulated and reproduced in subsequent studies D) The risk of air embolism is eliminated

C. contrast media dose and timing can be easily regulated and reproduced in subsequent studies The use of mechanical injection systems in CT produces the best results. However, precautions must be taken to prevent contrast media extravasation and care must be taken in the preparation and connection of the injector and cannula to avoid the risk of large air emboli.

The use of flow-control mechanical pressure injectors are recommended for CT studies of the chest, abdomen and pelvis because: A) there is a reduced risk of extravasation of the contrast media into the soft tissues B) side effects such as heat and nausea are reduced C) contrast media dose and timing can be easily regulated and reproduced in subsequent studies D) the risk of air embolus is eliminated

B. 1 and 2 only -- decreased noise on reconstructed image and decreased radiation dose to patient Iterative reconstruction techniques are used to increase the signal-to-noise ratio (SNR) of the reconstructed CT image. This reduction in image noise allows for reduced technical factors (tube milliamperage) and lower patient radiation dose. Iterative reconstruction techniques require increased reconstruction time, made acceptably short enough by today's powerful computer processors.

The use of iterative reconstruction techniques for CT data processing results in: 1. decreased noise on the reconstructed image 2. decreased radiation dose to the patient 3. decreased image reconstruction time a. 3 only b. 1 and 2 only c. 1 and 3 only d. 1, 2 an 3

D. 1, 2 and 3 -- increases palatability/improved patient comfort, better demonstration of enhancing bowel wall and no interference with 3D applications The use of water as a NEGATIVE contrast agent is becoming increasingly common for many CT applications for the abdomen and pelvis.

The use of water as an oral contrast material for CT of the abdomen and/or pelvis has several potential advantages, including: 1. increased palatability and improved patient comfort 2. better demonstration of enhancing bowel wall 3. no interference with 3D applications a. 1 only b. 1 and 2 only c. 1 and 3 only d. 1, 2 and 3

C. Agatston score A scoring system that quantifies the volume and density of calcium within the coronary arteries.

The volume and density of calcium measured in the coronary arteries on multidetector CT (MDCT) examinations are quantified with use of a value called the: a. Euler constant b. Hounsfield number c. Agatston score d. Lambert-Beer value

B. pitch CTDIvol is used to approximate the radiation dose for each section obtained during a helical scan. It corresponds to the axially acquired CTDIw divided by the helical pitch.

The weighted computed tomography dose index (CTDIw) can be converted into a volumetric CT dose index (CTDIvol) that is useful for estimating patient dose during helical acquisitions by dividing the CTDIw by the scan's: a. milliamperage (mA) b. pitch c. multiple scan average dose (MSAD) d. section width

D. black The window width selects the range of HU that is to be represented as shades of gray on the image. The window level determines which, of all the possible HU, are to be included. In this example, the total HU to be represented is 120. Because the center is set at 50, the HU depicted are -10 to 110. *This is calculated by dividing 120 in half, then subtracting the quotient from the center (50) to find the lower limit. Add the quotient to 50 to find the upper limit. Everything that falls below this range appears black, whereas everything above this range appears white*.

The window width of a specific CTimage is set at 120 and the level (or center) is set at 50. How is a structure with a measurement of -40 HU displayed? A) white B) light shade of gray C) dark shade of gray D) black

D. rotate-rotate Both the x-ray tube and detector array rotate around the patient during scanning with a third-generation CT unit.

Third-generation CT scanners use which of the following scan geometries? a. translate-rotate b. rotate-stationary c. transaxial d. rotate-rotate

A. 1 mm High-resolution CT (HRCT) acquisitions of the chest should be performed with thin sections, within the range of 1 - 2 mm.

This high-resolution computed tomography (HRCT) image of the chest was most likely acquired with a section width of: a. 1 mm b. 5 mm c. 7 mm d. 10 mm

D. 1, 2 and 3 -- breathing instructions are properly followed, motion artifact does not occur, and patient anxiety is kept at a minimum Good communication between the technologist and patient is an extremely important factor in the production of high-quality CT examinations. Patients can be made to feel more comfortable, and when they are given detailed instructions, their greater cooperation can improve the CT results.

Thorough explanation of the CT procedure and proper communication with the patient are vital in ensuring that: 1. breathing instructions are properly followed 2. motion artifact does not occur 3. patient anxiety is kept at a minimum a. 1 and 2 only b. 1 and 3 only c. 2 and 3 only d. 1, 2 and 3

D. encryption Digital encryption is used to encode data that is transferred across a network into a form decipherable only by software at the destined location.

To maintain security and patient privacy when medical images are transmitted across a network, the images may be encoded in a form decipherable only by software at the destination location. This process is referred to as: a. write-once, read-many (WORM) b. Hypertext Transfer Protocol (HTTP) c. teleradiology d. encryption

D. 1, 2 and 3 -- automatic exposure control, adult and pediatric protocols, and DICOM radiation dose structured reporting (RDSR). In the US, CT systems are required to feature DICOM radiation dose structured reporting (RDSR), a dose check system that alerts that technologists that the scan parameters may result in a patient dose that is higher than acceptable, an automated tube current modulation (ATCM) system that serves as an automatic exposure control (AEC), and reference protocols for both adult and pediatric patients.

To meet current industry standards for required dose reduction measures, a CT system must employ which of the following? 1. automatic exposure control (AEC) 2. adult and pediatric protocols 3. Digital Imaging and Communications in Medicine (DICOM) radiation dose structured reporting (RDSR) a. 1 only b. 1 and 2 only c. 2 and 3 only d. 1, 2 and 3

C. 80 - 130 seconds The nephrogram phase occurs when the contrast media reaches the kidneys.

To scan in the nephrographic phase of contrast enhancement, scans should be acquired approximately ________ after a bolus contrast injection. A) 15 - 25 seconds B) 30 - 70 seconds C) 80 - 130 seconds D) 3 15 minutes

B. overbeaming All detectors of the multidetector CT (MDCT) array must be exposed to x-rays of equal intensity. The beam must be expanded even further to avoid exposing the detectors to undesirable "penumbra." This process is referred to as overbeaming.

To uniformly expose the entire detector array in a multidetector CT (MDCT) system, the primary beam must be expanded beyond the physical extent of the array in a process known as: a. interpolation b. overbeaming c. photon flux d. back-projection

True or False CIN stands for Contrast-Induced Nephropathy

True

True

True or False The liver has a dual blood supply

True

True or False The liver has a dual blood supply?

True

True or False The number of detector rows is inversely proportional to the measured radiation dose.

True ***The reason for this is since the aorta arches to the left side of the body***

True or False There is no left Brachiocephalic Artery?

True

True or false The Levator Ani provide support to the pelvis content

True

True or flase The Levator Ani provide support to the pelvis content

C. hives Urticaria is a common adverse reaction to iodinated intravenous contrast material. Characterized by the presence of wheals or localized skin eruptions, it is commonly referred to as hives.

Urticaria is which of the following? a. severe nausea with associated vomiting b. urinary tract infection c. hives d. bronchospasm

B. osmolality Osmolality, or osmotic concentration, is the number of ions or particles formed when a substance (solute) dissociates in a given solution. *It is described as the number of particles in solution per kg of water.*

Used in determining the biologic effect of iodinated contrast media, the term _________ refers to the number of ions formed when a substance dissociates in solution. a. solubility b. osmolality c. concentration d. iodination

D) a scan of the abdomen; DFOV = 44; slice thickness = 5mm The slice thickness is irrelevant in determining pixel size. *Pixel size can be calculated by dividing the DFOV by the matrix*. Therefore, the pixel size of: choice A) is 0.31 mm; choice B) is 0.39mm; choice C) is 0.59mm; and choice D) is 0.86mm. *Slice thickness does not impact pixel size, only voxel size*.

Uses a standard 512 matrix for all studies, which contains the largest pixels? A) an image of the pituitary; DFOV = 16; slice thickness = 2mm B) an image of the knee: DFOV = 20; slice thickness = 1mm C) a scan of the neck; DFOV = 30; slice thickness = 3mm D) a scan of the abdomen; DFOV = 44; slice thickness = 5mm

D) Minimum intensity projection ***technique uses the minimum or lowest attenuation values to display the images. This protocol aids in diagnosing abnormalities of the central tracheobronchial system and for localizing extrabronchial air collections and in demonstrating strictures, concentric stenosis and dilations***

Various 3D rendering techniques are used today to enhance diagnostic techniques are used today to enhance diagnostic interpretation. Which of the following techniques is best utilized for the evaluation of the central tracheobronchial system? A) Maximum intensity projection B) Volume rendering C) Surface rendering D) Minimum intensity projection

D) Minimum intensity projection ***technique uses the minimum or lowest attenuation values to display the images. This protocol aids in diagnosing abnormalities of the central tracheobronchial system and for localizing extrabronchial air collections and in demonstrating strictures, concentric stenosis and dilation's***

Various 3D rendering techniques are used today to enhance enhance diagnostic interpretation. Which of the following techniques is best utilized for the evaluation of the central tracheobronchial system? A) Maximum intensity projection B) Volume rendering C) Surface rendering D) Minimum intensity projection

Quantum noise

Visibility of an object's edge may be limited by:

B. isotropic The ability to acquire an isotropic data set (voxels with equal dimensions) is a distinct advantage of multidetector CT (MDCT).

Voxels with equal dimensions along the x, y and z axes are referred to as: a. prospective b. isotropic c. retrospective d. anisotropic

D) BE view

What 3D rendering technique is illustrated in the figure? a) volume rendering (VRT) b) surface-shaded display (SSD) c) multiplanar reformation (MPR) d) BE view

B) Inferior Mesenteric Artery ***The inferior mesenteric artery originates from the aorta at approximately L3 and supplies blood to the left half of the transverse colon, descending colon, sigmoid colon, and upper rectum***

What abdominal arteries lies at approximately L3 and supplies blood to the distal half of the large intestines? A) Superior renal arteries B) Inferior mesenteric artery C) Superior mesenteric artery D) Superior rectal artery

B) Inferior Mesenteric Artery ***The inferior mesenteric artery originates from the aorta at approximately L3 and supplies blood to the left half of the transverse colon, descending colon, sigmoid colon, and upper rectum***

What abdominal arteryies lies at approximately L3 and supplies blood to the distal half of the large intestines? A) Superior renal arteries B) Inferior mesenteric artery C) Superior mesenteric artery D) Superior rectal artery

D) Transverse Abdominis ***The transverse abdominis muscle extends transversely to provide support for the abdominal viscera. Its function is to compress the abdominal viscera***

What abdominal wall muscle compresses the abdominal viscera when the trunk bends and its insertions are the xiphoid process, linea alba, and the pubis? A) Longissimus B) Psoas C) External oblique D) Transverse abdominis

D) Transvers Abdominis ***The transvers abdominis muscle extends transversely to provide support for the abdominal viscera. Its function is to compress the abdominal viscera***

What abdominal wall muscle compresses the abdominal viscera when the trunk bends and its insertions are the xiphoid process, linea alba, and the pubis? A) Longissimus capitis B) Psoas C) External oblique D) Transvers abdominis

Somatic

What are radiation effects that manifest themselves in the person being irradiated?

Prednisone Diphenhydramine

What are the 2 steroids/antihistamines that are given to a patient who is allergic to iodinated contrast?

mitral valve tricuspid valve aortic valve pulmonary valve

What are the 4 valves of the heart?

mitral valve, tricuspid valve, aortic valve, pulmonary valve.

What are the 4 valves of the heart?

scaphoid, lunate, triquetrum, pisiform, trapezium, trapezoid, capitate, hamate

What are the 8 carpal bones?

scaphoid, lunate, triquetrum, pisiform, trapezium, trapezoid, capitate, hamate Steve Left The Party To Take Carol Home

What are the 8 carpal bones??

C) Cisterns ***Are locations primarily around the base of the brain where the subaracbnoid space becomes widened.***

What are the structures located around the base of the brain where the subarachnoid space becomes widened? A) Lentiform nucleus B) Red nucleus C) Cisterns D) Quadrigeminal plate

C) Cisterns ***Are locations primarily around the base of the brain where the subarachnoid space becomes widened.***

What are the structures located around the base of the brain where the subarachnoid space becomes widened? A) Lentiform nucleus B) Red nucleus C) Cisterns D) Quadrigeminal plate

B) High x-ray intensity and heat dissipation ***With the introduction of spiral/helical CT scanning new technical advancements are needed to be made in order to deal with the continually rotating tube for longer periods of time***

What are the two principal demands placed on an x-ray tube for CT imaging? A) Heat dissipation and beam filtration B) High x-ray intensity and heat dissipation C) High x-ray intensity and rotation speed D) Line focus principle and rotating anode

D) Corpus Callosum ***Corpus Callosum is the largest and densest bundle of white matter in the cerebrum.***

What are the white matter tracts that connect the two cerebral hemispheres? A) Anterior commissure B) Posterior commissure C) Septum pellucidum D) Corpus callosum

C) Left subclavian artery ***the left subclavian artery is the third major branch of the aorta. This artery is located more laterally ascending through the upper thorax to exit through the thoracic inlet thus forming the left axillary artery***

What artery located in the supra aortic mediastinum is the most posterior and is situated adjacent to the left side of the trachea? A) Left common carotid artery B) Right common carotid C) Left subclavian artery D) Brachiocephalic trunk

C) Left subclavian artery ***the left subclavian artery is the third major branch of the aorta. This artery is located more laterally ascending through the upper thorax to exit through the thoracic inlet thus forming the left axillary artery***

What artery located in the supra aortic mediastinum is the most posterior and is situated to the left side of the trachea? A) Left common carotid artery B) Right common carotid C) Left subclavian artery D) Brachiocephalic trunk

Primary protective barrier

What barrier must be at least 1/16 in lead equivalent?

C) Carotid arteries ***an emboli or thrombus in the internal carotid arteries are a major reason for strokes***

What blood vessel is responsible for major strokes? A) Coronary arteries B) Femoral arteries C) Carotid arteries D) Jugular veins

C) Posterior Cerebral ***originates from the basilar artery. its located along the upper border of the pons and extends above the tentorium cerebellum supplying blood to the occipital lobe***

What branch of the Circle of Willis originates from the basilar artery to supply blood to the occipital lobe? A) anterior communicating B) Middle Cerebral C) Posterior cerebral D) Internal carotid

B. contrast administration is immediately followed by a saline solution bolus A "saline chaser" reduces streak artifacts over the superior vena cava that result from dense contrast material. Although increasing the scan delay (choice C) would reduce the streak artifact by allowing the contrast to dilute, the examination would be degraded.

What can be done to reduce streak artifacts such as those in the figure? A) increase the kVp B) contrast administration is immediately followed by a saline solution bolus C) begin scanning 45-seconds after contrast media is seen in the monitoring slice D) lift the patient's arms over his or her head

Indirect effect

What causes about 95% of the cellular response to radiation?

D. barium leaking into the peritoneal cavity from a perforation of the gastrointestinal tract The mortality rate from barium peritonitis is significant. To prevent this condition, a water-soluble solution is recommended whenever there is suspicion of a gastrointestinal tract perforation.

What causes barium peritonitis? A) aspiration of barium into the lungs B) the oral administration of a large volume of a barium sulfate solution in patients with an obstructed bowel C) the oral administration of a large volume of a barium sulfate solution in patients with renal insufficiency D) barium leaking into the peritoneal cavity from a perforation of the gastrointestinal tract

D. imperfect detector elements -- either faulty or simply out of calibration Sometimes ring artifacts can be eliminated simply by recalibrating the scanner. If that fails, a service engineer must be called to repair the defective detector.

What causes ring artifacts? A) electrical surges (i.e. a "short circuit") within the x-ray tube B) involuntary patient motion (e.g. heart motion) C) irregularly shaped objects that have a pronounced difference in density from surrounding structures D) imperfect detector elements -- either faulty or simply out of calibration

B) Multiplanar reformation ***The principle of multiplanar reformation is to reconstruct a new series if images at arbitrary or even curved planes defined by the operator, using the pixel data from a stack of transaxial images.***

What classification of image manipulation stacks the image data to generate a new series of images in a plane or orientation different from the perspective image? A) Prospective reconstruction B) Multiplanar reformation C) Object-based analylsis D) Scene-based analysis

B) Multiplanar reformation ***The principle of multiplanar reformation is to reconstruct a new series if images at arbitrary or even curved planes defined by the operator, using the pixel data from a stack of transaxial images.***

What classification of image manipulation stacks the image data to generate a new series of images in a plane or orientation different from the perspective image?? A) Prospective reconstruction B) Multiplanar reformation C) Object-based analylsis D) Scene-based analysis

Multiple Slice Average Dose

What does MSAD stand for?

A) Diaphragm *** The diaphragm is a dome shape muscle that spans the entire thoracic outlet separating the thoracic and abdominal cavity***

What dome shaped muscular sheet forms a convex floor for the chest and serves a septum between the thoracic and abdominal cavities? A) Diaphragm B) Costomediastinal recess C) Costodiaphragmatic recesses D) Intercostal Muscle

C) Duct of Wirsung ***Duct of Wirsung is the main pancreatic duct that carries enzymes to the duodenum. It runs the length of pancreatic gland to the ampulla of Vater***

What duct traverses the length of the pancreatic gland to join with the common bile duct at the hepatopancreatic ampulla? A) Cystic Duct B) Left and right intrahepatic ducts C) Duct of Wirsung D) Duct of Santorin

C) Duct of Wirsung ***Duct of Wirsung is the main pancreatic duct that carries enzymes to the duodenum. It runs the length of pancreatic gland to the ampulla of Vater***

What duct traverses the length of the pancreatic gland to join with the common bile duct at the hepatopancreatic ampulla? A) Cystic Duct B) Left and right intrahepatic ducts C) Duct of Wirsung D) Duct of Santorini

B) Second generation ***The second generation scanner employed a translate-translate scanning principle in which the tube and detector would translate across the patients, and translate back, and then rotate the assemble 10 degrees.***

What generation CT scanner utilized a translate-translate scanning geometry principle with 5-30 detector linear array assembly? A) First generation B) Second generation C) Third generation D) Fourth generation

B) Ionization Chamber *** Ionization chamber is used to measure the dose. When radiation falls upon the chamber, it ionizes the air in the chamber to produce ion pairs.***

What instrument is used to measures radiation exposure? A) Geiger Counter B) Ionization Chamber C) Reference Detector Chamber D) Solid-State Detector Apeture

B. patients on dialysis who still have some residual kidney function should not be given contrast media; patients with end-stage renal failure can be given contrast media If the intention is that the dialysis is short-term therapy, with the hope that the kidney will recover, the patient should not be given IV contrast media. The aim is to avoid any risk of further renal insult that could diminish residual renal function and result in the renal failure becoming chronic with the need for ongoing hemodialysis

What is a general rule regarding the administration of IV contrast media to patients receiving dialysis? A) patients on dialysis should never be given contrast media B) patients on dialysis who still have some residual kidney function should not be given contrast media; patients with end-stage renal failure can be given contrast media C) when contrast media is given, dialysis must be scheduled to be performed within 4 hours D) contrast media poses no risks to patients on dialysis

A) No afterglow ***Gas ionization detectors do not produced after-glow, since they do not employ scintillation material. The detectors use gas (xenon) ionization chambers which convert x-ray energy directly into electrical energy.***

What is a useful characteristic of a gas-filled detector for CT imaging? A) No afterglow B) Higher detector efficiency C) Direct digitization D) DAS not required

D. monitor Output devices accept processed data from the computer. Other examples of output devices are laser cameras, printers, and archiving equipment such as optical disks or magnetic tape. CT detector mechanisms and touch-sensitive plasma screens are examples of input devices that feed data into the computer. The microprocessor is part of the CPU.

What is an example of a computer output device? a. CT detector mechanisms b. microprocessor c. touch-sensitive plasma screen d. monitor

D) Windowing ***Windowing is a computer visualization tool by which the image gray scale is manipulated with regard to the CT number of the structure of interest. The two main components of windowing include window width (controls contrast) and window level (controls the brightness levels)***

What is considered a computer visualization tool responsible for controlling the brightness and contrast of an image on the computer monitor? A) 3D volume rendering B) Multi-planner reformation C) Region of interest measurements D) Windowing

D) Windowing ***Windowing is a computer visualization tool by which the image gray scale is manipulated with regard to the CT number of the structure of interest. The two main components of windowing include window width (controls contrast) and window level (controls the brightness levels)***

What is considered a computer visualization tool responsible for controlling the brightness and contrast of an image on the computer monitor? A) 3D volume rendering B) Multi-planner reformation C) Region of interest measurements D) Windowing

B) PACS ***Picture Archiving and Communication System. Is one of the 2 key elements that form the radiology department's infrastructure***

What is considered an electronic system for archiving, transmitting, viewing, and manipulating images in the radiology department? A) DICOM B) PACS C) HIS D) RIS

A) Voxel ***A voxel is synonymous with volume element for a 2D CT image in the z-axis direction. The voxel is expressed by the width of the sides of the pixels (area) and the width of the slice***

What is defined as the z-axis segmentation? A) Voxel B) Pixel C) Matrix D) FOV

B) Aortic Dissection ***Aortic Dissection occurs when blood enters the wall of an artery separating the vascular layer creating a cavity or false lumen in the vessel wall**

What is described as a tear in the wall of the aorta that causes blood to flow between layers of the wall of the aorta and forces the layers apart? A) Aortic Aneurysm B) Aortic Dissection C) Embolism D) Thrombosis

D) Reduction of radiation dose ***One of the advantages of utilizing a high-frequency generator is the ability to use high kVp to minimize the photoelectric absorption which in turn reduces radiation dose. Higher kVp means that the photons have higher energies and can penetrate thicker objects. The radiation dose is proportional to the square of the kVp***

What is one of the advantages to using high kVp during CT imaging? A) Reduction of imaging time B) Reduction of reconstruction time C) Reduction intensity at the image receptor D) Reduction of radiation dose

D) Reduction of radiation dose ***One of the advantages of utilizing a high-frequency generator is the ability to use high kVp to minimize the photoelectric absorption which in turn reduces radiation dose. Higher kVp means that the photons have higher energies and can penetrate thicker objects. The radiation dose is proportional to the square of the kVp***

What is one of the advantages to using high kVp during CT imaging? A) Reduction of imaging time B) Reduction of reconstruction time C) Reduction intensity at the image receptor D) Reduction of radiation dose

D) 640 mm ***The DFOV is the product of the pixel size and the matrix size***

What is the DFOV for a particular image having a pixel size of 0.625 mm and the matrix size of 10,242? A) 320 mm B) 320 cm C) 640 mm2 (the 2 is squared) D) 640 mm

D) 640 mm ***The DFOV is the product of the pixel size and the matrix size***

What is the DFOV for a particular image having a pixel size of 0.625 mm and the matrix size of 10,242? A) 320 mm B) 320 cm C) 640 mm^2 D) 640 mm

C) DLP ***DLP replicates the radiation output and the potential biological effects attributed to the scan. The DLP can be calculated by the product of the CTDI Volume and the scan length***

What is the dose descriptor that can be calculated if the length of the irradiated volume (scan length) and the CTDI volume are known? A) CTDI 100 B) MSAD C) DLP D) Exposure

B) Thrombosis ***Thrombosis is the formation of a blood clot inside a blood vessel, obstructing the flow of blood through the circulatory system***

What is the formation development or existence of a clot within the vascular system? A) Embolus B) Thrombosis C) Hematoma D) Dissection

C) S3 to the tip of the coccyx ***The rectum is the terminal part of the large intestine which extends from S3 to the tip of the coccyx. It is approximately 15 cm in lengt***

What is the landmark for the terminal part of the large intestine (rectum)? A) L5 B) L4 C) S3 to the tip of the coccyx D) S

C) S3 to the tip of the coccyx ***The rectum is the terminal part of the large intestine which extends from S3 to the tip of the coccyx. It is approximately 15 cm in lengt***

What is the landmark for the terminal part of the large intestine (rectum)? A) L5 B) L4 C) S3 to the tip of the coccyx D) S1

A) Ilium ***The ilium is the largest and most superior bone of the pelvis made up of the body and ala (wing like structure).***

What is the largest division of the acetabulum? A) Ilium B) Ischium C) Pubis D) Sacrum

C. 20.0 mm At a maximum, the MDCT cone beam may be collimated to a dimension equal to the entire multirow detector array. In the example, the total dimension of the detector array along the z-axis is 16 x 1.25 mm, or 20.0 mm.

What is the maximum beam collimation for a MDCT system with 16 rows of 1.25 mm detectors? a. 2.5 mm b. 5.0 mm c. 20.0 mm d. 40.0 mm

B) Osmolality ***Osmolality is the measurement of the total number of particles in the contrast solution per kilogram of water

What is the measure of the total number of particles in contrast media per kilogram of water? A) Viscosity B) Osmolality C) Ionicity D) Iodination

A) Increase scan speed ***Faster scanners reduced motion artifact because the patient has less time to move during the acquisition. This can only be accomplished with faster gantry rotation or more x-ray sources.***

What is the most straightforward technical way to reduce or eliminate the motion of an object? A) Increase scan speed B) Removing metal objects C) Gantry angulation D) Decreasing slice thickness

A) Increase scan speed ***Faster scanners reduced motion artifact because the patient has less time to move during the acquisition. This can only be accomplished with faster gantry rotation or more x-ray sources.***

What is the most straightforward technical way to reduce or eliminate the motion of an object? A) Increase scan speed B) Removing metal objects C) Gantry angulation D) Decreasing slice thickness

B. manubrium

What is the most superior part of the sternum? A) xiphoid process B) manubrium C) ensiform D) gladiolus

B) Obturator Foramen ***The union of the pubic rami and the ischium surrounds a large opening in the pelvis. The obturator foramen is enclosed by the oburator muscle***

What is the name for the largest foramen found in the body? A) Foramen magnum B) Obturator foramen C) Rotundum foramen D) Foramen ovale

C) Iterative reconstruction ***The iterative reconstruction method takes the attenuation coefficient intensities and back projects them onto the matrix with assumptions that all the intensities have the same value.***

What is the name given to the algorithm that compares an assumption with the measured values, makes corrections, and repeats the cycle? A) Analytical reconstruction B) Back projection C) Iterative reconstruction D) Fourier reconstruction

B) 12-20 BPM

What is the normal range of respiration for an adult? A) 5-20 BPM B) 12-20 BPM C) 20-30 BPM D) 35-50 BPM

B. 1.375 55/(64 x 0.625) = 55/40 55/40 = 1.375

What is the pitch in the following scenario: 64 detector channels used, 0.625mm slice thickness, table movement of 55mm per rotation? A) 1 B) 1.375 C) 1.5 D) 2

A) Pre-medication with steroids and antihistamines

What is the pre-medication strategy for a patient with a prior allergic reaction to iodinated contrast media before a contrast-enhanced CT? A) Pre-medication with steroids and antihistamines B) Increased fluids for 48 hrs before the exam C) Having the patient stop his/her blood thinning medication D) Administration of a negative contrast

A) Superior enhancement of iodine ***The principle behind the benefit of lower kV is in some clinical applications is that the attenuation coefficient of iodine increases as photon energy decreases.***

What is the principle behind the benefit of lower kV in some clinical applications? A) Superior enhancement of iodine B) Inferior enhancement of iodine C) There is actually no benefit of reducing kVp D) It increases patients dose

A) Bladder ***The bladder is roughly pyramid shaped with the apex pointing downward between the pubis and vagina in females and the rectum in males***

What is the pyramid-shaped muscular organ that rest on the pelvis floor immediately posterior to the symphysis pubis? A) Bladder B) Uterus C) Prostate Gland D) Seminal Vesicles

B) Effective Dose ***Is not actually a measurement of dose. But rather a model that predicts the stochastic risk, like cancer from radiation exposure.***

What is the quantifying risk from partial-body exposure to that from the equivalent whole body dose? A) Adsorbed Dose B) Effective Dose C) Threshold Effect D) Deterministic Effect

A) -275 to 175

What is the range of CT numbers if the window width is 450 and the window level is 50? A) -275 to 175 B) 275 to -175 C) 500 to -400 D) -500 to 400

B) As the number of detector rows increases dose decreases. ***For multi-slice scanners the number of detector rows dictates dose. As the detector rows increases from 4 to 64 the dose decreases.

What is the relationship between the dose and the number of detector rows? A) As number of detector rows increase dose increases B) As number of detector rows increases dose decreases C) As number of detector rows increases dose triples D) There is no relationship between dose and detector rows

C) Diaphragm to the ischial tuberosities

What is the scanning range when performing an abdominal survey that includes all the abdominal compartments and pelvis? A) Diaphragm to the iliac crest B) Lung bases to the iliac crest C) Diaphragm to the ischial tuberosities D) Kidneys through the bladder

B) Osmolality *** Osmolality is the measurements of the total number of particles in the contrast solution per kilograms of water. Its the direct measurement of the ionization of a solute in a solvent.***

What is the term that refers to the number of ions formed when a substance dissociated in a solution? A) Solubility B) Osmolality C) Concentration D) Iodination

Medical asepsis

What is the type of infection control in which microorganisms have been eliminated as much as possible by the use of water and chemical disinfectants?

B) Anterior longitudinal ligament ***Consists of a layer of dense connective tissue, that is firmly attached to the anterior surface of the vertebral bodies and intervertebral disc***

What ligament extends downward from C2 along the anterior surface of the vertebral surface to the sacrum? A) Transverse Ligament B) Anterior longitudinal ligament C) Posterior longitudinal ligament D) Supraspinatus ligament

B) Superior Vena Cava ***is the major vein located on the superior aspect of the right side of the heart.Its formed by the junction of the brachiocephalic veins and its function is to drain blood from the upper trunk into the right atrium***

What major vein is formed by the junction of the right and left brachiocephalic vein? A) Internal jugular B) Superior Vena Cava C) External jugular D) Retromandibular vein

B) Superior Vena Cava ***is the major vein located on the superior aspect of the right side of the heart.Its formed by the junction of the branchiocephalic veins and its function is to drain blood from the upper trunk into the right atrium***

What major vein is formed by the junction of the right and left brachiocephalic vein? A) Internal jugular B) Superior Vena Cava C) External jugular D) Retromandibular vein

B) Pia Mater ***Is the most innermost layer of the meninges surrounding the brain and spinal cord***

What mater is located in the innermost layer of the meninges? A) Dura Mater B) Pia mater C) Arachnoid mater D) Falx cerebri

D. 512 x 512 pixels The matrix size used to reconstruct an image can be calculated by dividing the display field of view (DFOV) by the pixel dimension. A pixel whose area is 0.25 mm^2 has a linear dimension of 0.5 mm. Dividing the DFOV of 250 mm by the pixel dimension 0.5 mm gives a matrix size of approximately 512 x 512 pixels.

What matrix size was used to reconstruct an image with a display field of view (DFOV) of 25 cm and a pixel area 0.25 mm^2? a. 80 x 80 pixels b. 256 x 256 pixels c. 320 x 320 pixels d. 512 x 512 pixels

B) Heating the media to body temperature ***Solutions with high viscosity, like with nonionic contrast media. Require greater injection pressures for administration. The contrast media viscosity may be reduced by warming the media to body temp***

What method is used to lower the viscosity of contrast media before injecting? A) Heating the media to room temperature B) Heating the media to body temperature C) Mixing the media with a saline solution D) Viscosity of the media can never be altered

D) Teres major ***Teres major muscle is a flat rectangular muscle originating at the inferior angle of the scapula. Its function is to rotate the arm laterally and to abduct the arm***

What muscle is not associated with rotator cuff injury? A) Supraspinatus B) Infraspinatus C) Subscapularis D) Teres major

D) Teres major ***Teres major muscle is a flat rectangular muscle originating at the inferior angle of the scapula. Its function is to rotate the arm laterally and to abduct the arm***

What muscle is not associated with rotator cuff injury? A) Supraspinatus B) Infraspinatus C) Subscapularis D) Teres major

B) Levator Ani ***The 2 levator ani muscles are the largest and most important of the pelvis floor originating from the symphysis pubis and ischial spines to form wing like arches that attach to the coccyx.***

What muscle is the largest and most important of the pelvic floor? A) Coccygeus muscle B) Levator Ani C) Gluteus maximus D) Obturator externus

A) 1

What number in the figure illustrates the anterior cerebral artery? a) 1 b) 2 c) 4 d) 5

B) 45

What number in the figure illustrates the right external iliac artery? a) 46 b) 45 c) 50 d) 26

B) 29

What number in the figure illustrates the superior mesenteric artery? a) 30 b) 29 c) 25 d) 38

Characteristic radiation

What occurs when an incident electron dislodges a k-shell electron?

Bremsstrahlung

What occurs when an incident electron interacts with the force field of an atomic nucleus?

Compton interation

What occurs when an incident photon interacts with an outer shell electron, producing a scatter photon and a recoil electron?

D) Total detector efficiency is increased ***When there are gaps between detectors, there is a possibility a photon will not hit the intended detector, thus causing no signal. When the detectors are firmly packed together, this phenomenon is less likely to occur resulting in an increased total detector efficiency***

What occurs when the interspace distance between detectors is reduced? A) Detector response time is reduced B) Patient dose is increased C) Signal acquisition time in increased D) Total detector efficiency is increased

A) Spleen ***The spleens function is to produced white blood cells, filter abnormal blood cells, store iron, and initiate immune responses***

What organ of the body is considered the largest lymph node and highly vascular? A) Spleen B) Gallbladder C) Stomach D) Liver

C. display processor The digitized data are sent from the reconstruction processor to the display processor, which then converts it to shades of gray.

What part of the CT system is responsible for converting the digitized data into shades of gray? A) photodiode B) array processor C) display processor D) analog-to-digital converter

B) cerebral hematoma The hyperdensity seen in the image lends the radiologist to interpret this case as a cerebral hematoma. The large hematoma is causing a large midline brain shift.

What pathological condition is demonstrated in the figure? a) brain infarct b) cerebral hematoma c) epidural hematoma d) acoustic neuroma

A) epidural hematoma Appears as a classic biconvex (lentiform or football), shaped lesion with the dura bulging inward. Blood collects between the dura mater and the skull that is mainly caused as a consequence of a blunt force trauma. In CT imaging during the acute stage, the hemorrhage appears hyperdense, but as it proceeds into the subacute and chronic stage, the hemorrhage becomes isodense to hypodense. Skull fractures are usually present with an epidural hematoma.

What pathological condition is demonstrated in the figure? a) epidural hematoma b) subdural hematoma c) subarachnoid hematoma d) glomus tumor

C. subarachnoid hematoma Subarachnoid hemorrhage is a collection of blood in the space between the arachnoid mater and pia mater. On CT images, a hyperdensity appears within the subarachnoid space, especially in the basal cisterns and into the pathways of the CSF.

What pathological condition is demonstrated in the figure? a) epidural hematoma b) subdural hematoma c) subarachnoid hematoma d) metastatic lesion

A) intraperitoneal free air A collection of air or gas in the peritoneal cavity, which can be caused by surgical perforation, ruptured intestine, or penetrating trauma. CT images viewed in the lung window demonstrates good subject contrast between oral contrast in the bowel and air within the cavity. In many cases, scanning in the decubitus position aids the radiologist in determining the exact location.

What pathological condition is demonstrated in the figure? a) intraperitoneal free air b) small bowel obstruction c) peritoneal metastases d) colitis

D) horseshoe kidney A common renal fusion anomaly in which two functioning kidneys on each side of the midline are connected by an isthmus of functioning renal parenchyma. The isthmus crosses midline between the aorta and the inferior vena cava.

What pathological condition is demonstrated in the figure? a) pyelonephritis b) vascular injury c) renal cyst d) horseshoe kidney

C) kidney laceration Low density areas of the parenchyma of the left kidney consistent with a deep laceration and hematoma. The hematoma is depicted by its hypodense appearance.

What pathological condition is demonstrated in the figure? a) renal cell carcinoma b) pyelonephritis c) kidney laceration d) angiomyolipoma

B) lymphoma Affected lymph nodes have the same attenuation value as muscle tissue. Lymphomas tend to form conglomerate masses, which may encase the mesenteric vessels known as the "sandwich" sign.

What pathological condition is demonstrated in the figure? a) soft tissue sarcoma b) lymphoma c) peritonitis d) abdominal aortic aneurysm

D) angiomyolipoma Common benign renal tumors composed of blood vessels, smooth muscle, and fat. The detection of fat within these renal masses confirms their diagnosis and distinguishes them from renal cell carcinoma.

What pathological condition is demonstrated in the figure? a) soft tissue sarcoma b) lymphoma c) peritonitis d) angiomyolipoma

C) emphysema Free air trapped within the lungs during the inhalation process with the inability to be exhaled.

What pathological syndrome is depicted in the figure? a) pleural effusion b) asbestosis c) emphysema d) pneumothorax

B. coronal

What plane is the MPR depicted? A) sagittal B) coronal C) oblique D) curved

C. Volume rendering VR is a 3D semistransparent representation of the imaged structure. It as become the favored 3D image technique because all voxels contribute to the image.

What postprocessing technique created the image of the heart? A) MinIP B) MIP C) volume rendering D) endoluminal imaging

C. iodine concentration The iodine atoms in the contrast material are exclusively responsible for an enhanced structure's increased beam attenuation capacity. The degree to which this occurs is directly related to the concentration of iodine in the contrast material.

What property of intravenous contrast media is responsible for its capacity to increase a structure's ability to attenuate the x-ray beam? a. osmolality b. viscosity c. iodine concentration d. ionic nature

A) Angular-Longitudinal tube current modulation ***Feature incorporates the properties of both angular and longitudinal tube current modulation. This technique adjusts the dose according to the patient-specific attenuation in all three planes***

What radiation dose saving technique adjusts the tube current based on the patients overall attenuation by modulating the tube current in the angular (X-Y) and longitudinal (Z) dimensions to adapt to the patients shape? A) Angular-longitudinal tube current modulation B) Prospective ECG triggering C) High-resolution CT D) Automated tube current modulation

C. reconstruction algorithm *It is important to differentiate reconstruction algorithms from setting a window width and level.* Changing the window setting merely changes how the image is viewed. Changing the reconstruction algorithm will change the way the raw data are manipulated to reconstruct the image.

What scan parameter determines how the raw data are filtered in the reconstruction process? A) window width B) window level C) reconstruction algorithm D) kVp

C) Horizontal fissure ***is the division of the upper and middle lobes from the lower lobe of the right lung****

What space separates the upper and middle lobes from the lower lobe of the right lung? A) Pleural cavity B) Oblique fissure C) Horizontal fissure D) Hilum

B) Medulla Oblongata ***Is located in the lower brain stem just below the pons. Its the communication between the bran and the spinal cord***

What structure forms the lower portion of the brain stem containing white matter verve fibers communicating between the brain stem and the spinal cord? A) Pons B) Medulla oblongata C) Lenticular nucleus D) Extreme capsule

B) Anterior horn of the lateral ventricle ***Anterior horn of the lateral ventricles are part of the lateral ventricles are found within the frontal lobe. The roof is formed by the corpus callosum, the floor and lateral wall are formed by the head of the caudate nucleus, and the medial wall is formed by the septum pellucidum***

What structure is adjacent to the head of the caudate nucleus? A) Thalamus B) Anterior horn of the lateral ventricle C) Posterior horn of the lateral ventricle D) Pons

B) Anterior horn of the lateral ventricle ***Anterior horn of the lateral ventricles are part of the lateral ventricles are found within the frontal lobe. The roof is formed by the corpus callosum, the floor and lateral wall are formed by the head of the caudate nucleus, and the the medial wall is formed by the septum pellucidum***

What structure is adjacent to the head of the caudate nucleus? A) Thalamus B) Anterior horn of the lateral ventricle C) Posterior horn of the lateral ventricle D) Pons

D) Midbrain, Pons, and the Medulla Oblongata ***The brain stem is the posterior part of the brain, adjoining the structurally continuous with the spinal cord.***

What structures make up the brain stem? A) Cerebellar tonsils and thalamus B) Thalamus, midbrain, and the medulla oblongata C) Thalamus, Midbrain, and the Pons D) Midbrain, Pons, and the medulla oblongata

B) mAs ***Quantity refers the number of photons. tube current in CT is calculated by the product of the true mA and the rotation time***

What technical factor determines the quantity of photons reaching the detectors? A) kVp B) mAs C) Pitch D) Filters

B) mAs ***mAs is directly proportional to dose***

What technical factor would you change to reduce radiation dose? A) kVp B) mAs C) Collimation D) Detector Configuration

B. acquire scans while the patient is in a right decubitus position When initial scans fail to differentiate the margins of the pancreas from the duodenum, the patient is often given additional oral contrast material and additional slices are obtained with the patient lying in a right decubitus position.

What technique is often used to differentiate the margins of the pancreas from the duodenum? A) water-soluble contrast medium administered by enema B) acquire scans while the patient is in a right decubitus position C) unenhanced imaging; no oral or intravenous contrast is used D) scan approximately 2 hours after the patient ingests 600mL of a barium sulfate oral contrast medium

C. collimation of the x-ray beam and width of the detectors in the z-axis MDCT systems modify many of the rules established with SDCT. MDCT contains multiple parallel rows of detector elements that can be combined in various ways to yield slices with different section thicknesses. In SDCT, slice thickness was controlled exclusively by the degree of physical collimation. But in MDCT, slice thickness is also affected by the width of the detectors in the z-axis.

What two factors control the slice thickness of an MDCT scanner? A) gantry rotation time and mAs setting B) table speed and kVp setting C) collimation of the x-ray beam and width of the detectors in the z-axis D) reconstruction algorithm and detector material

A. mA and scan time *The product of mA and the scan time (mAs or tube current), determines the amount/quantity of x-ray photons.* The kVp determines the quality (average photon energy) of the x-ray beam. Neither DFOV nor reconstruction algorithm have any effect on the x-ray beam; rather, they control how the raw data are back projected to create an image.

What two factors define the quantity of the x-ray energy? A) mA and scan time B) kVp and heat capacity C) reconstruction algorithm and heat dissipation D) DFOV and x,y coordinates

C. MIP (maximum intensity projection) MIPs examine each voxel along a line from the viewer's eye through the data set and selects only the voxel with the highest value for inclusion in the displayed image. This method tends to display bone and contrast-filled structures; lower-attenuation structures are not well visualized.

What type of 3D display is depicted? A) volume rendering B) surface rendering C) MIP (maximum intensity projection) D) MinIP

D) Beam hardening ***Beam hardening occurs due to laws of physics interaction with matter. As a beam passes through an object the main energy increases, because the lower energy photons are absorbed more than the higher energy photons producing a more penetrating beam of radiation.***

What type of artifacts is found in the pelvis running across the two femoral heads, especially when a prostheses is present? A) Edge gradient B) Aliasing C) Cone beam D) Beam hardening

D) Beam hardening ***Beam hardening occurs due to laws of physics interaction with matter. As a beam passes through an object the main energy increases, because the lower energy photons are absorbed more than the higher energy photons producing a more penetrating beam of radiation.***

What type of artifacts is found in the pelvis running across the two femoral heads, especially when a prostheses is present? A) Edge gradient B) Aliasing C) Cone beam D) Beam hardening

Direct

What type of current is required for proper operation of the x-ray tube?

C. bone algorithm This type of filtering is applied to scan data before back projection occurs. Many different filters are available that use different algorithms depending on which part of the data must be enhanced or suppressed. Bone (or high contrast algorithms) accentuate the difference between neighboring pixels to optimize spatial resolution, but must make sacrifices in low contrast resolution. These filters are most often used when there are great extremes of tissue density and when optima low contrast resolution is not necessary.

What type of filter attenuates the difference between neighboring pixels to enhance spatial resolution? A) bow-tie filter B) smoothing algorithm C) bone algorithm D) soft-tissue algorithm

A) Butterfly

What type of needle is equipped with a plastic projections on either side that aid in holding the needle during veinpuncture? A) Butterfly B) Small gauge straight needle C) Angiocatheter D) Large gauge straight needle

A. CTDI The computed tomography dose index (CTDI) can only be calculated if slices are contiguous, that is, there are no overlapping or gapped slices. The multiple scan average dose (MSAD) will allow for gapped or overlapping slice protocols. Although the dose-length product (DLP) more closely reflects the radiation dose for a specific CT examination, its value is affected by variances in patient anatomy. Therefore, CTDI is a more useful measurement for comparing radiation doses among different scanners and different protocols.

What type of radiation dose measurement is used when scanner manufacturers report the doses to the Food and Drug Administration and prospective clients? a. CTDI b. MSAD c. DLP d. rem

A) Simple Expressed Consent

What type or consent is obtained prior to a noninvasive CT procedure? A) Simple expressed B) Implied C) Written informed D) Informed

B) Femoral Vein ***Originates from smaller vessels withing the thigh that extend through the thigh that extends through the thigh to terminate in the external iliac veins as it passes over the pubic bone to enter the pelvis**

What veins are located in the thigh that passes over the pubic bone to enter the pelvis? A) Common iliac vein B) Femoral Vein C) Internal Iliac vein D) Gluteal vein

B) Femoral Vein ***Originates from smaller vessels withing the thigh that extend through the thigh that extends through the thigh to terminate in the external iliac veins as it passes over the pubic bone to enter the pelvis**

What veins are located in the thigh that passes over the pubic bone to enter the pelvis? A) Common iliac veins B) Femoral Veins C) Internal Iliac veins D) Gluteal vains

C) Medial Cubital Vein ***Medial cubital vein ascends in an oblique and medial course to create an anastomosis between the basilic and cephalic vein***

What veins would you use for placing an IV catheter for a contrast-enhanced CT? A) Vein of Galen B) Basilar vein C) Medial cubital vein D) Radial vein

C) Brachiocephalic Artery ***Brachiocephalic artery is the first major branch off the aorta on the right side. It divides into the right subclavian and right common carotid artery***

What vessel branches off the aortic arch and extends to the right and divides into the right subclavian and the right common carotid? A) Ascending Aorta B) Descending Aorta C) Brachiocephalic Artery D) Right Vertebral Artery

C) Brachiocephalic Artery ***Brachiocephalic artery is the first major branch off the aorta on the right side. It divides into the right subclavian and right common carotid artery***

What vessel branches off the aortic arch and extends to the right and divides into the right subclavian and the right common carotid? A) Ascending Aorta B) Descending Aorta C) Brachiocephalic Artery D) Right Vertebral Artery

B. the incorrect preparation and inadequate connection of the injector syringe and tubing Large air embolism after contrast administration by means of a mechanical injector can only occur as a result of human error. Although butterfly infusion sets and straight needles are not recommended when using a mechanical injector, this is because of the increased risk of contrast extravasation (not air embolism). Although technologists should make every attempt to remove air bubbles from the syringe and tubing, small quantities of air can be absorbed by the body without harm.

When a mechanical injector is used, a large air embolism can result from: A) a malfunctioning pressure limit switch B) the incorrect preparation and inadequate connection of the injector syringe and tubing C) the use of butterfly infusion sets or straight needles D) tiny undetected bubbles in the contrast medium

B) The vein on the contralateral side. ***removing the axillary lymph nodes increase the risk for developing lymphedema.***

When administering IV contrast to a patient with a mastectomy, which vein should be used? A) The vein on the same side B) The vein on the contralateral side C) The dorsal pedal veins D) Mastectomy patients should never receive intravenous contrast.

B) Adenoma ***Adrenal adenoma is a common benign tumor arising from the cortex of the adrenal gland.***

When an adrenal lesion measures less than -20 HU, its said to be what kind of lesion? A) Cystic lesion B) Adenoma C) Metastatic D) Lymphoma

C. A smaller number of detectors ar activated The scan field of view (SFOV) size is determined by the number of detectors activated along the in-plane (x-, y-) axes during data acquisition.

When an operator reduces the scan field of view (SFOV) for a particular body part, which of the following technical changes occurs? a. The displayed image appears larger b. Spatial resolution increases c. A smaller number of detectors are activated d. The displayed image appears smaller

D. 1, 2 and 3 -- diameter of data acquisition, number of activated detectors within the array and correction factors for the specific area of anatomic interest Each CT scanner has several choices for scan field of view (SFOV). The choice made by the technologist activates a certain percentage of the detector array so that information is acquired from a circular portion of the anatomic section. Built into the SFOV selection are additional correction factors used to process different types of tissue. A CT scanner may have a specific selection for scans of the head, which attempts to limit the artifact occurring at the delineation of bone from bone tissue.

When choosing a scan field of view (SFOV), the CT technologist is controlling the: 1. diameter of data acquisition 2. number of activated detectors within the array 3. correction factors for the specific area of anatomic interest a. 2 only b. 1 and 2 only c. 1 and 3 only d. 1, 2 and 3

D. when a survey of an area is needed Gapped images are taken when representative slices are sufficient and imaging every part of the region is not required. Because some areas are not exposed, studies comprising gapped slices will reduce the radiation dose to the patient.

When do scan protocols incorporate the use of gapped slices? A) when the object of interest is very small (>2mm) B) when the metallic objects are present in the area scanned C) when the patient is unable to hold still for the duration of the scan D) when a survey of an area is needed

A. placing a foam spacer between the patient surface and the bismuth shield A thin foam spacer may be placed between the patient surface and the bismuth shield to reduce the scatter artifact that may occur when using in-plane bismuth shielding.

When employing bismuth shielding, which of the following techniques will reduce streaking artifact while still offering radiation dose savings to sensitive tissues? A) Placing a foam spacer between the patient surface and the bismuth shield B) Placing the bismuth shielding outside of the scan acquisition range C) Placing the bismuth shielding after the acquisition of the localizer image D) Placing a foam spacer on top of the bismuth shield to intercept the incoming beam

Congestive heart failure

When evaluating a patient's lab results, an elevated creatinine level is an indication of what medical condition?

B. 70 bpm Motion artifact can be a problem when the patient's heart rate is greater than 70 bpm, or if the heart rate is irregular. When imaging larger cardiac structures, heart rate is less of an issue, although a slower, regular heart rate is still preferred.

When imaging the heart, particularly when the structures of interest are small, such as the coronary arteries, a regular heart rate of less than _____________ is important. A) 50 bpm B) 70 bpm C) 90 bpm D) 110 bpm

B. tube milliamperage (mAs) Protocol optimization must include size-based dose adjustments by reducing the tube current (mAs). Optimal scans of pediatric patients can be achieved with substantially reduced mAs.

When implementing a scan protocol for a CT of the abdomen without contrast on a pediatric patient (40 lb/18 kg), which technical factor should be primarily adjusted? a. detector configuration b. tube milliamperage (mAs) c. scan field of view (SFOW) d. reconstruction algorithm

A. 10% The limiting resolution of a particular CT scan is determined at a point on the graph where the signal frequency corresponding to a particular object has reached 10%. When the modulation transfer function (MTF) is lower than 10% (0.1), the object is no longer resolved.

When one is assessing a modulation transfer function (MTF) graph such as the figure, the limiting resolution is determined at a point where the signal frequency corresponding to a particular object has reached: a. 10% b. 20% c. 50% d. 100%

A. the maximum attenuation occurring within the voxel Maximum intensity projection (MIP) images display only the maximum pixel value along a ray traced through the object to the viewer's assumed perspective in front of the viewing monitor. As the viewer faces an MIP image, each pixel represents the maximum attenuation that occurs in the associated voxel.

When one is viewing a maximum intensity projection (MIP) image, each pixel represents: A) the maximum attenuation occurring within the voxel B) the average attenuation occurring within the voxel C) the minimum attenuation occurring within the voxel D) all attenuation occurring within the voxel above a set threshold value

C. the minimum attenuation occurring within the voxel Minimum intensity projection (min-IP) images display the minimum pixel value along each ray to the viewer. As the viewer faces a min-IP image, each pixel represents the minimum attenuation that occurs in the associated voxel. Min-IP reformations are used primarily during the evaluation of the biliary tree, colon, lungs and trachea.

When one is viewing a minimum intensity projection (min-IP) image, each pixel represents: A) the maximum attenuation occurring within the voxel B) the average attenuation occurring within the voxel C) the minimum attenuation occurring within the voxel D) all attenuation occurring within the voxel above a set threshold value

B. the average attenuation occurring within the voxel A standard multiplanar reformation (MPR) image is 1 voxel thick, with the pixels facing the viewer, each representing the average attenuation occurring within the represented voxels.

When one is viewing a multiplanar reformation (MPR) image, each pixel represents: A) the maximum attenuation occurring within the voxel B) the average attenuation occurring within the voxel C) the minimum attenuation occurring within the voxel D) all attenuation occurring within the voxel above a set threshold value

D) Time for normal clot to formation in plasma. **The PTT range is 30-45 seconds. Patients on blood thinner have a 1.5-2.5 seconds of stopping time***

When performing an invasive CT exam, such as RFA of the liver, a partial thromboplastin time (PTT) is drawn. What does the partial thromboplastin time (PTT) time measure? A) the time required to increase the platelet count by 10% B) Time required to manufacture prothrombin C) Time required to locate an decrease the platelet count by 10% D) Time for normal clot to formation in plasma

Direct effect

When radiation strikes DNA, what occurs?

C. at least 6 hours before contrast administration Steroids must be given a minimum of 6 hours (but preferably 12 hours) before the examination. Probably the most common premedication regimen for adult patients calls for three 50-mg doses of prednisone, beginning 12 hours before the schedule examination and given every 6 hours. Often a single 50-mg dose of diphenhydramine (Benadryl) is also given shortly before the examination.

When the decision to premedicate is made by the radiologist, it is important that the steroids be started: a) 30 minutes before contrast administration b) at least 2 hours before contrast administration c) at least 6 hours before contrast administration d) 72 hours before contrast administration

A. inside the gantry The high-frequency generator used to produce the three-phase power used in modern CT (located in the gantry) may be positioned in a corner of the gantry or fixed to the rotating tube assembly.

Where is the high frequency generator often located in a modern CT scanner? a. inside the gantry b. just outside the scan room c. beneath the CT gtable d. inside the operator's console

C) shaded surface display The principle behind the shaded surface rendering is to realistically produce a 3D scene of the surface of a structure of interest from the acquired data set.

Which 3D rendering technique is used in the above figure? a) volume rendered technique b) multiplanar reformations c) shaded surface display d) maximum intensity projection

A. ROM, RAM Primary storage refers to the computer's input/output channels. Primary memory is used to store data that are likely to be in active use.

Which are parts of the system's primary memory? a. ROM, RAM b. SAM, CD-R c. MOD, thumb drive d. DVD-R, magnetic tape

B) motion

Which artifact(s) are represented in the figure? a) cone beam b) motion c) partial volume averaging d) metal

B. dosimetric data Responsibility for performing and documenting quality control tests is often shared between CT technologists and medical physicists. However, *a medical physicist must perform dose measurements.*

Which aspect of the CT quality control program must be performed by a medical physicist? A) spatial resolution measurements B) dosimetric data C) noise and uniformity D) all aspects of the CT quality control program must be performed by a medical physicist

C) Myocardium ***Mycardium is the middle layer of the 3 layers of the heart. This layer is composed of a thick muscular muscle that contracts with each heartbeat. 10% of the total cardiac blood volume of each heartbeat is required solely for supply to the heart muscle***

Which chamber of the heart consists of the strongest cardiac muscle? A) Pericardium B) Epicardium C) Myocardium D) Endocardium

C) Myocardium ***Myocardium is the middle layer of the 3 layers of the heart. This layer is composed of a thick muscular muscle that contracts with each heartbeat. 10% of the total cardiac blood volume of each heartbeat is required solely for supply to the heart muscle***

Which chamber of the heart consists of the strongest cardiac muscle? A) Pericardium B) Epicardium C) Myocardium D) Endocardium

C) Right Ventricle ***Right ventricle lies on the diaphragm and is the largest portion of the anterior surface of the heart. It receives deoxygenated blood from the right atrium and forces it into the pulmonary trunk via the semilunar valves***

Which chamber of the heart pumps blood into the pulmonary trunk through the pulmonary semilunar valve? A) Left Ventricle B) Left Atrium C) Right Ventricle D) Right Atrium

C) Right Ventricle ***Right ventricle lies on the diaphragm and is the largest portion of the anterior surface of the heart. It receives deoxygenated blood from the right atrium and forces it into the pulmonary trunk via the semilunar valves***

Which chamber of the heart pumps blood into the pulmonary trunk through the pulmonary semilunar valve? A) Left Ventricle B) Left Atrium C) Right Ventricle D) Right Atrium

B. mAs = 200; kVp = 120; pitch = 1.8; 5-mm slice thickness In general, increasing radiation dose results from increasing mAs, increasing kVp, decreasing pitch, and decreasing slice thickness. Although choice A includes the lowest mAs, it is more than offset by the increased kVp and decreases in pitch and slice thickness.

Which combination of factors is likely to result in the lowest radiation dose to the patient? A) mAs = 180; kVp = 140; pitch = 0.8; 2-mm slice thickness B) mAs = 200; kVp = 120; pitch = 1.8; 5-mm slice thickness C) mAs = 280; kVp = 120; pitch = 1.5; 2-mm slice thickness D) mAs = 300; kVp = 120; pitch = 1.0; 5-mm slice thickness

D. out-of-field artifact Occurs when portions of the patient lie outside the selected scan field of view.

Which image artifact can result from an improperly centered patient? A) edge gradient artifact B) undersampling artifact C) tube arc artifact D) out-of-field artifact

C. although no definite risk to the fetus has been identified, not enough studies have been done to conclude that there are no risks It is recommended that if a patient is known to be pregnant, both the potential radiation risk and the potential added risks of contrast media should be considered before proceeding with the study. The radiologist should confer with the patient's referring physician.

Which is a TRUE statement regarding the risk of iodinated contrast media to the fetus? A) at the usual clinical doses, iodinated contrast media does not cross the human placenta and therefore will not enter the fetus B) many studies have been done on the effects of iodinated contrast media in pregnant women C) although no definite risk to the fetus has been identified, not enough studies have been done to conclude that there are no risks D) fetal abnormalities have been reported in pregnant patients who were given iodinated contrast media at the usual clinical dose

C. an indwelling catheter set with a flexible plastic cannula should be used, preferably 20-gauge or larger Coronary angiograms require the use of mechanical injectors. Whenever these injectors are used both straight needles and butterfly infusion sets should be avoided to reduce the risk of contrast extravasation. A Huber needle is a noncoring hooked needle used for accessing implantable ports.

Which is a TRUE statement regarding venipuncture in a peripheral vein for the placement of an IV line for a CT coronary angiogram? A) a 22-gauge butterfly infusion set should be used B) an 18-gauge straight needle should be used C) an indwelling catheter set with a flexible plastic cannula should be used, preferably 20-gauge or larger D) a Huber needle should be used

C. they deliver the same amount of iodine at a lower flow rate Higher concentration agents are particularly well-suited when multislice scanners are used and the scan duration is quite short.

Which is an advantage of using higher concentration (>350mg iodine/mL) agents? A) they possess lower osmolality, so are associated with fewer adverse effects B) they produce fewer high-contrast (streak) artifacts in the injected vein C) they deliver the same amount of iodine at a lower flow rate D) they are of lower viscosity, so are less likely to remain in the venous injection path, and therefor, a saline flush is not necessary

C. compact disks Offline storage refers to data that is kept in a location not immediately accessible and requires manual intervention to use. Offline storage can be time-consuming because it requires personnel to locate the appropriate disc, and then load it into the reader.

Which is an example of an offline archiving device? A) hard drive B) optical jukebox C) compact disks D) RAID

B) Nonionic contrast is considered a low-osmolality contrast media that does not separate in water. ***Iodine concentration is maintained without increasing the number of particle in solution. therefore it does not increase the osmolality of the blood serum and does not change the osmotic pressure in the blood stream.***

Which is of the following is a property of nonionic contrast? A) It contains a negatively charged anion that consists of a benzene ring with a negatively charged amino acid group B) Iodine concentration is maintained without increasing the number of particles in solution: Therefor it does not increase the osmolality of the blood serum and doe not change the osmotic pressure in the bloodstream. C) Renal effects are especially significant D) May result hypervolemic shock

A) Higher Osmolality ***Ionic contrast is considered a high-osmolality contrast media (HOCM) which separates in water yielding two particles***

Which is the following is a characteristic of ionic contrast media? A) Higher osmolality B) Does not increase the osmolality of the blood serum C) Does not separate in water D) Ratio of iodine atoms to active particles is 3:1

B) Annular Ligament ***the narrow portion tightens around the radial neck to prevent inferior displacement of the radius. This ligament is an important structure of the elbow joint due to it allows the radius to rotate freely***

Which ligament in the upper extremity forms a ring that encircles the radial head? A) Radial collateral ligament B) Annular ligament C) Supraspinatus ligament D) Ulna collateral ligament

B. 3

Which number in the figure corresponds to the trapezius muscle? a. 4 b. 3 c. 6 d. 5

b. 5

Which number on the figure corresponds to the anterior horn of the lateral ventricle? a. 1 b. 5 c. 3 d. 2

B. 2

Which number on the figure corresponds to the internal capsule? a. 1 b. 2 c. 4 d. 5

C. 4

Which number on the figure corresponds to the left common carotid artery? a. 1 b. 6 c. 4 d. 3

A. 1

Which number on the figure corresponds to the pedicle? a. 1 b. 4 c. 2 d. 3

c. 4

Which number on the figure corresponds to the septum pellucidum? a. 2 b. 3 c. 4 d. 5

C. 1

Which number on the figure corresponds to the superior mesenteric vein? a. 3 b. 5 c. 1 d. 2

C. 4

Which number on the figure corresponds to the superior vena cava? a. 2 b. 3 c. 4 d. 5

C. 2

Which number on the figure corresponds to the zygomatic bone? a. 3 b. 1 c. 2 d. 6

C. Perspective volume rendering PERSPECTIVE VOLUME RENDERING provides a viewpoint of being within the lumen of the object, similar to an endoscopic view. IMMERSIVE RENDERING is a type of 3D reconstruction commonly used during CT colonography and CT bronchography.

Which of the following 3D imaging techniques is utilized to provide "fly-through" images during CT colonoscopy? a. orthographic volume rendering b. shaded surface display (SSD) c. perspective volume rendering d. minimum intensity projection (min-IP)

C. CT of brain to R/O subdural hematoma Subdural hematomas increase in CT number and may be well visualized without the IV administration of an iodinated contrast agent.

Which of the following CT studies of the head is typically performed without a contrast agent? a. CT angiogram for circle of Willis b. coronal scan to R/O pituitary tumor c. CT of brain to R/O subdural hematoma d. CT of brain to R/O metastatic disease

B. Bolus technique Bolus administration of contrast agent requires the entire volume of material be injected over the shortest possible time.

Which of the following IV contrast agent administration methods provide the greatest overall plasma iodine concentration? a. drip infusion b. bolus technique c. biphasic technique d. CT portography

C. virtual bronchoscopy CT bronchography is a three-dimensional examination of the tracheobronchial tree. It may include detailed endobronchial views, on which the viewer has the perspective of flying through the trachea and bronchi, as in conventional bronchoscopy. These specialized 3D reconstructions are commonly referred to as VIRTUAL BRONCHOSCOPY.

Which of the following MDCT examinations may include endobronchial views? A) virtual colonoscopy B) high-resolution CT (HRCT) of the lungs C) virtual bronchoscopy D) computed tomography angiogram (CTA) of the aorta

B) Linearity ***Refers to the relationship of the CT number of the reconstructed object to the measured linear attenuation coefficient. Linearity is measured using Catphan phantom with distinctive materials representing bone, contrast, fat, water, and air.***

Which of the following QA test requires the use of a Catphan phantom with inserts representing specific body densities? A) Spatial resolution B) Linearity C) Noise D) CT number

B) Linearity ***Refers to the relationship of the CT number of the reconstructed object to the measured linear attenuation coefficient. Linearity is measured using Catphan phantom with distinctive materials representing bone, contrast, fat, water, and air.***

Which of the following QA test requires the use of a Catphan phantom with inserts representing specific body densities? A) Spatial resolution B) Linearity C) Noise D) CT number

B. PACS Picture Archival and Communication Systems (PACS) are computerized networks whose responsibility is to store, retrieve, distribute and display CT and other imaging medical images.

Which of the following abbreviations is used to identify a computerized network that stores, retrieves, communicates and displays digital medical images? a. WORM b. PACS c. DICOM d. HIPAA

C. 3.75mm sections reconstructed every 3.75mm Contiguous images are those acquired with equal section thickness and interval.

Which of the following acquisitions may be characterized as contiguous? A) 2.5mm sections reconstructed every 1.25mm B) 5.0mm sections reconstructed every 7.5mm C) 3.75mm sections reconstructed every 3.75mm D) 20 second cine acquisition with 1.25mm sections

C. decrease display field of view (DFOV) Referred to as TARGETING, decreasing the DFOV causes an increase in the image size on the monitor. The DFOV controls the amount of scanned information to be displayed on the matrix. If a small portion of information is to be displayed on the entire matrix, it will appear larger to the viewer.

Which of the following actions would serve to magnify the CT image on the display monitor? a. decrease matrix size b. increase scan field of view c. decrease display field of view d. increase display field of view

C) Window Control ***Window width controls the contrast of the image by selecting a specific range of shades of gray from the Hounsfield scale displayed on the CT monitor***

Which of the following allows the technologist to alter the displayed picture contrast? A) Increasing kVp B) Decreasing mAs C) Window controls D) Retrospective reconstruction

D. liver Peritoneal cavity is formed by the membranous sac call the PERITONEUM. The PERITONEUM contains the stomach, liver, gallbladder, spleen, ovaries, transverse colon, and most of the small bowel.

Which of the following anatomic organs is located within the peritoneal cavity? a. pancreas b. kidneys c. uterus d. liver

C. 1 and 2 only Twenty-gauge and larger (18, 16, 14, ect.) should be utilized for flow rates EXCEEDING 3 mL/sec. 22-gauge angiocatheters are sufficient for flow rates UP TO 3 ml/sec.

Which of the following angiocatheter sizes may be safely used for the automated power injection of iodinated contrast agents at flow rates higher than 3 mL/sec? 1. 18 gauge 2. 20 gauge 3. 22 gauge a. 1 only b. 3 only c. 1 and 2 only d. 1, 2 and 3

C. edge gradient effect Occurs when the CT x-ray beam passes through areas of abrupt changes in density that are represented by high spatial frequencies. This type of streak artifact commonly occurs at the interface of dense bone and soft tissue in anatomic areas such as the brain.

Which of the following artifacts is evident on the image in the figure? a. blooming b. aliasing c. edge gradient effect d. ring artifact

D. ring Ring artifacts are caused by detector malfunctions and are beyond the technologists' control. Patient motion, partial volume averaging and edge gradient artifacts can all be limited by the CT technologist through adequate preparation and careful scan procedures.

Which of the following artifacts is not affected by the CT technologist? a. motion b. partial volume c. edge gradient d. ring

D. cone shaped Multidetector CT (MDCT) utilizes a cone shaped beam that is incident upon an expanded array of detectors. The detector array consists of multiple rows along the z-axis and requires a cone-shaped beam to measure incident radiation.

Which of the following best describes the shape of the x-ray beam utilized in multidetector CT (MDCT)? a. pencil beam b. fan shaped c. electron beam d. cone shaped

C. sagittal multiplanar reformation (MPR)

Which of the following best describes the type of image displayed in the image? A) sagittal maximum intensity projection (MIP) B) direct sagittal acquisition C) sagittal multiplanar reformation D) volume-rendered 3D

A. coronal maximum intensity projection (MIP) MIP images are a routine component of CTA examinations of the chest. This CTA of the pulmonary arteries (CTPA) demonstrates the well-opacified pulmonary vasculature in coronal thick-slab format. From the perspective of the viewer, each pixel displays the maximum attenuation value contained within the thickness of the slab.

Which of the following best describes the type of image displayed on the figure? A) coronal maximum intensity projection (MIP) B) direct coronal acquisition C) coronal multiplanar reformation (MPR) D) coronal minimum intensity projection (min-IP)

D. oblique sagittal MPR The image is a MPR image through the orbit in the oblique sagittal plane.

Which of the following best describes the type of image featured in the figure? A) direct coronal acquisition B) sagittal maximum intensity projection (MIP) C) coronal multiplanar reformation (MPR) D) oblique sagittal MPR

D. lungs The lungs are considered to have high inherent contrast because they are primarily composed of air and provide a background that makes nearly any object contained within the lungs easy to see. On the contrary, the physical properties of the body's soft tissues and many other organs possess low inherent contrast, often making it difficult to differentiate adjacent structures; to increase the inherent contrast of these organs, iodinated contrast medium is often administered.

Which of the following can be described as possessing high inherent contrast? A) soft tissue B) kidneys C) liver D) lungs

B) Scaphoid ***Approximately 70% of all carpal bones fractures involve the scaphoid bone***

Which of the following carpal bones is the most frequently fractured? A) Triquetral B) Scaphoid C) Trapezoid D) Trapezium

D. 1, 2 and 3 -- using in-plane bismuth shielding with longitudinal and angular tube current modulation (ATCM), increasing the tube potential (kVp) without a compensatory decrease in tube current (mA) and increasing technical parameters to produce a noise-free image Using in-line bismuth shielding with a CT system that employs real-time (angular and longitudinal) automatic tube current modulation can result in an increase in patient exposure as the system inappropriately measures patient density to include the shielding material. Increases in tube potential can result in an overall decrease in patient exposure if the tube current (mA) is decreased appropriately. To reduce patient dose, the reconstructed CT image should contain a tolerable amount of noise.

Which of the following clinical scenarios could result in an unnecessary increase in radiation dose to the patient? 1. using in-plane bismuth shielding with longitudinal and angular tube current modulation (ATCM) 2. increasing the tube potential (kVp) without a compensatory decrease in tube current (mA) 3. increasing technical parameters to produce a noise-free image a. 3 only b. 1 and 2 only c. 2 and 3 only d. 1, 2 and 3

D. 2 and 3 only -- noise and uniformity Noise is most commonly measured by scanning a water-filled phantom with a consistent set of technical factors. The image noise is equal to the standard deviation of pixel values within a region of interest (ROI) measurement of the image. Uniformity may be evaluated by positioning several ROI measurements at different locations along the center and periphery of the image. The CT values should not differ by more than 2 HU from one location to another.

Which of the following components of CT image quality are being evaluated in the figure? 1. low-contrast detectability 2. noise 3. uniformity a. 2 only b. 1 and 2 only c. 1 and 3 only d. 2 and 3 only

B. 1 and 2 only -- 400-600 mL of oral contrast agent 45-90 minutes before the exam and 250 mL of oral contrast agent immediately before the exam Generally, 400-600 mL of oral contrast administered 45-90 minutes before the exam should opacify the small bowel. An additional dose of contrast agent (250 mL) immediately before the examination should ensure gastric opacification. Administration of contrast agents via enema is indicated for evaluation of the distal large bowel.

Which of the following contrast agent administration techniques should be used for a general CT survey of the abdomen? 1. 400-600 mL oral contrast agent 45-90 minutes before the exam 2. 250 mL oral contrast agent immediately before the exam 3. 700-900 mL contrast agent administered as an enema immediately before the exam a. 1 only b. 1 and 2 only c. 1 and 3 only d. 1, 2 and 3

D. 1, 2 and 3 -- diatrizoate meglumine (Gastrografin), effervescent agents and iopamidol (Isovue) Opacification of the stomach and bowel loops is imperative during general CT examinations of the abdomen and pelvis. Effervescent agents may be used to ensure proper gastric distention. Intravenous contrast agents are used to opacify blood vessels and are helpful in anatomic differentiation and evaluation of mass vascularity.

Which of the following contrast media may be utilized during a CT evaluation of the gastrointestinal tract? 1. diatrizoate meglumine (Gastrografin) 2. effervescent agents 3. iopamidol (Isovue) a. 1 only b. 1 and 3 only c. 2 and 3 only d. 1, 2 and 3

D. Renal pyramid Renal pyramids are cone-shaped regions of the renal medulla. The wider base of the renal pyramids border the renal cortex.

Which of the following corresponds to the low-attenuation area indicated by number 3 on the figure? a. renal cortex b. renal pelvis c. renal calyx d. renal pyramid

D. slices can be combined to produce two 5-mm slices The thinnest that images can be reconstructed for a data set is determined by the slice thickness used for data acquisition. Images can be added together to create thicker slices for viewing, but in many cases data cannot be divided to produce thinner slices. On systems that will allow data to be divided, the slice can never be thinner than the smallest detector element.

Which of the following could be retrospectively reconstructed for MDCT data that acquires four 2.5-mm slices with each gantry rotation? A) slices can be divided to produce sixty-four 0.15-mm slices B) slices can be divided to produce thirty-two 0.31-mm slices C) slices can be combined to produce eight 5-mm slices D) slices can be combined to produce two 5-mm slices

D) 5

Which of the following demonstrates the posterior arch of C1 in the figure? a) 13 b) 4 c) 12 d) 5

D. Implied consent Occurs when a patient is in need of immediate medical services but is unconscious or physically unable to consent to treatment. The assumption is made that the patient would consent if able.

Which of the following describes the situation in which an assumption is made that an unconscious or otherwise physically unable patient consents to medical treatment? a. informed consent b. witnessed consent c. patient proxy d. implied consent

C. Ionization chamber A device used to accurately measure radiation exposure. Radiation causes ionization within the chamber, which is measured by an electrode. The amount of charged particles is proportional to the radiation exposure. Extremely accurate.

Which of the following devices is used to measure the patient dose from a CT examination? a. Geiger counter b. proportional counter c. ionization chamber d. film badge

A. 9.6 cm Targeted reconstructions should be performed bilaterally utilizing a small display field of view (DFOV) or a higher zoom factor to maximize resolution of the small bony components of the inner ear.

Which of the following display field of view (DFOV) values was most likely used to display the image? a. 9.6 cm b. 18.0 cm c. 25.6 cm d. 51.2 cm

C. kernal Voxel dimensions are determined by multiplying pixel dimensions by the slice thickness. Pixel dimensions are directly controlled by the matrix size and the field of view.

Which of the following factors does not affect the dimensions of a voxel? a. slice thickness b. matrix size c. kernel d. display field of view

D. 1, 2 and 3 -- Age, sex and race GFR is calculated using the patient's measured serum creatinine level and takes into account the patient's age, sex and race.

Which of the following factors may affect a patient's calculated glomerular filtration rate (GFR)? 1. age 2. sex 3. race a. 1 and 2 only b. 2 and 3 only c. 1 and 3 only d. 1, 2 and 3

C. kVp The degree of attenuation depends on multiple factors, including the x-ray energy and the atomic structure and density of the exposed tissue.

Which of the following factors may affect the attenuation of an object during CT data acquisition? a. beam pitch b. mA c. kVp d. algorithm

D. increase mAs Because of the increased collimation, a narrower slice results in fewer x-ray photons. To compensate, mAs must be increased.

Which of the following factors should be adjusted when slice thickness is decreased from 10 to 3mm? A) increase display field B) decrease matrix size C) lower window level D) increase mAs

C) Measurement data ***Measurement data undergo pre-processing to correct any errors that may have been resulted from beam hardening, bad detectors, motion artifact, or interpolation artifacts. These corrections are made before the image reconstructed algorithm is applied.***

Which of the following forms of data is required to be pre-processed? A) Raw data B) Image data C) Measurement data D) Retrospective data

C) Measurement data ***Measurement data undergo pre-processing to correct any errors that may have been resulted from beam hardening, bad detectors, motion artifact, or interpolation artifacts. These corrections are made before the image reconstructed algorithm is applied.***

Which of the following forms of data is required to be pre-processed? A) Raw data B) Image data C) Measurement data D) Retrospective data

D. cone beam artifact Unique to the cone beam geometry of MDCT systems and typically manifest much like partial volume artifacts.

Which of the following image artifacts is unique to multidetector CT (MDCT) systems? a. step artifact b. aliasing artifact c. out of field artifacts d. cone beam artifact

B. surgical asepsis Insertion of any IV line requires the sue of sterile technique to prevent microorganisms from entering the bloodstream.

Which of the following infection control techniques is required at the site of an intravenous injection of iodinated contrast media? a. contact isolation b. surgical asepsis c. medical asepsis d. enteric precautions

B) Bremsstrahlung

Which of the following interactions is the primary source of diagnostic x-rays? A) Photoelectric effect B) Bremsstrahlung C) Compton D) Pair production

C) Radiologist is responsible for all testing ***A good quality assurance program requires the testing to be performed on a regular basis designated by the quality control team, the measurements should be interpreted upon completion of the test to assure the scanner is working according to its standers, along with good documentation.***

Which of the following is NOT a tenet of a good quality assurance program? A) Documentation B) Prompt interpretation C) Radiologist is responsible for all testing D) Performed on a regular basis

B) Calculates the mean pixel values in an area ***Cine viewing is an image display tool that allows the user to interpret a large amount of data sets. Its main advantages are that it provides a more accurate evaluation of structures, speeds up the interpretation time, and increases the efficiency of the PACS system.***

Which of the following is NOT an advantage of cine viewing? A) More accurate evaluation of structures B) Calculates the mean pixel values in an area C) Speeds up the interpretation time D) Increases the efficiency of the PACS system

B) Calculates the mean pixel values in an area ***Cine viewing is an image display tool that allows the user to interpret a large amount of data sets. Its main advantages are that it provides a more accurate evaluation of structures, speeds up the interpretation time, and increases the efficiency of the PACS system.***

Which of the following is NOT an advantage of cine viewing? A) More accurate evaluation of structures B) Calculates the mean pixel values in an area C) Speeds up the interpretation time D) Increases the efficiency of the PACS system

B. quality images are produced even if the patient is uncooperative Overt patient motion will create artifacts that can greatly degrade image quality. It is unlikely that a diagnostic examination can be produced in a patient unable to lie still for the length of time required to complete the test. Although newer scanners can often complete the arterial phase acquisition in just seconds, the patient must be able to lie still for that short time.

Which of the following is NOT an advantage of using helical CT for the diagnosis of PE? A) it is cost effective B) quality images are produced even if the patient is uncooperative C) it is often helpful in establishing an alternative diagnosis in the absence of PE D) it is less invasive than pulmonary angiography

B) Segmenting the voxel data ***CT artifacts seriously degrade the quality of a CT image, since they are described as inconsistencies between the reconstructed CT numbers in the image and the true attenuation coefficients of the object***

Which of the following is NOT considered a degradation of the reconstruction image due to an artifact? A) Degrading the image quality B) Segmenting the voxel data C) Affecting the perfectibility of detail D) Leading the misdiagnosis

C. the peripheral dose, or skin dose, of a body scan can be two to three times higher than the central dose As the scan field and patient size increase, the x-ray beam must travel a greater distance and penetrate more tissue to arrive at the center. The greater distance and increased thickness of the absorber provide more opportunity for the beam to be attenuated. Hence, the uniformity of the dose decreases as the scan field of view and patient thickness increase. Therefore, body scans are less uniform than head scans.

Which of the following is TRUE concerning CT dose uniformity? A) uniformity of dose increases as the scan field of view and patient thickness increase B) the dose in body scans is more uniform than that of head scans C) the peripheral dose, or skin dose, of a body scan can be two to three times higher than the central dose D) compared with children, adults can e expected to receive a more uniform dose

c. the molecules in an ionic contrast solution dissociate to form ions in water The term osmolality refers to the number of particles in solution compared with water. *The classification ionic or nonionic refers to whether the molecules of the solution form ions in a water solution.* Although many nonionic contrast agents are also low osmolality, that is not true of all nonionic agents. Hexabrix is an example of a contrast medium that has low osmolality but is ionic.

Which of the following is TRUE of an ionic contrast medium? A) all ionic contrast media have high osmolality B) ionic contrast media are typically more expensive than the nonionic types C) the molecules in an ionic contrast solution dissociate to form ions in water D) ionic contrast agents have fewer particles in solution than nonionic contrast

A) All photons have the same energy ***In a homogeneous beam (gamma ray) all the photons have the same energy***

Which of the following is a characteristic of a homogeneous radiation beam? A) All photons have the same enegery B) The quantity and quality of the beam changes as it passes through the absorber C) All photons have different energies D) Known as a polychromatic beam

A) All photons have the same energy ***In a homogeneous beam (gamma ray) all the photons have the same energy***

Which of the following is a characteristic of a homogeneous radiation beam? A) All photons have the same enegery B) The quantity and quality of the beam changes as it passes through the absorber C) All photons have different energies D) Known as a polychromatic beam

D) Less Toxic ***When a compound has lower osmolality it contains fewer particles thus resulting in fewer or less sever adverse reactions. Also low-osmolality agents tend to be less toxic to the system***

Which of the following is a characteristic of a low-osmolality contrast media? A) Can decrease blood supply to the kidneys B) Can cause hypervolemia C) Can cause shock in dehydrated patients D) Less toxic

D. pneumothorax One of the most common complications of CT-guided needle biopsy of the lung. Pneumothorax describes a collection of air in the pleural space. A pneumothorax causes a portion of the lung to collapse, often resulting in the placement of a chest tube to reinflate the lung.

Which of the following is a common complication of CT-guided biopsy of the lung? a. pulmonary embolism b. aspiration c. pneumoconiosis d. pneumothorax

D. 1, 2 and 3 -- cephalic, antecubital and basilic veins Common sites for IV contrast administration are the antecubital, basilic, cephalic and accessory cephalic veins.

Which of the following is a common site for the intravenous injection of iodinated contrast media? 1. cephalic vein 2. antecubital vein 3. basilic vein a. 1 only b. 3 only c. 2 and 3 only d. 1, 2 and 3

D) Pharynx *** Pharynx is the fibromuscular tube located within the neck. Its divided into 3 sections. Nasopharynx, oropharynx, and laryngopharynx***

Which of the following is a funnel-shaped fibro-muscular tube 12 cm in length that acts as an opening for both the respiratory and digestive systems? A) Epiglottis B) Auditory Tube C) Palatine tonsil D) Pharynx

D) Pharynx *** Pharynx is the fibromuscular tube located within the neck. Its divided into 3 sections. Nasopharynx, oropharynx, and laryngopharynx***

Which of the following is a funnel-shaped fibromuscular tube 12 cm in length that acts as an opening for both the respiratory and digestive systems?? A) Epiglottis B) Auditory Tube C) Palatine tonsil D) Pharynx

C. petrous bone A high-attenuation object is one that absorbs or scatters a high percentage of the x-ray beam. The trachea and lungs are composed mostly of air, which does not attenuate much of the x-ray beam. A renal cyst is fluid-filled and has low-attenuation. The petrous bone is dense, consisting of calcium and other dense minerals, and will attenuate much of the x-ray beam; it will be represented as a white area on the CT image.

Which of the following is a high-attenuation structure? a. renal cyst b. trachea c. petrous bone d. lung

A. cupping The cupping artifact that occurs at the superior portion of the skull is another form of partial volume artifact. The dense skull table averages with the low-attenuating brain tissue, causing an artifact by which the brain parenchyma may appear abnormally dense.

Which of the following is a manifestation of the partial volume artifact? a. cupping b. Hounsfield bar c. aliasing d. streaking

C. Iohexol Nonionic contrast include iopamidol (Isovue), iohexol (Omnipaque) and iopromide (Ultravist).

Which of the following is a nonionic contrast material? a. iodamide b. iothalamate c. iohexol d. diatrizoate

B. Intramuscular Parenteral administration: an injection of medication into the body. Common routes of parenteral administration: intramuscular, intravenous, intradermal and subcutaneous

Which of the following is a parenteral route of medication administration? a. sublingual b. intramuscular c. transdermal d. oral

C) Bronchospasm with significant hypoxia

Which of the following is a sign of an anaphylactic reaction? A) Diffuse urticarial B) Facial edema without dyspnea C) Bronchospasm with significant hypoxia D) Limited cutaneous edma

C. photodiode Scintillation crystals are used in cooperation with photodiodes in a scintillation-type CT detector. The photodiode is a solid-state device that absorbs the light flashes given off by the crystal. The photodiode then emits an electrical signal in response to this light.

Which of the following is a solid state device used to record the light flashes given off by a scintillation crystal? a. photomultiplier tube b. anode c. photodiode d. input phosphor

D. vomiting Mild adverse reactions to iodine: nausea, vomiting, mild urticaria, a warm flushed feeling Moderate adverse reactions: dyspnea Severe adverse reactions: pulmonary edema and shock

Which of the following is an example of a mild reaction to iodinated intravenous contrast media? a. dyspnea b. shock c. pulmonary edema d. vomiting

D. tiny, contrast-filled arteries that are just 1mm apart Spatial resolution is also called detail resolution. Spatial resolution is the system's ability to resolve, as separate forms, small objects that are very close together.

Which of the following is an imaging challenge that depends on spatial resolution? A) liver lesion that is surrounded by healthy live tissue B) distinguishing between the white matter and gray matter of the brain C) large abdominal abscess D) tiny, contrast-filled arteries that are just 1mm apart

A) Converts continuous signal into discrete numbers ***The transmitted data produce a signal at the detectors, which is an analog form. In order for the computer to process this information, it must be converted into discrete or binary numbers. This process is performed by the ADC.***

Which of the following is an important component of the ADC? A) Converts continuous signal into discrete numbers B) Second step in image digitization C) Converts digital signal into analog signal D) Assigns an integer to the brightness value that was sampled

A) Converts continuous signal into discrete numbers ***The transmitted data produce a signal at the detectors, which is an analog form. In order for the computer to process this information, it must be converted into discrete or binary numbers. This process is performed by the ADC.***

Which of the following is an important component of the ADC? A) Converts continuous signal into discrete numbers B) Second step in image digitization C) Converts digital signal into analog signal D) Assigns an integer to the brightness value that was sampled

D) Cardiopulmonary Arrest

Which of the following is considered a severs reaction to an iodinated contrast reaction? A) Dyspnea B) Itching C) Nasal Stuffiness D) Cardiopulmonary Arrest

C. 2 and 3 only -- Fourier reconstruction and Filter back projection

Which of the following is considered an analytic form of image reconstruction? 1. iterative technique 2. Fourier reconstruction 3. filter back projection a. 1 only b. 1 and 2 only c. 2 and 3 only d. 1, 2 and 3

D. aliasing artifact A type of streak caused when an insufficient number of views (data samples) is obtained during data acquisition. It is typically because of a technical failure of the CT system.

Which of the following is considered as equipment induced CT image artifacts? a. step artifact b. metallic artifact c. cupping artifact d. aliasing artifact

A. bed index Multiple scan average dose (MSAD) accounts for the effects of image spacing, or BED INDEX, on patient radiation dose during axial scanning. MSAD increases when slice thickness is greater than image spacing and decreases when slice thickness is less than the BED INDEX.

Which of the following is considered in the calculated of the multiple scan average dose (MSAD) for an axial (step-and-shoot) a. bed index b. gantry rotation time c. mA setting d. pitch

B. Iothalamate meglumine Ionic contrast media are salts consisting of sodium and/or meglumine.

Which of the following is considered one of the ionic radiopaque contrast media (RCM)? a. iohexol b. iothalamate meglumine c. iopamidol d. ioversol

A. Iodixanol Iodixanol (Visipaque) is an example of a nonionic iso-osmolar contrast agent. Iohexol (Omnipaque), iopamidol (Isovue), and ioversol (Optiray) are all nonionic low-osmolar contrast media (LOCM).

Which of the following is considered one of the iso-osmolar contrast media (IOCM)? a. iodixanol b. iopamidol c. iohexol d. ioversol

C) Scan field of view (SFOV) ***The SFOV is the region or volume for which complete data sets can be acquired. The size depends on the fan angle. The object to be scanned must be within the SFOV to avoid artifacts due to data inconsistencies***

Which of the following is considered the area within the gantry from which the raw data are acquired located at the isocenter of the gantry? A) Display field of view (DFOV) B) Reconstruction algorithm C) Scan field of view (SFOV) D) detector cell size

C) Dyspnea

Which of the following is deemed a moderate reaction to contrast media? A) Convulsions B) Seizures C) Dyspnea D) Nasal congestion

A) right hepatic flexure

Which of the following is demonstrated by #24 in the figure? a) right hepatic flexure b) gall bladder c) transverse colon d) ileum

B) middle ear

Which of the following is demonstrated by #50 in the figure? a) mastoid air cells b) middle ear c) semicircular canal d) internal auditory canal

B) right inferior pulmonary vein

Which of the following is demonstrated by label #52? a) left atrium b) right inferior pulmonary vein c) left ventricle d) right inferior pulmonary artery

C) left renal pelvis

Which of the following is illustrated by #23 in the figure? a) left renal artery b) left renal vein c) left renal pelvis d) cyst in the left kidney

D) clivus

Which of the following is illustrated by #41 in the figure? a) occipital condyle b) mastoid process c) temporal bone d) clivus

D) pancreas

Which of the following is illustrated by label #18 in the figure? a) portal vein b) splenic vein c) stomach d) pancreas

C) portal vein

Which of the following is illustrated by label #22 in the figure? a) celiac trunk b) splenic artery c) portal vein d) aorta

B) external carotid artery

Which of the following is illustrated by label #3 in the figure? a) internal carotid artery b) external carotid artery c) middle cerebral artery d) posterior communicating artery

B) maxillary sinus

Which of the following is illustrated by label #9 in the figure? a) ethmoid sinus b) maxillary sinus c) sphenoid sinus d) bony erosion of the orbital floor

B. prone high-resolution CT (HRCT) of the lungs The "triple rule-out" MDCT procedure is a comprehensive evaluation of the chest for cardiac and noncardiac pain. The single acquisition consists of a coronary computed tomography angiogram (CTA) for coronary artery disease, a CTA of the aorta for aneurysm, and a CTA of the pulmonary arteries for embolism.

Which of the following is not a component of the "triple rule-out" multidetector (MDCT) protocol for patients complaining of chest pain? A) computed tomography angiogram (CTA) of the pulmonary arteries B) prone high-resolution CT (HRCT) of the lungs C) CTA of the aorta D) coronary CTA

D. cecum The cecum is the proximal portion of the large bowel; where the small bowel and large bowel connect (the ileocecal valve). Esophagus, Stomach, Pylorus, Duodenal bulb, Small bowel - DJI (Duodenum, Jejunum, Ileum), Cecum, Appendix, Large bowel - (Ascending colon, Transverse colon, Descending colon), Sigmoid colon, Rectum, Anus

Which of the following is not a portion of the small bowel? a. duodenum b. jejunum c. ileum d. cecum

C. decreased risk of contrast-induced nephrotoxicity (CIN) Automatic power injectors DO offer shorter injection times, more consistent administration and greater enhancement. Automatic power injectors do NOT decrease the risk of CIN.

Which of the following is not an advantage of an automatic power injector over the manual bolus method of intravenous contrast agent administration? A) uniform contrast enhancement throughout the examination B) consistent contrast agent administration for all patients C) decreased risk of contrast-induced nephrotoxicity (CIN) D) shorter injection times

B. silver halide There are no silver halide crystals in CT detectors.

Which of the following is not commonly used as a CT scintillation detector material? a. ceramic rare earth b. silver halide c. bismuth germanate d. cadmium tungstate

B) Segmenting the voxel data ***CT artifacts seriously degrade the quality of a CT image, since they are described as inconsistencies between the reconstructed CT numbers in the image and the true attenuation coefficients of the object***

Which of the following is not considered a degradation of the reconstruction image due to an artifact? A) Degrading the image quality B) Segmenting the voxel data C) Affecting the perfectibility of detail D) Leading the misdiagnosis

B) Obtaining informed consent *** The Joint Commission recommends healthcare personal to utilize 2 form of identification, the patients full legal name and there birth date.

Which of the following is not part of the patient identification process? A) Establishing an initial rapport with the patient B) Obtaining informed consent C) Asking the patient to state his/her full legal name D) Asking the patient his/her date of birth

B) Patient must be adequately informed and sign a consent form

Which of the following is not required for valid consent? A)Patient must be of legal age B) Patient must be adequately informed and sign a consent form C) Patient must be mentally competent D) Consent must be offered voluntarily

A. 2 only -- z-filtering During CT image reconstruction using the back-projection method, the data are manipulated with a convolution kernel or algorithm to remove image unsharpness (blurring). this method is called the FILTERED BACK-PROJECTION.

Which of the following is sued to remove image blurring during the back-projection method of CT image reconstruction? 1. z-filtering 2. convolution kernel 3. Feldkamp-Davis-Kress (FDK) algorithm a. 2 only b. 3 only c. 1 and 2 only d. 2 and 3 only

C) Xenon ***Gas ionization detectors are filled with the noble or inert gas, xenon at high pressure. This is done to achieve high x-ray absorption***

Which of the following is the appropriate type of gas utilized in gas ionization detectors? A) Helium B) Argon C) Xenon D) Neon

C) Xenon ***Gas ionization detectors are filled with the noble or inert gas, xenon at high pressure. This is done to achieve high x-ray absorption***

Which of the following is the appropriate type of gas utilized in gas ionization detectors?? A) Helium B) Argon C) Xenon D) Neon

C) Thinner slice acquisitions ***Is to reduced the width of the slice.***

Which of the following is the best way of reducing the partial volume averaging artifact? A) Faster slice acquisitions B) Iterative reconstruction C) Thinner slice acquisitions D) Increased tube output

B. streaking The presence of metal within the patient causes a streak artifact on the image. The artifact occurs when the dense metal absorbs a large amount of the radiation, interfering with the signal produced.

Which of the following is the common term for the artifact present on the image in the figure? a. aliasing b. streaking c. edge gradient d. tube arcing

B. 15 degrees

Which of the following is the correct angle of insertion for placing a butterfly needle into a vein for intravenous administration of a contrast agent? a. 5 degrees b. 15 degrees c. 45 degrees d. 60 degrees

B) Humerus

Which of the following is the longest and largest bone of the upper extremity? A) Scapula B) Humerus C) Radius D) Ulna

B. wall thickening The most important identifying sign of gastrointestinal (GI) pathology on CT examination is wall thickening. The ability to evaluate the gastric and bowel walls is a crucial component of the successful CT evaluation of the GI system. Distention and opacification with oral media, coupled with contrast enhancement through intravenous administration, offer the best demonstration of the GI wall.

Which of the following is the most common sign of gastrointestinal (GI) pathology on CT images? a. fluid collection b. wall thickening c. air distention d. dense fecal matter

A. quantum noise The most common cause of CT image noise is the fluctuation in the number of x-ray photons measured by the detectors. When a CT scanner attempts to reconstruct an image from an insufficient amount of transmitted radiation measurements, quantum noise occurs.

Which of the following is the most common type of noise found in the CT image? a. quantum noise b. electronic noise c. artifactual noise d. filter noise

D. portal venous phase The preferred timing for CT acquisition of the spleen is the portal venous phase (60-70 seconds), when a more homogenous pattern of enhancement is demonstrated.

Which of the following is the preferred contrast enhancement phase for CT acquisition of the spleen? a. precontrast phase b. equilibrium phase c. arterial phase d. portal venous phase

A. 65 - 75 bpm Patients with slower heart rates exhibit longer diastolic phases, which yield higher-quality cardiac CT examinations. The preferred heart rate for optimal imaging on most MDCT systems is 65 beats per minute. Newer systems at the 64-slice level and beyond are capable of acquiring adequate cardiac CT images at higher heart rates.

Which of the following is the preferred range for patient heart rate for optimal cardiac CT studies? a. 65 - 75 bpm b. 75 - 85 bpm c. 85 - 95 bpm d. >100 bpm

Sphygmomanometer

Which of the following is used to measure blood pressure?

B. 1 and 2 only -- ovaries and Fallopian tubes Adnexa is used to describe an appendage of an organ. The ovaries and Fallopian tubes constitute the uterine adnexal region.

Which of the following is/are a component(s) of the adnexal area of the uterus? 1. ovaries 2. fallopian tubes 3. vagina a. 1 only b. 1 and 2 only c. 2 and 3 only d. 1, 2 and 3

A. 1 only -- anaphylaxis Severe reactions to iodinated contrast material: anaphylaxis, shock, cardiac arrest and death. Mild or moderate reactions to iodinated contrast material: urticaria and vomiting.

Which of the following is/are severe adverse reaction(s) to iodinated intravenous contrast media? 1. anaphylaxis 2. urticaria 3. vomiting a. 1 only b. 1 and 2 only c. 1 and 3 only d. 1, 2 and 3

D) Spatial resolution and temporal resolution ***Refers to the precision of a measurement with respect to time. Often there is a trade-off between temporal resolution and spatial resolution. fast acquisition result in less samples taken thus degrading spatial resolution***

Which of the following key performance parameters of image quality are considered indirectly proportional to each other? A) Spatial resolution B) Contrast resolution and linearity C) Temporal resolution and contrast resolution D) Spatial resolution and temporal resolution

D) Spatial resolution and temporal resolution ***Refers to the precision of a measurement with respect to time. Often there is a trade-off between temporal resolution and spatial resolution. Fast acquisition result in less samples (profiles) taken thus degrading spatial resolution***

Which of the following key performance parameters of image quality are considered indirectly proportional to each other? A) Spatial resolution and noise B) Contrast resolution and linearity C) Temporal resolution and contrast resolution D) Spatial resolution and temporal resolution

C. 1 and 3 only -- BUN and creatinine The laboratory measurements of blood urea nitrogen (BUN) and serum creatinine (a component of urine) can be used to indicate the status of a patient's renal function.

Which of the following laboratory measurements can be used to evaluate the renal function of a patient? 1. blood urea nitrogen (BUN) 2. partial thromboplastin time (PTNT) 3. creatinine a. 1 only b. 1 and 2 only c. 1 and 3 only d. 1, 2 and 3

B. creatinine Serum creatinine levels are the most accurate and dependable laboratory measures of renal function. BUN level may be affected by many variables and is not a suitable test of renal function. BUN levels are usually evaluated in conjunction with creatinine levels for a more accurate measurement of renal function

Which of the following laboratory values is the most dependable measure of renal function? a. blood urea nitrogen (BUN) b. creatinine c. prothrombin time (PT) d. partial thromboplastin time (PTT)

D) Transverse Ligament ***The transverse ligament is also termed the cruciform ligament due to its cross-like appearance. It extends across the ring of C1 to form a sling over the odontoid process***

Which of the following ligaments is referred to as the cruciform ligament? A) Ligamentum Flava B) Supraspinous ligament C) Anterior longitudinal ligament D) Transverse ligament

D) Transverse Ligament ***The transverse ligament is also termed the cruciform ligament due to its cross-like appearance. It extends across the ring of C1 to form a sling over the odontiod process***

Which of the following ligaments is referred to as the cruciform ligament? A) Ligamentum Flava B) Supraspinous ligament C) Anterior longitudinal ligament D) Transverse ligament

Air, fat, water, muscle, bone, tooth enamel

Which of the following lists of substances that make up the human body best places them in increasing order of density? Air, fat, water, muscle, bone Bone, muscle, water, fat, air Air, fat, muscle, water, bone Air, fat, water, muscle, bone, tooth enamel

B. iterative reconstruction The filtered back-projection, or convolution, method of image reconstruction had previously been the most common type of CT reconstruction. Today, improved processing speeds have allowed iterative reconstruction to become a primary method of CT image reconstruction. Many CT systems now employ interactive techniques, which offer reduced image noise, allowing for a decrease in technical parameters (mA, kVp) and reduced patient dose.

Which of the following mathematical reconstruction methods may be used by a CT system to decrease image noise and offer patient dose savings? a. convolution b. iterative reconstruction c. Fourier transform d. back-projection

B. 3 only -- iohexol Low-osmolar contrast media (LOCM) are nonionic agents that do not dissociate into charged particles (ions) in solution. Examples of LOCM are iohexol (Omnipaque), iopamidol (Isovue) and ioversol (Optiray)

Which of the following may be considered a low-osmolar contrast medium? 1. iothalamate meglumine 2. diatrizoate sodium 3. iohexol a. 1 only b. 3 only c. 1 and 2 only d. 1, 2 and 3

C. 1 and 2 only -- iothalamate meglumine and diatrizoate sodium High osmolar contrast media (HOCM) are ionic agents that dissociate into charged particles (ions) in solution. Ionic contrast media are salts consisting of sodium and/or meglumine. Iothalamate meglumine and diatrizoate sodium are commonly known by the brand names Conray and Hypaque, respectively. Another HOCM, Gastrografin, is a solution of diatrizoate meglumine and diatrizoate sodium.

Which of the following may be considered one of the high-osmolar contrast media (HOCM)? 1. iothalamate meglumine 2. diatrizoate sodium 3. iohexol a. 1 only b. 3 only c. 1 and 2 only d. 1, 2 and 3

C. 1 and 2 only -- instructing the patient to take nothing by mouth (NPO) for 4 hours prior to exam, and 1200 mL of oral contrast agent administered 90 minutes before the exam Routine preparations for general studies of the abdomen and pelvis include the administration of an oral contrast agent for opacification of the gastrointestinal tract. Patients are typically instructed to refrain from eating or drinking anything else for up to 6 hours before the exam. Polyethylene glycol (PEG) is used for bowel cleansing before a colonoscopy and may have certain applications for CT preparation limited to CT colonography.

Which of the following may be included as part of a routine preparation for a general contrast-enhanced CT examination of the abdomen and pelvis? 1. instructing the patient to take nothing by mouth (NPO) for 4 hours before the exam 2. 1200 mL of oral contrast agent administered 90 minutes before the exam 3. having the patient drink 4 liters of polyethylene glycol (PEG) 24 hours before the exam a. 1 only b. 2 only c. 1 and 2 only d. 1 and 3 only

C. 2 and 3 only -- DLP = MSAD x slice width (cm) x # of slices and DLP = CTDIvol x scan length (cm) Dose length product is a measure of the weighted radiation dose (CTDIw) over a given scan length (CM) and is typically given in units of a subunit of gray (cGy or mGy) per centimeter (cGy-cm). Current CT systems will typically report the DLP as part of the radiation dose structured report (RDSR), as a means of providing an estimation of patient dose for a given CT acquisition. DLP is estimated from either the CTDIvol or the multiple slices, taken from comparable dose measurements using an anthropomorphic phantom.

Which of the following may be used to calculate the dose length product of a CT acquisition? 1. DLP = slice width (cm) x pitch 2. DLP = MSAD x slice width (cm) x # of slices 3. DLP = CTDIvol x scan length (cm) a. 1 and 2 only b. 1 and 3 only c. 2 and 3 only d. 1, 2 and 3

C. 1 and 3 only Epinepherine (Adrenalin) is an adrenergic drug used as a bronchodiolator. Diphenhydramine (Benadryl) may be used to block the physiologic effects of the body's release of histamine, thus reducing the allergic effects of the contrast material. Atropine should be used only to comat bradycardia during a vagal reaction to contrast material.

Which of the following medications may be administered to a patient who is haivng a severe anaphylactoid reaction to iodinated contrast material? 1. epinephrine 2. atropine 3. diphenhydramine a. 1 only b. 1 and 2 only c. 1 and 3 only d. 1, 2 and 3

A. Sublingual nitroglycerin IF NOT CONTRAINDICATED, sublingual nitroglycerin may be administered just before the cardiac multidetector CT (MDCT) study to cause dilation of the coronary vessels, improving their visualization.

Which of the following medications may be administered to dilate the coronary vasculature prior to cardiac CT examination? A) sublingual nitroglycerine B) a beta-adrenergic receptor blocking agent (beta blocker) C) atropine D) albuterol

D. 1, 2 and 3 -- reduce mA, limit phases of acquisition and increase pitch The selected mA should be adapted to each patient according to size and/or weight. The "child'-sizing" of CT protocols is an important method of reducing radiation dose to the pediatric patient. Eliminating unnecessary CT scans at the discretion of the physician is an additional way to reduce pediatric dose.

Which of the following methods may be employed to reduce the radiation dose to the pediatric patient undergoing CT? 1. reduce mA 2. limit phases of acquisition 3. increase pitch a. 1 only b. 1 and 2 only c. 1 and 3 only d. 1, 2 and 3

D. 1.0-mm contiguous slices; reconstructed using both a standard and a bone (i.e., high-contrast) algorithm; with IV contrast; filmed in two separate window settings Acoustic neuromas, or more properly vestibular schwannoma, originate in the internal auditory canal (IAC). Although medium and large acoustic neuromas extend outside the IAC and into the brain cavity, *many of these tumors are quite small and are confined to the IAC. Contrast-enhanced CT using a narrow slice thickness is typically needed to detect small acoustic neuromas.*

Which of the following might be an appropriate protocol for a CT study that is performed for suspected acoustic neuroma? A) 10-mm contiguous slices; standard algorithm; with IV contrast; filmed with a narrow window width B) 8-mm contiguous slices; bone algorithm; without IV contrast; filmed at two separate window settings C) 5-mm slice thickness; 3mm table increment; without IV contrast; soft (low contrast) algorithm; filmed with a lung window D) 1.0-mm contiguous slices; reconstructed using both a standard and a bone (i.e., high-contrast) algorithm; with IV contrast; filmed in two separate window settings

D. Right lateral decubitus

Which of the following most likely describes the patient position during the formation of the image in the figure? a. supine b. prone c. left lateral decubitus d. right lateral decubitus

D) Triceps Muscle ***Because it's associated with 3 heads of proximal attachment***

Which of the following muscles is named because its associated with three heads of proximal attachment of the upper extremity? A) Deltoid muscle B) Brachialis muscle C) Subscapularis muscle D) Triceps muscle

D) Triceps Muscle ***because its associated with 3 heads of proximal attachment***

Which of the following muscles is named because its associated with three heads of proximal attachment of the upper extremity? A) Deltoid muscle B) Brachialis muscle C) Subscapularis muscle D) Triceps muscle

A) Sternocleidomastoid ***This muscle is a broad strap like muscle that originates on the sternum and clavicle and inserts onto the mastoid tip. Its function is turn the head from side to side and to turn the neck***

Which of the following muscles of the neck originates on the sternum and clavicle and inserts onto the mastoid tip? A) Sternocleidomastoid B) Levator scapulae mucsle C) trapezius muscle D) Anterior scalene muscle

A) Sternocleidomastoid ***This muscle is a broad strap like muscle that originates on the sternum and clavicle and inserts onto the mastoid tip. Its function is turn the head from side to side and to turn the neck***

Which of the following muscles of the neck originates on the sternum and clavicle and inserts onto the mastoid tip? A) Sternocleidomastoid B) Levator scapulae mucsle C) trapezius muscle D) Anterior scalene muscle

D. 1, 2 and 3 -- explanation of the examination techniques, the possible risks and benefits of the examination and alternatives to the procedure involved Informed consent must be obtained from the patient before any invasive procedure. The components of informed consent are thorough explanations of the procedure, any possible risks in addition to the proposed benefits, and alternatives to the procedure.

Which of the following must be included when one is obtaining informed consent for an invasive procedure? 1. explanation of the examination techniques 2. the possible risks and benefits of the examination 3. alternatives to the procedure involved a. 1 only b. 1 and 2 only c. 1 and 3 only d. 1, 2 and 3

C) True Vocal Cords ***Are named for the production of sound***

Which of the following neck structures are directly responsible for voice production? A) Vestibular Folds B) Glottic Space C) True Vocal Cords D) Infraglottic Space

B. spatial resolution Small focal spots improve the geometric efficiency of the x-ray beam, leading to greater spatial resolution.

Which of the following parameters or factors is improved by the selection of a smaller x-ray tube filament? a. scan time b. spatial resolution c. heat rating d. signal-to-noise ratio (SNR)

C. DFOV = 35; slice thickness = 0.625mm *An isotropic voxel is a cube, measuring the same in the x,y and z directions.* The pixel size is the x,y size (because pixels are square). The pixel size for the four choices are: A) 0.23, B) 0.49, C) 0.68 and D) 0.86 *For the voxel to be near-isotropic, the pixel size must be close to that of the slice thickness (the z direction).*

Which of the following parameters will produce nearisotropic voxels? A) DFOV = 12; slice thickness = 1mm B) DFOV= 25; slice thickness = 5mm C) DFOV = 35; slice thickness = 0.625mm D) DFOV = 44; slice thickness = 2mm

B) Sphenoid Sinus ***Sphenoid sinus is directly below the sella turcica. On the cross-sectional ct images they are the most posterior***

Which of the following paranasal sinuses is most posterior? A) Cavernous sinus B) Sphenoid sinus C) Mastoid sinus D) Maxillary sinus

A. bronchiectasis Interstitial disease of the lungs are usually diffuse pathologic processes involving the interstitium, or FRAMEWORK, of the lungs. Interstitial lung disease examples: bronchiectasis, emphysema, asbestosis, and sarcoidosis

Which of the following pathologic processes may be considered an interstitial disease of the lungs? a. bronchiectasis b. mediastinal lymphadenopathy c. pulmonary metastasis d. bronchogenic carcinoma

B) Abscess ***majority of abscess appear as loculated fluid collections, often with internal debris, fluid levels, and sometimes air-fluid levels and have a definable wall with irregular thickening***

Which of the following pathological conditions appear as a loculated fluid collection with irregular thickening and fat planes due to inflammation? A) Cyst B) Abscess C) Ascites D) Hemorrhage

A) Salivary Gland Abscess ***submandibular salivary gland abscesses are mucous-filled cysts caused by trauma or obstruction.***

Which of the following pathological conditions appear as hypodense interior masses with a ring of enhancement in its peripheral? A) Salivary gland abscess B) goiter C) Thymus CA D) Carotid stenosis

A) Salivary Gland Abscess ***submandibular salivary gland abscesses are mucous-filled cysts caused by trauma or obstruction.***

Which of the following pathological conditions appear as hypodense interior massess with a ring of enhancement in its peripheral? A) Salivary gland abscess B) goiter C) Thymus CA D) Carotid stenosis

A) Cholesteatoma ***Cholesteatoma appears as a soft tissue homogenous density in the middle ear with thickening of the tympanic membrane.***

Which of the following pathological conditions is characterized by a soft tissue homogeneous mass with focal bone destruction of the middle ear? A) Cholesteatoma B) Cavernous hemangioma C) Acoustic neuroma D) Glomus tumor

B) Orbital fracture ***Orbital imaging should be performed in both the axial and coronal plans due to the pyramidal shape of the bony orbit.***

Which of the following pathological conditions must be performed in two plans, the axial and the coronal, due to the pyramidal shape of there bony structure? A) Mandibular fracture B) Orbital Fracture C) Temporal bone imaging D) Sinus imaging

B) Orbital fracture ***Orbital imaging should be performed in both the axial and coronal plans due to the pyramidal shape of the bony orbit.***

Which of the following pathological conditions must be performed in two plans, the axial and the coronal, due to the pyramidal shape of there bony structure? A) Mandibular fracture B) Orbital Fracture c) Temporal bone imaging D) Sinus imaging

C. 1 and 2 only -- beta blocker and nitroglycerin Beta-Adrenergic receptor blocking agents (beta-blockers) may be used to reduce a patient's heart rate, and sublingual nitroglycerin may be administered to cause dilation of the coronary vessels. The use of these pharmaceuticals may improve the overall visualization of the coronary vessels during a cardiac CT examination.

Which of the following pharmaceuticals may be administered before a cardiac CT procedure in an effort to improve visualization of the coronary vessels? 1. Beta blocker 2. nitroglycerin 3. metformin a. 1 only b. 2 only c. 1 and 2 only d. 1, 2 and 3

D. excretory After a delay of 3 - 15 minutes, excretory phase acquisition demonstrates the urinary tract while it is opacified with a contrast agent. Transitional cell lesions of the urothelium appear as filling defects within the bladder, ureters, and/or renal pelvis.

Which of the following phases of renal contrast enhancement best demonstrates transitional cell carcinoma (TCC) of the bladder? a. early arterial b. corticomedullary c. nephrographic d. excretory

B) Compton Interaction ***Compton interaction occurs between moderate energy (above 75 keV) and outer shell electrons resulting in the ionization of the target atom.***

Which of the following photon interactions produced a photon that is deflected from its original site of interaction? A) Characteristic radiation B) Compton interaction C) Photoelectric interaction D) Deterministic effect

A. 1.0 Pitch values greater than 1 allow for the acquisition of a given scan volume in a shorter time, resulting in a reduction in patient radiation dose. All other factors remaining consistent, the higher the pitch, the greater the dose savings at a cost of x-axis resolution. A multidetector CT (MDCT) system may automatically increase the tube current (mA) when the pitch is increased to maintain a predetermined image noise level. To maintain dose savings when pitch is increased, the operator will need to manually reduce the mA setting or increase the amount of noise tolerated int he reconstructed image.

Which of the following pitch settings would result in the largest radiation dose to the patient? a. 1.0 b. 1.2 c. 1.7 d. 2.0

A. P wave Atrial systole refers to the contraction of the left and right atria. The onset of the P wave on an electrocardiogram corresponds to atrial systole.

Which of the following portions of the electrocardiogram (ECG) corresponds to the period of atrial systole? a. P wave b. QRS complex c. alpha wave d. T wave

B. dose length product (DLP) The CTDIvol is a measure of dose to each slice along a helically acquired volume. The dose length product (DLP) is calculated by multiplying the CTDIvol by the total scan length.

Which of the following radiation dose indices is a measure of the total radiation exposure for an entire series of CT images? a. CTDI100 b. dose length produce (DLP) c. CTDIvol d. multiple scan average dose (MSAD)

B) Filtered back projection ***The filtered back projection of reconstruction uses a method similar to back projection, since it averages the attenuation values for all projections that cross the data point, but performs the process in the spatial frequency domain.***

Which of the following reconstruction algorithm is known as the convolution method? A) Back projection B) Filtered back projection C) Algebraic reconstruction D) Iterative reconstruction

B) Adaptive statistical iterative reconstruction ***is a hybrid algorithm that blends with filtered back projection (FBP). It estimates raw data sets and compares the sets to the measured data sets from the CT system.***

Which of the following reconstruction algorithms models photons and electric noise in the CT system? A) Filtered back projection reconstruction B) Adaptive statistical iterative reconstruction C) Modeled based iterative reconstruction D) Algebraic reconstruction

B) Adaptive statistical iterative reconstruction ***is a hybrid algorithm that blends with filtered back projection (FBP). It estimates raw data sets and compares the sets to the measured data sets from the CT system.***

Which of the following reconstruction algorithms models photons and electric noise in the CT system? A) Filtered back projection reconstruction B) Adaptive statistical iterative reconstruction C) Modeled based iterative reconstruction D) Algebraic reconstruction

D. iterative reconstruction Uses multiple image reconstruction passes (iterations) to arrive at a final image with reduced noise and artifacts, leading to improved overall quality. This reduction in noise allows technical factors to be significantly reduced, resulting in decreased patient radiation dose.

Which of the following reconstruction methods may be employed by a CT system for the purpose of reducing patient radiation dose? a. convolution reconstruction b. interpolation reconstruction c. back-projection reconstruction d. iterative reconstruction

A. Multiplanar reformation (MPR) Demonstrating the same density and contrast as a conventional helically acquired axial image, the coronal display in the figure is a standard MPR image. Multiplanar Reformation (MPR) describes the process of displaying CT images in a different orientation from the one used in the original reconstruction process. The reformatted image is 1 voxel thick, with the pixels facing the viewer, each representing the average attenuation occurring within the represented voxels.

Which of the following reformation techniques best describes the image in the figure? A) Multiplanar reformation (MPR) B) Volume-rendered 3D C) Maximum intensity projection (MIP) D) Minimum intensity projection (min-IP)

D. maximum intensity projection (MIP) MIP images display only the maximum pixel value along a ray traced through the object to the viewer's assumed perspective in front of the viewing monitor. Tissues with lower attenuation values are not displayed, leaving high-attenuation structures such as bone and contrast-enhanced soft tissue structures free of superimposition.

Which of the following reformation techniques best describes the image in the figure? A) volume-rendered 3D B) multiplanar-reformation (MPR) C) minimum intensity projection (min-IP) D) maximum intensity projection (MIP)

A) Parotid gland ***Parotid gland contains a fair amount of fat, which displays as hypodense structures on cross sections CT images***

Which of the following salivary glands are seen as hypodense structures on cross-sectional images? A) Parotid gland B) Subbuccal gland C) Submandibular gland D) Sublingual gland

D. 0.625 mm sections, 32 cm DFOV Thin-section reconstruction using the smallest display field of view (DFOV) possible results in the smallest pixel and voxel dimensions. Voxels with equal dimensions along the x, y and z axes are described as ISOTROPIC.

Which of the following sets of technical factors would yield an isotropic data set when a multidetector CT (MDCT) image is reconstructed using a 512^2 matrix? a. 5.0 mm sections, 17 cm DFOV b. 3.0 mm sections, 25 cm DFOV c. 1.25 mm sections, 48 cm DFOV d. 0.625 mm sections, 32 cm DFOV

B. 1.2 mg/dL; 14 mg/dL Normal range for creatinine: 0.5 - 1.5 mg/dL Normal range for blood urea nitrogen (BUN): 7 - 25 mg/dL Creatinine and BUN are laboratory measurements that may be used to evaluate the renal function of patient's scheduled to undergo a CT examination that involves intravenous administration of an iodinated contrast agent.

Which of the following sets of values would be considered normal levels for creatinine and blood urea nitrogen (BUN) respectively? a. 3.1 mg/mL; 1.2 mg/dL b. 1.2 mg/dL; 14 mg/dL c. 0.7 mg/dL; 39 mg/dL d. 5.2 mg/dL; 11 mg/dL

A. 19 gauge Larger-lumen (lower-gauge) needles allow for more rapid intravenous administration of contrast. The lumen is considerably larger in a 19-gauge needle than in a 25-gauge needle. The viscosity of contrast material necessitates the use of a relatively large-bore needle.

Which of the following sizes of butterfly needles allows for the most rapid administration of iodinated intravenous contrast media? a. 19 gauge b. 21 gauge c. 23 gauge d. 25 gauge

C. increases in collimation increase the intensity of the primary beam Collimation occurs as lead shutters close down upon the beam, limiting its projected area. Constructed of lead, the collimator shutters absorb portions of the primary beam, thereby reducing its intensity.

Which of the following statements about collimation of the CT x-ray beam is false? A) collimation of the x-ray beam occurs both before and after the patient B) collimation of the beam occurs in the z-axis, thus affecting slice thickness C) increases in collimation increase the intensity of the primary beam D) collimation of the CT x-ray beam is used to limit the detection of scatter radiation

B. The scintillation detector has a higher capture efficiency Because of its solid nature, the scintillation detector interacts with a higher percentage of incident x-ray photons, giving it a better capture efficiency.

Which of the following statements comparing the efficiency of scintillation and gas ionization detectors is correct? A) Both have approximately the same capture efficiency B) The scintillation detector has a higher capture efficiency C) Unlike the scintillation detector, the gas ionization detector has a problem with afterglow D) Gas ionization detectors have a higher conversion efficiency

B. MSAD increases with overlapping scans During axial scanning, overlapping scans increase the patient radiation dose, whereas gaps between slices decreases the dose.

Which of the following statements is true regarding the relationship between multiple scan average dose (MSAD) and image spacing during axial (step-and-shoot) scanning? A) MSAD decreases with overlapping scans B) MSAD increases with overlapping scans C) MSAD increases with noncontiguous scans D) MSAD equals the product of image spacing and pitch

D. Predetector collimation removes scatter radiation before it reaches the detector The purpose of PREDETECTOR or POSTPATIENT COLLIMATION are to remove scatter radiation and to shape the portion of the beam that is incident on each detector.

Which of the following statements regarding predetector collimation of the CT x-ray beam is true? A) Predetector collimation reduces patient radiation dose B) Predetector collimation reduces the production of scatter radiation C) Predetector collimation determines the scan field of view (SFOV) D) Predetector collimation removes scatter radiation before it reaches the detectors

D) Inferior mesenteric arteries ***The kidneys are retroperitoneal bean shaped organs located in the RUQ and LUQ. The kidneys along with the renal arteries and veins are embedded in perirenal fat in an attempt to prevent bumps and jolts to the body.***

Which of the following structure is not surrounded by perirenal fat? A) Superior poles of both kidneys B) Renal Arteries C) Renal Veins D) Inferior mesenteric arteries

D) Inferior mesenteric arteries ***The kidneys are retroperitoneal bean shaped organs located in the RUQ and LUQ. The kidneys along with the renal arteries and veins are embedded in perirenal fat in an attempt to prevent bumps and jolts to the body.***

Which of the following structure is not surrounded by perirenal fat? A) Superior poles of the kidneys B) Renal Arteries C) Renal Veins D) Inferior mesenteric arteries

A) tricuspid valve

Which of the following structures is demonstrated by label #70? a) tricuspid valve b) semilunar aortic valve c) semilunar pulmonary valve d) mitral valve

D. 2 and 3 only -- image segmentation and physiologic gating Technical adjustments to improve the temporal resolution of the CT system may help reduce involuntary motion artifacts. Reductions in scan time (gantry rotation), image segmentation, and physiologic gating can be used to reduce the effects of involuntary motion.

Which of the following technical adjustments may be employed to improve the temporal resolution of a multidetector CT (MDCT) system? 1. decrease in section width 2. image segmentation 3. physiologic gating a. 1 only b. 3 only c. 1 and 2 only d. 2 and 3 only

B. 1 and 2 only -- acquisition of an isotropic data set and overlapping section increment In modern MDCT systems, the highest quality 3D and multiplanar reformation (MPR) images are obtained within the acquisition of an isotropic or near-isotropic data set. The other equally important piece is an overlapping data set, for which the reconstruction interval is less than the section width.

Which of the following technical adjustments may be employed to reduce step artifact in a multiplanar reformation (MPR) CT image? 1. acquisition of an isotropic data set 2. overlapping section increment 3. increase in effective mAs a. 1 only b. 1 and 2 only c. 2 and 3 only d. 1 , 2 and 3

C. decrease of tube rotation from 360 - 180 degrees Reducing the total rotation of the x-ray tube reduces the scan time and patient dose. This type of partial scan, commonly called a "half-scan" or "segmenting", is most often used to improve the temporal resolution of the acquisition.

Which of the following technical changes would serve to decrease patient radiation dose during a CT examination? A) increase in matrix size B) change from soft tissue to bone algorithm C) decrease of tube rotation from 360 to 180 degrees D) decrease in display field of view (DFOV)

C. mAs Image quality aside, reductions in mA and/or scan time (seconds) are direct methods of decreasing patient dose during a CT examination.

Which of the following technical factors has a direct effect on patient dose? a. matrix size b. algorithm c. mAs d. window level

D. 2 and 3 only -- beam collimation and detector configuration The thickness of the reconstructed section is controlled primarily by the detector configuration in a multidetector CT (MDCT) system. Beam collimation also exerts an indirect effect on section width by controlling the portion of the detector array exposed to transmitted radiation.

Which of the following technical factors is/are involved in the determination of section width for multidetector CT (MDCT) images? 1. scan field of view (SFOV) 2. beam collimation 3. detector configuration a. 2 only b. 1 and 2 only c. 1 and 3 only d. 2 and 3 only

C. prospective ECG gating During MDCT cardiac studies, prospective gating can be used to reduce patient radiation dose. In this technique, electrocardiogram (ECG)-triggered tube current modulation allows for pulses of x-ray energy to be used rather than continuous exposure. Both retrospective ECG gating and multisegment reconstruction require constant radiation exposure throughout multiple heartbeats.

Which of the following techniques may be employed to reduce patient radiation dose during a cardiac CT examination? a. retrospective electrocardiogram (ECG) gating b. z-axis interpolation c. prospective ECG gating d. multisegment reconstruction

B. 3 only -- scanning while the patient phonates the letter "E" Phonation during acquisition may be used to demonstrate abnormal mobility of the vocal cord(s). A thin-section (0.5 - 2 mm) sequence is obtained during which the patient is instructed to phonate a low, stead "E" sound during the entire scan duration.

Which of the following techniques may be used to improve visualization of the vocal cords during a CT examination of the neck? 1. oral administration of thick parium paste 2. CT acquisition with the mouth open as widely as tolerable 3. scanning while the patient phonates the letter "E" a. 2 only b. 3 only c. 1 and 3 only d. 1, 2 and 3

B. hyperdense Subdural hematomas are collections of blood that occur throughout the subdural space after traumatic injury to the head. An acute subdural hematoma is one that has manifested clinically during the first 24 hours after the injury. During this stage, the hematoma appears hyperdense compared with normal brain tissue because of the initial clotting that has occurred and the concentration of hemoglobin in fresh blood.

Which of the following terms describes the appearance of an acute subdural hematoma on a CT image of the brain? a. radiolucent b. hyperdense c. hypodense d. isodense

A. Leiomyoma A benign mass of smooth muscle. Leiomyomas commonly occur in the uterus and are readily identified on CT examinations as bulky, nonspecific uterine enlargement(s).

Which of the following terms is used to describe a fibroid tumor of the uterus? a. leiomyoma b. cystic teratoma c. endometriosis d. corpus luteum

B. Dyspnea Dysphagia: difficulty swallowing Dysphasia: language disorder marked by deficiency in the generation of speech, and sometimes also in its comprehension, due to brain disease or damage.

Which of the following terms is used to describe a patient who is having difficulty breathing? a. dyslexia b. dyspnea c. dysphagia d. dysphasia

B. algorithm As a set of rules or steps used to solve a mathematical problem. The programs used by the CT computer to reconstruct the image are often referred to as algorithms.

Which of the following terms is used to describe a set of rules for solving a mathematical problem? a. reconstruction b. algorithm c. function d. array

B. bolus Bolus injection: when the entire volume of medication or contrast agent is administered at once over a short period of time.

Which of the following terms is used to describe the intravenous injection of medication or contrast agent in one complete dose over a short time? a. infusion b. bolus c. IV drip d. Infus-A-Port

D. photon influence Described as the quantity of x-radiation passing through a unit area.

Which of the following terms may be used to describe the quantity of radiation emitted from the CT x-ray tube toward the patient? a. effective mAs b. photon flux c. constant mAs d. photon influence

B. barium sulfate Barium sulfate is not water soluble and is not easily absorbed by the body. If barium sulfate leaks out of the digestive tract because of perforation, this agent may cause peritonitis.

Which of the following type(s) of oral contrast material could cause peritonitis if leakage from the digestive tract occurs from perforation? a. iopamidol (Gastrografin) b. barium sulfate c. diatrizoate (Hypaque) d. effervescent granules

D) Distributed processing ***processing information by more than one computer connected by a network system. It's characterized by having each computer perform a particular task and the combined work contributes to a large goal.***

Which of the following types of CT computer architecture requires a higher structure environment that allows hardware and software to communicate, share resources and exchange information freely? A) Parallel processing B) Serial processing C) Multiprocessing D) Distributed processing

D) Distributed processing ***processing information by more than one computer connected by a network system. Its characterized by having each computer perform a particular task and the combined work contributes to a large goal.***

Which of the following types of CT computer architecture requires a higher structure environment that allows hardware and software to communicate, share resources and exchange information freely? A) Parallel processing B) Serial processing C) Multiprocessing D) Distributed processing

C. Enteric precautions Attempt to protect from the spread of injection through direct or indirect contact with fecal matter. Gowns and gloves are common protective devices used for enteric precautions. Surgical masks NOT warranted.

Which of the following types of isolation techniques protects against infection transmitted through fecal material? a. acid-fast bacillus isolation b. contact isolation c. enteric precautions d. drainage-secretion precautions

D. mGy-cm (milligrays per centimeter) Dose length product (DLP) is expressed in units of milligrays per centimeter. DLP is the total patient dose over a given scan acquisition length (z) and can be illustrated as the product of CTDIvol and scan length.

Which of the following units is used to express the total patient dose from a helically acquired CT examination? a. roentgen (R) b. curie (Ci) c. R-cm (roentgens per centimeter) d. mGy-cm (milligrays per centimeter)

C) Vertebral arteries ***Vertebral arteries are a branch of the subclavian arteries and join to form the basilar artery. They run bilaterally through the transverse foramina from C7 through C1***

Which of the following vessels does not contribute to the vascular supply to the brain? A) Posterior cerebral artery B) Anterior communicating artery C) Vertebral artery D) Internal carotid artery

A. increase mA from 180 to 360 Increased pitch, noise level (index), and table speed will all result in a measurable radiation dose savings. Increasing the tube current (milliamperage, mA) will result in a direct and proportionate increase in tube output and subsequent patient radiation dose.

Which of the following will NOT reduce patient exposure during CT? a. increase mA from 180 to 360 b. increase pitch from 1.0 to 1.5 c. increase tolerated noise level (index) d. increase table speed from 32 to 48 mm/sec

A. WL -300, WW 1000 A wide window setting (WL -300, WW 1000) may be used to properly demonstrate the small soft tissue vocal structures from the surrounding air-filled endolarynx. WL = window length; WW = window width

Which of the following window settings was most likely used to display the image in the figure? a. WL -300, WW 1000 b. WL -50, WW 50 c. WL +50, WW 150 d. WL +400, WW 3000

C. WL +50, WW 400 With the lung fields black, the bony structures white, and the mediastinum and muscles gray, the image in the figure is displayed in a soft tissue window.

Which of the following window settings was used to display the image in the figure? a. WL -400, WW 1500 b. WL 0, WW 150 c. WL +50, WW 400 d. WL +275, WW 2500

D. angiocatheter Angiocatheters and higher-gauge needles are preferred for administration of a contrast agent with the use of a power injector. The stability of an angiocatheter maintains proper placement within the vein while withstanding the high pressure applied by the power injector.

Which of the following would be best suited for intravenous injection of contrast material with a power injector? a. butterfly needle b. central venous line c. 23-gauge spinal needle d. angiocatheter

A. good patient-technologist communication Thorough communication between the technologist and the patient is vital in ensuring superior examination quality. If the patient has been informed of the examination process, his or her anxiety may be reduced and cooperation improved. During all CT procedures, the patient must be instructed to hold still in an effort to reduce motion artifact on the CT images.

Which of the following would be considered the best method to reduce respiratory motion on the CT image? a. good patient-technologist communication b. reduced scan times c. immobilization devices d. glucagon administration

A. metallic dental fillings

Which of the following would be the most common cause of the artifact in the figure? a. metallic dental fillings b. partial volume averaging c. beam hardening d. detector malfunction

B. 1 and 2 only -- change to small focal spot size and acquire 2.5-mm sections The spatial resolution of a CT examination can be improved with the use of small focal spots, narrow sections, and large matrices.

Which of the following would serve to increase spatial resolution of a CT examinations using a large focal spot size, 5-mm sections, and a 512^2 matrix? 1. change to small focal spot size 2. acquire 2.5-mm sections 3. reconstruct images in a 320^2 matrix a. 1 only b. 1 and 2 only c. 1 and 3 only d. 1, 2 and 3

B. 1 and 2 only -- increase in filtration and kVp Streaking edge gradient artifacts may be reduced with increases in the beam's average photon energy. Increases in kVp and filtration would accomplish this. This type of artifact may also be reduced with a decrease in reconstructed section width, which helps minimize the edge gradient effect. A decrease in partial volume averaging may lessen the streaking that is apparent form edge gradient inconsistencies.

Which of the following would serve to reduce the image artifact present in the figure? 1. increase in filtration 2. increase in kVp 3. increase in section width a. 1 only b. 1 and 2 only c. 1 and 3 only d. 1, 2, and 3

Photoelectric

Which photon-tissue interaction makes radiography possible because of its creation of contrast?

Compton

Which photon-tissue interaction produces radiation that may expose others in the room during fluoroscopy?

A. 1 The image noise is equal to the standard deviation of pixel values within an ROI measurement of the image. ROI number 1 on the figure demonstrates the highest value of standard deviation (SD); SD = 2.87.

Which region of interest (ROI) on the figure demonstrates the greatest amount of image noise? A) 1 B) 2 C) 3 D) impossible to determine from the information provided

C. cone-beaming As the number of detector channels increases, a wider collimation is required and the x-ray beam becomes cone-shaped, rather than fan-shaped. This can result in cone-beam artifacts, which are more pronounced for the outer detector rows. *The larger the cone beam (i.e. more detector channels), the more pronounced the effect.*

Which type of artifact is only present on images produced from MDCT systems? A) edge gradient B) aliasing C) cone-beam D) tube arcing

C. 1 and 3 only -- Any prior allergic reaction to contrast media and history of asthma These factors increase the incidence of adverse reactions to iodinated contrast media. It is therefore vital to question the patient regarding these factors before contrast agent administration. The presence of the human immunodeficiency virus (HIV) or hepatitis does not increase the incidence of adverse reactions to iodinated contrast media. Because universal precautions govern the protection of all patients and health care professionals, there is no need to question the patient regarding these factors.

While one is obtaining a thorough history from a patient before the IV injection of an iodinated contrast material, which of the following topics should be included? 1. any prior allergic reactions to contrast media 2. presence of HIV or hepatitis infection 3. history of asthma a. 1 only b. 1 and 2 only c. 1 and 3 only d. 1, 2 and 3

C. effective dose Effective dose approximates the relative risk from exposure to low doses of ionizing radiation. The unit of measurement for effective dose is the Sievert (Sv). Different tissues are assigned weighting factors on the basis of their individual radiosensitivities.

Whole-body risk based upon the radiosensitivity of exposed tissues may be estimated using the unit: a. stochastic dose b. kerma c. effective dose d. CT dose index (CTDI)

B) scanners offer more mA options and the effect of mAs on image quality is more predictable

Why is it more common practice to manipulate the mAs, rather than the kVp, for a CT examination? A) changing the mAs does not affect the radiation dose to the patient B) scanners offer more mA options and the effect of mAs on image quality is more predictable C) tube cooling is not affected by adjusting the mAs settings D) changing the mAs will not affect the quality of the images

A. to minimize streaks Streaks are inherent in the back-projection process; therefore, it is essential to apply a mathematical filtering process before back projection.

Why is it necessary to apply a filter function to an attenuation profile to create a CT image? A) to minimize streaks B) to increase the signal-to-noise ratio C) to eliminate ring artifact D) to reduce radiation exposure to the patient

A. because of narrow collimation used in CT, not much radiation is scattered to regions outside the selected scan area Because the x-ray beam does not typically scatter great distances in CT, shielding in not nearly as effective in reducing the radiation dose in the patient as it is in general radiography. However, shielding in CT is not without benefit and is particularly useful in demonstrating to the patient (or the patient's family) that everything possible is being done to minimize the risks from radiation exposure.

Why is lead-shielding less beneficial in CT than it is in general radiography? A) because of narrow collimation used in CT, not much radiation is scattered to regions outside the selected scan area B) because of the high kVp used in CT, photons will penetrate a lead shield C) scatter radiation is excessive in CT and is not controlled by shielding D) radiosensitive organs such as the thyroid gland can never be shielded without impairing the scans

C. write-once, read-many (WORM) Describes certain CT and DVD media capable of storing data only once. The stored images can be read multiple times, but once recorded on , the media may not be reused.

Writable CT and DVD media are optical disks that may be described as: a. read-only b. lossy c. write-once, read-many (WORM) d. lossless

Half-value layer

X-ray beam quality is expressed in terms of:

Ionization

X-rays may remove electrons from atoms in the body by a process called:

Photons

X-rays travel as bundles of energy called:

A. Coumadin Coumadin/warfarin belongs to the class of medications called anticoagulants. Anticoagulants are used to treat a variety of health conditions including stroke, and inhibit coagulation (clotting).

You complete the CT examination and will remove the IV catheter. Which of the following medications may necessitate holding pressure on the puncture site for a longer period? a. Coumadin b. Benadryl c. Tylenol d. Synthroid

mAs

the amount of darkness on a radiograph is controlled primarily by:

C. 100 seconds The total injection time may be calculated by dividing the contrast agent volume by the flow rate. In this example, the flowrate is 1.5 mL/sec, for a total of 150 mL. 150 mL / 1.5 mL/sec = 100 seconds

the injection rate of an automatic injector is set at 1.5 mL/sec. What is the injection time for a contrast agent volume of 150 mL? a. 60 seconds b. 90 seconds c. 100 seconds d. 120 seconds

B) 50 mSv per year

what is the dose limitation for radiation workers in the United States per year? A) 20 mSv B) 50 mSv C) 75 mSv D) 100 mSv


Kaugnay na mga set ng pag-aaral

NUR 3420 Pharmacology PrepU Chapter 49

View Set

EMT B: Ch30 - Chest Injuries TEST Q

View Set

CDX - Safety and Foundation Pretest, Chapters 1-2

View Set

Chapter 06: Nursing Care of Mother and Infant During Labor and Birth

View Set

NCLEX Cardiovascular, Hematologic, and lymphatic

View Set

Chapter 21 Teacher and Counselor Funds prepU

View Set